Preview (15 of 140 pages)

NCLEX RN ACTUAL EXAM TEST BANK OF REAL QUESTIONS &
ANSWERS
NCLEX 2023/2024 graded A
Part 4
QUESTION 601
During the admitting mental health assessment, a client demonstrates involuntary muscular
activity. He has a marked facial tic around the mouth that is distracting to the nurse during the
interview. The nurse recognizes the behavior and documents it as:
A. Dyskinesia
B. Akathisia
C. Echopraxia
D. Echolalia
Answer: A
Explanation:
(A) The client is demonstrating dyskinesia, which is involuntary muscular activity, such as
tic, spasm, or myoclonus.
(B) Akathisia is regular rhythmic movements usually of the lower limbs, such as constant
motor restlessness.
(C) Echopraxia is mimicking the movements of another person.
(D) Echolalia is mimicking the speech of another person.
QUESTION 602
A client’s wife is concerned over his behavior in recent months. He has been diagnosed with
Parkinson’s disease, and she is telling his nurse that he has been doing “strange things”. The
nurse reassures the wife that the following behavior is normal with Parkinson’s disease:
A. “Your husband will experience some periods of muscle flaccidity. Be sure to make him sit
down during these periods.”
B. “Your husband may move his hands in motions that look like he is rolling a pill between
his fingers.”
C. “Twitching of the muscles is to be expected and can occur at any time during the day.”
D. “Parkinson’s disease causes severe pain in the joints. You should give your husband
Tylenol at those times.”

Answer: B
Explanation:
(A) Clients with Parkinson’s disease generally experience stiffness and rigid movement.
(B) Pill-rolling movements are a symptom experienced by the Parkinson client.
(C) Twitching of the muscles is not an expected symptom of Parkinson’s disease.
(D) Parkinson’s disease does not cause joint pain. Mild muscular pain may be present.
QUESTION 603
A 45-year-old male client experiences a sense of depression because he has not yet achieved
his life’s goals. His career has not been satisfying. He is still looking for the right job. His
wife spends too much money, and his children seem to ignore him while being very selfish.
He is tired of all of their attitudes and is considering buying a red Corvette convertible. While
obtaining these data concerning the client’s feelings about his life, the nurse is able to
determine he is experiencing what psychological crisis according to Erikson’s stages?
A. Identity versus role confusion
B. Integrity versus despair
C. Intimacy versus isolation
D. Generativity versus self-absorption
Answer: D
Explanation:
(A) Identity versus role confusion is experienced by adolescents making the transition from
childhood to adulthood as they attempt to develop a sense of identity.
(B) Integrity versus despair is experienced by the elderly as they reflect on their life in an
attempt to find meaning.
(C) Intimacy versus isolation is experienced by young adults as they establish intimate bonds
of love and friendship.
(D) Generativity versus self-absorption is experienced by middle-aged adults as they fulfill
life goals that involve family, career, and society. The client is experiencing this crisis.
QUESTION 604
To prevent thrombophlebitis in a client on complete bed rest, the nursing care plan should
include:
A. Dangle the client’s legs over the edge of the bed every shift.
B. Massage the client’s calves briskly every shift.

C. Keep the client’s legs extended and discourage any movement.
D. Have the client tighten and relax leg muscles several times daily.
Answer: D
Explanation:
(A) Dangling the client’s legs over the edge of the bed will contribute to stasis and pooling of
blood and increases the risk of thrombus formation.
(B) Massaging the client’s calves could result in dislodging an embolus.
(C) Decreased movement will contribute to pooling of blood and increased risk of venous
thrombosis.
(D) Tightening and relaxing leg muscles increases circulation and decreases the risk of
venous thrombosis.
QUESTION 605
Which behavior by a female client feeding her newborn demonstrates that she needs more
teaching related to safety and infant feeding?
A. She uses the bulb syringe to help clear her baby’s nose when milk is regurgitated.
B. She places her infant on her right side after feeding her.
C. She props the bottle in the crib to feed her baby, which allows her to write birth
announcements and feed her baby at the same time.
D. She burps her baby by placing her in a sitting position, supporting her head and neck and
gently massaging her back.
Answer: C
Explanation:
(A) This practice is the proper use of the bulb syringe to clear the infant’s airway in case of
regurgitation.
(B) Placing the infant on either side or on the stomach prevents aspiration of regurgitated
milk.
(C) “Bottle propping is an unsafe practice because it increases the likelihood of aspiration.”
(D) This practice is one correct way of burping an infant.
QUESTION 606
A 4 days postpartum client who is gravida 3, para 3, is examined by the home health nurse
during her first postpartum home visit. The nurse notes that she has a pink vaginal discharge

with a serosanguineous consistency. The nurse would most accurately chart the client’s lochia
as:
A. Rubra
B. Rosa
C. Serosa
D. Alba
Answer: C
Explanation:
(A) Lochia rubra is bloody with clots and occurs 1–3 days postpartum.
(B) There is no such term as lochia rosa.
(C) Lochia serosa is a pink-brown discharge with a serosanguineous consistency that occurs
4–9 days postpartum.
(D) Lochia alba is yellow to white in color and occurs approximately 10 days postpartum.
QUESTION 607
A male client seeks counseling after his wife of 19 years threatened to divorce him. For most
of their marriage, he has physically and verbally abused her. When asked about his behavior
in the process of the nursing assessment, the client states, “I was mean to my wife because
she insists on cooking meals and wearing clothes that I do not like.” This defense mechanism
is an example of:
A. Repression
B. Regression
C. Reaction formation
D. Rationalization
Answer: D
Explanation:
(A) Repression is blocking a desire from conscious expression. The client is conscious of his
desires.
(B) Regression is returning to an earlier form of expression, which is not demonstrated here.
(C) Reaction formation is acting out the opposite of true feelings. The client felt anger
concerning his wife’s cooking and acted out his feelings.
(D) Rationalization is unconsciously falsifying an experience by giving a “rational
explanation.” The client is attempting to justify his behavior by giving an explanation.

QUESTION 608
A female client has experienced varying degrees of depression throughout her life. Now that
she is postmenopausal, her depression has increased. She is unable to motivate herself to
clean her house or even to get out of bed and get dressed in the morning. The client was
begun on fluoxetine (Prozac) therapy. When educating her about fluoxetine, what might the
nurse caution her about?
A. A daily dose of fluoxetine may be taken in the morning or evening.
B. Fluoxetine is not sedating; therefore, restrictions on driving and other hazardous activities
are not necessary.
C. Rashes or pruritus usually occur early in the therapy and are treatable without
discontinuing the medication.
D. It is safe to take over-the-counter or other prescription medications with fluoxetine.
Answer: C
Explanation:
(A) A daily dose of fluoxetine should be taken in the morning. Afternoon doses may cause
nervousness and insomnia.
(B) Although fluoxetine is less sedating than other antidepressants, it may still cause dizziness
or drowsiness in some clients. The nurse should caution clients to avoid driving or hazardous
activities until the central nervous system effects of the drug are demonstrated.
(C) Rashes or pruritus do commonly occur early in therapy and respond to antihistamines or
topical corticosteroids.
(D) Advise the client not to take over-the-counter or other prescription drugs without
consulting with the physician. Fluoxetine does interact with other common drugs such as
monoamine oxidase inhibitors, diazepam, insulin, oral antidiabetic agents, tricyclic
antidepressants, and tryptophan.
QUESTION 609
A male client is experiencing extreme distress. He begins to pace up and down the corridor.
What nursing intervention is appropriate when communicating with the pacing client?
A. Ask him to sit down. Speak slowly and use short, simple sentences.
B. Help him to recognize his anxiety.
C. Walk with him as he paces.
D. Increase the level of his supervision.

Answer: C
Explanation:
(A) The nurse should not ask him to sit down. Pacing is the activity he has chosen to deal
with his anxiety. The nurse dealing with this client should speak slowly and with short, simple
sentences.
(B) The client may already recognize the anxiety and is attempting to deal with it.
(C) Walk with the client as he paces. This gives support while he uses anxiety-generated
energy.
(D) Increasing the level of supervision may be appropriate after he stops pacing. It would
minimize self-injury and/or loss of control.
QUESTION 610
A 1-year-old child is to receive an IM injection ordered by his pediatrician. He has fallen
asleep in his mother’s arms when the nurse approaches. Which approach is most appropriate
at this time?
A. Give the injection in the vastus lateralis site before the child awakens.
B. Awaken the child first and give the injection in the ventrogluteal site.
C. Awaken the child first and give the injection in the dorsogluteal site.
D. Ask the mother to place the child on the examination table and leave the room, and then
give the injection in an appropriate site.
Answer: B
Explanation:
(A) If awakened first, the child will know that nothing painful will be done without the child
being alerted.
(B) The ventrogluteal site is a safe site for children because it is a large muscle free of major
nerves and blood vessels.
(C) The dorsogluteal site is not recommended in children who have not been walking for at
least 1 year because the muscle is not fully developed.
(D) The parent will be able to offer support and comfort during and after the injection.
QUESTION 611
A female client is seeking counseling for personal problems. She admits to being very
unhappy lately at both home and work. During the nursing assessment, she uses many

defense mechanisms. Which statement or action made by the client is an example of adaptive
suppression?
A. “I did not get the raise because my boss does not like me.”
B. “I felt a lump in my breast 2 weeks ago. I put off getting it checked until after my sister’s
wedding.”
C. “My son died 3 years ago. I still cannot bring myself to clean out his room.”
D. “My husband told me this morning that he wants a divorce. I am upset, but I cannot
discuss the matter with him until after my company’s board meeting today.”
Answer: D
Explanation:
(A) This statement is an example of adaptive rationalization. She is coping with her
disappointment by rationalizing. This is adaptive because no harm is done to self or others. It
is used to protect her ego.
(B) This is an example of maladaptive suppression. She is suppressing the seriousness of the
lump. It is maladaptive because delaying treatment will cause harm to her.
(C) The client’s actions are an example of maladaptive denial. She is denying her son’s death
by not facing his possessions. Until she faces his death, she cannot face reality.
(D) This is an example of adaptive suppression. She realizes the impact of her husband’s
statement but delays discussion until she can devote her full attention to the matter.
QUESTION 612
A young child has been placed in a spica cast. The chief concern of the nurse during the first
few hours is:
A. Prevention of neurovascular complications
B. Prevention of loss of muscle tone
C. Immobilization of the affected limb
D. Using heated fans to dry the cast
Answer: A
Explanation:
(A) Because the extremity may continue to swell and the cast could constrict circulation, the
nurse should elevate the limb and observe for capillary refill, warmth, mobility of toes and
circulation.
(B) Although muscle tone may diminish over time in the affected limb, this is not the
immediate concern.

(C) The limb has been immobilized already by the cast, and therefore immobilization is not a
concern.
(D) Heated fans and dryers are discouraged because the outside cast will dry quickly, yet the
area beneath the cast remains wet and could cause burns.
QUESTION 613
Newborns are routinely screened for phenylketonuria. The nursery nurse ensures that this
screening test is performed:
A. Immediately after birth, because the most accurate result is obtained at this time
B. After 2–3 days of milk ingestion
C. At 2–3 days of age regardless of amount of milk feedings
D. At 1 month, because the biochemical buildup of phenylalanine takes 1 month to detect
Answer: B
Explanation:
(A) The infant has not ingested any protein immediately after birth, which is necessary to
detect excessive serum phenylalanine.
(B) It is important that the infant take in 2–3 full days of milk or formula feedings to preclude
a false-negative reading.
(C) At 2–3 days of age, inadequate milk could have been ingested owing to a delay in the
initial feeding.
(D) The biochemical buildup of serum phenylalanine is detectable after 2–3 days of milk or
formula ingestion.
QUESTION 614
A female client plans to bottle-feed her newborn. Her physician has ordered bromocriptine
(Parlodel) to suppress lactation. Which of the following instructions about bromocriptine
should be given by the nurse?
A. Bromocriptine stimulates the production of prolactin.
B. Hypertension is a primary side effect.
C. Bromocriptine is generally taken for 5 days.
D. Her blood pressure must be stable before starting bromocriptine.
Answer: D
Explanation:
(A) Bromocriptine inhibits the secretion of prolactin.

(B) Hypotension is a side effect of this drug; hypertension is not.
(C) Bromocriptine is generally taken for 14 days.
(D) The administration of bromocriptine is delayed at least 4 hours postpartum and given
only when the client’s blood pressure is stable, because it can cause hypotension and syncope.
QUESTION 615
For the past several months, an elderly female client with Alzheimer’s disease has
experienced paranoia; hallucinations; and aggressive, disruptive behavior. The family is
utilizing haloperidol as needed to control her behavior. On nursing assessment, you note that
the client demonstrates involuntary movements of the tongue and fingers. This may most
likely indicate:
A. Tardive dyskinesia, which may be a side effect of antipsychotic medication
B. Early symptoms of Parkinson’s disease
C. A more advanced stage of Alzheimer’s disease than previously experienced by the client
D. The need to change her medication from haloperidol to another antipsychotic drug to
lessen symptoms
Answer: A
Explanation:
(A) Tardive dyskinesia is a common side effect of antipsychotic medications such as
haloperidol. Discontinuing the medication can alleviate symptoms.
(B) Although mild tremors are an early sign of Parkinson’s disease, haloperidol must be
discontinued first and the client further evaluated.
(C) These symptoms do not necessarily indicate a more advanced stage of Alzheimer’s
disease.
(D) Most antipsychotic drugs are chemically similar and will produce the same side effects.
QUESTION 616
A client is receiving peritoneal dialysis. He has been taught to warm the dialyzing fluid prior
to instilling it because:
A. Warmed solution helps keep the body temperature maintained within a normal range
during instillation
B. Warmed solution helps dilate the peritoneal blood vessels
C. Warmed solution decreases the risk of peritoneal infection
D. Warmed solution promotes a relaxed abdominal muscle

Answer: B
Explanation:
(A) Instilling a cool solution does not significantly lower the body temperature during
peritoneal dialysis.
(B) Warmed solution does help dilate the peritoneal blood vessels, facilitating the exchange of
fluids.
(C) Warming the dialysate does not decrease the risk of peritoneal infection. Sterile technique
decreases this risk.
(D) Relaxing the abdominal muscles does not facilitate peritoneal dialysis.
QUESTION 617
A female client comes for her second prenatal visit. The nurse-midwife tells her, “Your blood
tests reveal that you do not show immunity to the German measles.” Which notation will the
nurse include in her plan of care for the client? “Will need . . .”
A. Rh-immune globulin at the next visit
B. Rh-immune globulin within 3 days of delivery
C. Rubella vaccine at the next visit
D. Rubella vaccine after delivery on the day of discharge
Answer: D
Explanation:
(A) Rh immune globulin is given to Rh-negative mothers to prevent the maternal Rh immune
response.
(B) Rh immune globulin is given to Rh-negative mothers to prevent the maternal Rh immune
response.
(C) The rubella vaccine is not given during pregnancy because of its teratogenicity.
(D) Nonimmune mothers are vaccinated early in the postpartum period to prevent future
infection with the rubella virus.
QUESTION 618
An infant weighing 15 lb has just been treated for severe diarrhea in the hospital. Discharge
instructions by the nurse will include maintenance fluid requirements for the pediatric client.
Which of the following values best indicates the nurse’s understanding of normal fluid
requirements for this infant?
A. 240 mL/day

B. 680 mL/day
C. 330 mL/day
D. 960 mL/day
Answer: B
Explanation:
(A, C, D) These answers are incorrect.
(B) Normal fluid requirement for this pediatric client is based on the fact that 0–10 kg of
weight equals 100 mL/kg per day. This infant weighs 15 pounds (6.8 kg). Thus, 100 mL X 6.8
= 680 mL/day.
QUESTION 619
A male client tells his nurse that he has had an ulcer in the past and is afraid it is “flaring up”
again. The nurse begins to ask him specific questions about his symptoms. The nurse knows
that a symptom that might indicate a serious complication of an ulcer is:
A. Pain in the middle of the night
B. A bowel movement every 3–5 days
C. Melena
D. Episodes of nausea and vomiting
Answer: C
Explanation:
(A) Clients with ulcers generally experience abdominal pain. It is common to have pain in the
early morning hours with an ulcer.
(B) Constipation is not a symptom associated with ulcers and would indicate a need to look at
other factors.
(C) Melena is blood in the stools. This could indicate a slow bleeding ulcer, which could
result in significant amounts of blood loss over time.
(D) Nausea and vomiting may be present as a result of the ulcer, especially if it is a gastric
ulcer. This does not indicate an immediate life-threatening complication.
QUESTION 620
A 78-year-old female client has a total hip arthroplasty. Her nurse should know that which of
the following is contraindicated?
A. Encourage exercises in the unaffected extremities.
B. Encourage her to cross and uncross her legs.

C. Check neurological and circulatory status of the affected leg hourly.
D. Place a trochanter roll along the upper thigh of the affected leg.
Answer: B
Explanation:
(A) Exercising the unaffected extremities will prevent contractures and emboli.
(B) Crossing and uncrossing the affected leg after surgery can dislocate the joint.
(C) Neurological and circulatory status of the affected leg has been compromised by surgery.
Hourly checks are needed to monitor the status of the leg.
(D) A trochanter roll will prevent the upper thigh from rolling outward, increasing the
chances of dislocation.
QUESTION 621
A female client has been recently diagnosed as bipolar. She has taken lithium for the past
several weeks to control mania. What must be included in client education regarding lithium
toxicity?
A. Maintain a normal diet; however, limit salt intake to no more than 3 g/day.
B. Take lithium between meals to increase absorption.
C. Withhold lithium if experiencing diarrhea, vomiting, or diaphoresis.
D. For pain or fever, avoid aspirin or acetaminophen (Tylenol). Nonsteroidal antiinflammatory drugs are preferred.
Answer: C
Explanation:
(A) The client should maintain a normal diet including normal salt intake. A low-sodium diet
can cause lithium retention, leading to toxicity.
(B) Lithium must be taken with meals because it is irritating to the gastric mucosa.
(C) Diarrhea, vomiting, or diaphoresis can cause dehydration, which will increase lithium
blood levels. If these symptoms occur, the nurse should instruct the client to withhold lithium.
(D) Lithium is not to be taken with over-the-counter drugs without specific instruction. Some
drugs raise lithium levels, whereas others lower lithium levels.
QUESTION 622
A 48-hour-old male infant is ordered to have phototherapy. When his mother questions the
nurse about its purpose, the nurse explains that phototherapy:
A. Prevents the development of ophthalmia neonatorum

B. Assists the baby’s clotting mechanism
C. Breaks down bilirubin in the skin into substances that can be excreted in stool or urine
D. Increases levels of unconjugated bilirubin, thereby preventing kernicterus (brain damage)
Answer: C
Explanation:
(A) The instillation of erythromycin ophthalmic preparation, not phototherapy, prevents
ophthalmia neonatorum.
(B) The administration of vitamin K (AquaMEPHYTON) assists the infant’s clotting
mechanism.
(C) Excessive bilirubin accumulates when the infant’s liver cannot handle the increased load
caused by the breakdown of red blood cells postnatally. This excessive bilirubin seeps out of
the blood and into the tissues, staining them yellow. Phototherapy accelerates the removal of
bilirubin from the skin by breaking it down into substances that can be excreted in stool or
urine.
(D) Phototherapy decreases levels of unconjugated bilirubin, thereby preventing kernicterus.
QUESTION 623
A 4-week-old infant is admitted to the emergency room in respiratory distress. Which of the
following statements indicates the nurse’s knowledge of the anatomy of the respiratory
system in pediatric clients?
A. The diameter of the trachea is much smaller in children than in adults.
B. The tongue is proportionally smaller in children than in adults.
C. The pediatric airway is more rigid than that of the adults.
D. The length of the pediatric airway is longer in children than in adults.
Answer: A
Explanation:
(A) The airway in children is much smaller than it is in adults. The diameter of the trachea in
the newborn is 4 mm and that of the adult is 20 mm. A small change in the diameter of the
airway can make a major difference in the pediatric client.
(B) The tongue is proportionally larger in children and fills most of the oral cavity, thereby
decreasing air space.
(C) The entire pediatric airway is elastic. Elasticity diminishes with age, however.

(D) The distances between respiratory structures are shorter than that of adults, and therefore
organisms are able to move more rapidly down the throat, leading to more extensive
respiratory involvement.
QUESTION 624
A client has chronic obstructive pulmonary disease. She is slowly losing weight, and her
daughter is very concerned about increasing her nutrition. The nurse helps the daughter
devise a plan of care for her mother. The plan of care should include which of the following
interventions to promote nutrition?
A. Offer her oral hygiene before and after meals.
B. Encourage her to consume milk products.
C. Encourage her to engage in an activity before a meal to stimulate her appetite.
D. Restrict her fluid intake to three glasses of water a day.
Answer: A
Explanation:
(A) Clients with respiratory diseases are generally mouth breathers. Cleaning the oral cavity
may improve the client’s appetite, increase her feelings of well-being, and remove the taste
and odor of sputum.
(B) Milk causes thick sputum; therefore, milk products would not be beneficial for this client.
(C) Exercise prior to a meal would require increased O2 consumption and most likely would
decrease the client’s ability to eat.
(D) Clients with respiratory diseases need increased fluid to liquefy secretions.
QUESTION 625
A female client at 37 week’s gestation has just undergone a nonstress test. The results were
two fetal movements with a corresponding increase in fetal heart rate (FHR) of 15 bpm
lasting 15 seconds within a 20- minute period. Her results would be classified as:
A. Reactive; needs follow-up contraction stress test
B. Reactive; no contraction stress test required
C. Non-reactive; needs follow-up contraction stress test
D. Non-reactive; no contraction stress test required
Answer: B
Explanation:
(A) A contraction stress test is unnecessary following a reactive (normal) nonstress test.

(B) The results are considered reactive, indicating that the fetus is not showing distress.
Therefore, a contraction stress test, which is a more in-depth test for fetal distress, is
unnecessary.
(C) A nonreactive test would show fewer than two fetal movements or a failure of the FHR to
increase at least 15 bpm with the movements in a 20-minute period.
(D) A contraction stress test should follow a nonreactive nonstress test to validate fetal
distress.
QUESTION 626
A client who is gravida 1 para 1 vaginally delivered a 7- lb girl. She received a midline
episiotomy at delivery. When assessing the level of her uterus immediately following
delivery, the nurse would expect the fundus to be located:
A. At the umbilicus
B. At the symphysis pubis
C. Midway between the umbilicus and the xiphoid process
D. Midway between the umbilicus and the symphysis pubis
Answer: D
Explanation:
(A) Within 12 hours of delivery, the fundus of the uterus rises to, or slightly above or below,
the umbilicus. Fundal height generally decreases 1 fingerbreadth, or 1 cm/day.
(B) The uterus descends into the pelvic cavity at approximately 10–12 postpartal days and
can no longer be palpated abdominally.
(C) Within 12 hours of delivery, the fundus of the uterus rises to, or slightly above or below,
the umbilicus. Fundal height generally decreases 1 fingerbreadth, or 1 cm/day. An enlarged
uterus may indicate subinvolution or postpartal hemorrhage.
(D) Immediately following delivery, the uterus lies midline, about midway between the
umbilicus and the symphysis pubis.
QUESTION 627
A client is a victim of domestic violence. She is now receiving assistance at a shelter for
battered women. She tells the nurse about the cycle of violence that she has been
experiencing in her relationship with her husband of 5 years. In the “tension-building phase”,
the nurse might expect the client to describe which of the following?
A. Promises of gifts that her husband made to her

B. Acute battering of the client, characterized by his volatile discharge of tension
C. Minor battering incidents, such as the throwing of food or dishes at her
D. A period of tenderness between the couple
Answer: C
Explanation:
(A) This description is characteristic of the “honeymoon” or “respite phase.”
(B) This description is characteristic of the “battering phase.”
(C) This description is characteristic of the “tension-building phase prior to the volatile
discharge of tension found in the battering phase.”
(D) This description is characteristic of the “honeymoon” or “respite phase.”
QUESTION 628
The physician of a client diagnosed with alcoholism orders neomycin 0.5 g q6h to prevent
hepatic coma. Neomycin decreases serum ammonia levels by:
A. Decreasing nitrogen-forming bacteria in the intestines
B. Acidifying colon contents by causing ammonia retention in the colon
C. Decreasing the uptake of vitamin D, thereby drawing more water into the colon
D. Irritating the bowel and promoting evacuation of stool
Answer: A
Explanation:
(A) Neomycin interferes with protein synthesis in the bacterial cell, causing bacterial death.
Neomycin reduces the growth of the ammonia-producing bacteria in the intestines and is used
for the treatment of hepatic coma.
(B) This choice describes the action of lactulose, another drug commonly used to decrease
systemic ammonia levels.
(C) Neomycin’s action does not decrease uptake of vitamin D to reduce serum ammonia
levels.
(D) Bowel irritation with diarrhea is more likely to occur with administration of lactulose
rather than of neomycin. Besides, diarrhea is a side effect of a drug, not the action of the drug.
QUESTION 629
A 6-year-old girl is visiting the outpatient clinic because she has a fever and a rash. The
doctor diagnoses chickenpox. Her mother asks the nurse how many baby aspirins her
daughter can have for fever. The nurse should:

A. Advise the mother not to give her aspirin
B. Ask if the client is allergic to aspirin before giving further information
C. Assess the function of the client’s cranial nerve VIII
D. Check the aspirin bottle label to determine milligrams per tablet
Answer: A
Explanation:
(A) Aspirin taken during a viral infection has been implicated as a predisposing factor to
Reye’s syndrome in children and adolescents. Children and adolescents should not be given
aspirin.
(B) Allergy to aspirin is not related to Reye’s syndrome.
(C) Tinnitus, caused by damage to the acoustic nerve, occurs with aspirin toxicity, but this is
not related to Reye’s syndrome.
(D) A 6-year-old child should not be given any baby aspirin.
QUESTION 630
The healthcare team determines that an elderly client has had progressive changes in memory
over the last 2 years that have interfered with her personal, social, or occupational
functioning. Her memory, learning, attention, and judgment have all been affected in some
way. These symptoms describe which of the following conditions?
A. Dementia
B. Parkinsonism
C. Delirium
D. Mania
Answer: A
Explanation:
(A) These changes are common characteristics of dementia.
(B) Parkinson’s disease affects the muscular system. Progressive memory changes are not
presenting symptoms.
(C) Delirium includes an altered level of consciousness, which is not found in dementia.
(D) Mania includes symptoms of hyperactivity, flight of ideas, and delusions of grandeur.
QUESTION 631
A 48-year-old male client is hospitalized with mild ascites, bruising, and jaundice. He has a
20-year history of alcohol abuse. The client is diagnosed with cirrhosis. His serum ammonia

level is high, indicating hepatic encephalopathy. He has esophageal varices. Which of the
following may cause the varices to rupture?
A. Lifting heavy objects
B. Walking briskly
C. Ingestion of barbiturates
D. Ingestion of antacids
Answer: A
Explanation:
(A) Lifting heavy objects will increase intrathoracic pressure, thus placing the client at risk
for rupturing esophageal varices.
(B, C, D) This activity will not cause an increase in intrathoracic pressure.
QUESTION 632
After 7 hours in restraints and a total of 30-mg haloperidol in divided doses, a client
complains of stiffness in his neck and his tongue “pulling to one side”. These extrapyramidal
symptoms (EPS) will most likely be relieved by the administration of:
A. Lorazepam (Ativan)
B. Benztropine (Cogentin)
C. Thiothixene (Navane)
D. Flurazepan (Dalmane)
Answer: B
Explanation:
(A) Lorazepam is an antianxiety agent that produces muscle relaxation and inhibits cortical
and limbic arousal. It has no action in the basal ganglia of the brain.
(B) Benztropine acts to reduce EPS by blocking excess CNS cholinergic activity associated
with dopamine deficiency in the basal ganglia by displacing acetylcholine at the receptor site.
(C) Thiothixene is an antipsychotic known to block dopamine in the limbic system, thereby
causing EPS.
(D) Flurazepan is a hypnotic that acts in the limbic system, thalamus, and hypothalamus of
the CNS to produce sleep. It has no known action in the vasal ganglia.
QUESTION 633
A chronic alcoholic client’s condition deteriorates, and he begins to exhibit signs of hepatic
coma. Which of the following is an early sign of impending hepatic coma?

A. Hiccups
B. Anorexia
C. Mental confusion
D. Fetor hepaticus
Answer: C
Explanation:
(A) Hiccups are not a sign of impending hepatic coma.
(B) Anorexia is not a sign of impending hepatic coma.
(C) One of the earliest symptoms of hepatic coma is mental confusion. Asterixis, a flapping
tremor of the hand, may also be seen.
(D) This sign is associated with the later stages of hepatic coma. Fetor hepaticus, a
characteristic odor on the breath that smells like acetone, may sometimes be noted when the
liver fails.
QUESTION 634
A client who has been diagnosed with anorexia nervosa refuses to eat lunch. The most
therapeutic response by the nurse to her refusal is:
A. “Okay, missing one meal won’t hurt.”
B. “You’ll have to eat lunch, or we’ll force-feed you.”
C. “It’s not appropriate for you to try to manipulate the staff into granting your wishes.”
D. “We will not allow you to starve yourself. You may choose to eat voluntarily or be fed.”
Answer: D
Explanation:
(A) This response reinforces the client’s maladaptive behavior, thereby contributing to the
client’s risk.
(B) Ultimatums are not therapeutic.
(C) This comment invites an argument because it puts the client on the defensive and stabs at
her self-esteem, which is already compromised.
(D) Setting limits assures the client that staff has genuine concern for her safety and wellbeing. Giving her an actual choice will give the client an increased sense of control over her
life and avoid an argument or power struggle.

QUESTION 635
A client is resting comfortably after delivering her first child. When assessing her pulse rate,
the nurse would recognize the following finding to be typical:
A. Thready pulse
B. Irregular pulse
C. Tachycardia
D. Bradycardia
Answer: D
Explanation:
(A) A thready pulse is indicative of hypotension and excessive blood loss and is often rapid.
(B) Pulse irregularities or dysrhythmias do not occur in the normal postpartal woman.
(C) Tachycardia occurs less frequently than bradycardia and is related to increased blood loss
or prolonged difficult labor and/or birth.
(D) Puerperal bradycardia with rates of 50–70 bpm commonly occurs during the first 6–10
days of the postpartal period. It may be related to decreased cardiac strain, decreased blood
volume, contraction of the uterus, and increased stroke volume.
QUESTION 636
The nurse is caring for a 2-year-old girl with a subdural hematoma of the temporal area as a
result of falling out of bed and notices that she has a runny nose. The nurse should:
A. Call the doctor immediately
B. Help her to blow her nose carefully
C. Test the discharge for sugar
D. Turn her to her side
Answer: C
Explanation:
(A) The nasal discharge could be due to a cold. It is necessary to gather additional assessment
data to identify a possible cerebrospinal fluid leak.
(B) If the discharge is cerebrospinal fluid, it would not be safe to encourage the girl to blow
her nose.
(C) Cerebrospinal fluid is positive for sugar; mucus is not.
(D) Turning her to her side will have no effect on her “runny nose”. It is necessary to gather
further assessment data.

QUESTION 637
A client delivered a term infant 1 hour ago. Her uterus on assessment is boggy and is U +1 in
contrast to the previous assessment of U 2. The immediate nursing response is to:
A. Administer methergine IM
B. Remove the retained placental fragments
C. Assist the client to the bathroom and provide cues to stimulate urination
D. Massage the fundus until firm
Answer: D
Explanation:
(A) Methergine is given following placental delivery to promote uterine contractions and
prevent hemorrhage. Methergine may be administered in this clinical situation, but fundal
massage would be the first response.
(B) Removal of retained placental fragments is done by the physician and is not the first
response.
(C) If the fundus rises and is deviated, particularly to the right, the nurse should suspect
bladder distention secondary to bladder and urethral trauma associated with birth and
decreased bladder tone following delivery. Therefore, women have a diminished sensation to
void.
(D) A boggy fundus rises and is indicative of blood pooling, predisposing the woman to clot
formation. Massage the uterus until firm. Too vigorous massage will result in atonia. Clots
may be expelled by a kneading motion of the uterus by the nurse.
QUESTION 638
A 64-year-old client is admitted to the hospital with benign prostatic hypertrophy (BPH). He
has a history of adult-onset diabetes and hypertension and is scheduled to undergo a resection
of the prostate. When recording his health history, the nurse asks about his chief complaint.
The most serious symptom that may accompany BPH is:
A. Acute urinary retention
B. Hesitancy in starting urination
C. Increased frequency of urination
D. Decreased force of the urinary stream
Answer: A
Explanation:

(A) Acute urinary retention requires urgent medical attention. If measures such as a warm tub
bath or warm tea do not occur after 6 hours, the client should go to the ED for catheterization.
(B, C, D) This choice is a symptom of BPH, but it is not serious or life threatening.
QUESTION 639
Following TURP, which of the following instructions would be appropriate to prevent or
alleviate anxiety concerning the client’s sexual functioning?
A. “You may resume sexual intercourse in 2 weeks.”
B. “Many men experience impotence following TURP.”
C. “A transurethral resection does not usually cause impotence.”
D. “Check with your doctor about resuming sexual activity.”
Answer: C
Explanation:
(A) Sexual activity should be delayed until cleared by the client’s physician.
(B) Although many men experience retrograde ejaculation following prostate surgery,
potency is seldom affected.
(C) Although the client may experience retrograde ejaculation, it will not limit his ability to
engage in sexual intercourse.
(D) Although the client should obtain clearance from his physician before resuming sexual
activity, this statement does not give the client any information or reassurance about future
sexual activity or potency that could decrease his anxiety.
QUESTION 640
Before giving methergine postpartum, the nurse should assess the client for:
A. Decreased amount of lochial flow
B. Elevated blood pressure
C. Flushing
D. Afterpains
Answer: B
Explanation:
(A) Methergine is given to contract the uterus and to control postpartal hemorrhage;
therefore, lochial flow should decrease.
(B) Methergine may elevate the blood pressure. A client with an elevated blood pressure
should not receive methergine, but she could be given oxytocin if necessary.

(C) Flushing is not a side effect of methergine.
(D) Afterpains are increased with methergine usage. The client should be informed that this is
a normal response.
QUESTION 641
A male client is experiencing auditory hallucinations. His nurse enters the room and he tells
her that his mother is talking to him, and he will take his medicine after she leaves. The nurse
looks around the room and sees that she and the client are the only ones in the room. The
nurse’s most therapeutic response will be:
A. “I don’t see your mother in the room. Let’s talk about how you’re feeling.”
B. “OK, I’ll come back later when you’re feeling more like taking your medicine.”
C. “She may be here, but I can’t see her.”
D. “Why don’t you finish talking to her, and I’ll wait.”
Answer: A
Explanation:
(A) This response uses the principle of reality orientation by the nurse telling the client that
he or she does not see anything, but it does recognize his feelings.
(B) This response does not make it clear that the nurse does not see anyone else in the room,
and the nurse leaves the client alone to continue hallucinating.
(C) This response leaves room for doubt; the nurse is further confusing the client by this
statement.
(D) This response reinforces the hallucination and implies that the nurse sees his mother, too.
QUESTION 642
Degenerative disorders are attributed to many factors. As a nurse assigned to a convalescent
home, one must often educate families about how such conditions occur. Which of the
following statements might the nurse need to explore when a daughter tries to explain to her
mother what caused her degenerative disorder?
A. “Some folks believe that aging causes this, Mother.”
B. “Perhaps, it’s the way your parents used those double- bind messages, Mother.”
C. “I know some people who are having this problem and they were exposed to chemicals at
work, Mother.”
D. “It can be caused by lots of things, toxic agents and even alcohol, Mother.”
Answer: B

Explanation:
(A) Aging is a factor in the cause of degenerative disorders.
(B) Double-bind messages may be found in the histories of families of individuals who
develop schizophrenia, but they are not related to degenerative disorders.
(C) Chemicals (toxic agents) in work environments are predisposing factors to degenerative
disorders.
(D) Alcohol causes some degenerative disorders, such as Wernicke’s syndrome.
QUESTION 643
A 42-year-old male client has been treated at an alcoholic rehabilitation center for
physiological alcohol dependence. The nurse will be able to determine that he is preparing for
discharge and is effectively coping with his problem when he shares with her the following
information:
A. “I know that I will not ever be able to socially drink alcohol again and will need the
support of the AA group.”
B. “I know that I can only drink one or two drinks at social gatherings in the future, but at
least I don’t have to continue AA.”
C. “I really wasn’t addicted to alcohol when I came here, I just needed some help dealing
with my divorce.”
D. “It really wasn’t my fault that I had to come here. If my wife hadn’t left, I wouldn’t have
needed those drinks.”
Answer: A
Explanation:
(A) The client has insight into the severity of his alcohol addiction and has chosen one of the
most effective treatment strategies to support him—Alcoholics Anonymous.
(B) The client is still using denial and is not dealing with his alcohol addiction.
(C) The client is exhibiting denial about his alcohol addiction and projecting blame on his
divorce.
(D) The client is projecting blame onto his wife for being in the hospital while still denying
his alcohol addiction.
QUESTION 644
A client at 9 week’s gestation comes for an initial prenatal visit. On assessment, the nurse
discovers this is her second pregnancy. Her first pregnancy resulted in a spontaneous

abortion. She is 28 years old, in good health, and works full-time as an elementary school
teacher. This information alerts the nurse to which of the following:
A. An increased risk in maternal adaptation to pregnancy
B. The need for anticipatory guidance regarding the pregnancy
C. The need for teaching regarding family planning
D. An increased risk for subsequent abortions
Answer: B
Explanation:
(A, D) There are no data to support this.
(B) Anticipatory guidance and health maintenance is a first-line defense in the promotion of
healthy mothers and healthy babies.
(C) There are no data to support this at this time. This will be a concern later.
QUESTION 645
A client is admitted to the psychiatric unit after lavage and stabilization in the emergency
room for an overdose of antidepressants. This is her third attempt in 2 years. The highest
priority intervention at this time is to:
A. Assess level of consciousness
B. Assess suicide potential
C. Observe for sedation and hypotension
D. Orient to her room and unit rules
Answer: B
Explanation:
(A) The client was stabilized in the ED and consequently would not be sent to the psychiatric
unit if comatose.
(B) Suicide assessment is always appropriate for clients with a history of previous attempts or
depression, because either of these factors places the client at high risk.
(C) The admission assessment should include observation for sedation and hypotension, but
this is not in priority over suicide assessment.
(D) Orientation to room and unit rules is of low priority at this time.

QUESTION 646
A seventh grader lost consciousness after being hit in the head with a basketball. In the
emergency room his vital signs are stable, and he demonstrates no neurologic deficit. He will
not be admitted to the hospital. It is most important that you advise his mother to:
A. Encourage him to drink plenty of fluids
B. Expect him to have nausea with vomiting
C. Keep him awake for the next 12 hours
D. Wake him up every 1–2 hours during the night
Answer: D
Explanation:
(A) Fluid intake should be normal. Fluid intake may be restricted when there is a risk for
increased ICP in a hospitalized client.
(B) Nausea is possible, but vomiting without nausea is more likely with increased ICP.
Neither one should be expected, but the mother should know to notify the physician or
hospital if they occur.
(C) The child does not need to be kept awake. It is important that he can be aroused from
sleep.
(D) If the child cannot be awakened from sleep after head injury, it is an indication of serious
increase in ICP. The mother should call an ambulance right away.
QUESTION 647
A newborn girl’s father expresses concern that the newborn does not have good control of her
hands and arms.
It is important for the father to realize certain neurological patterns that characterize the
newborn:
A. Mild hypotonia is expected in the upper extremities.
B. Purposeless, uncoordinated movements of the arms are indicative of neurological
dysfunction.
C. Function progresses in a head-to-toe, proximal-distal fashion.
D. Asymmetrical movement of the extremities is not unusual and will disappear with
maturation of the central nervous system.
Answer: C
Explanation:

(A) Term neonates are predominantly in a flexed position with strong active muscle tone that
increases. Newborns are slightly hypertonic.
(B) Neonatal movements may be jerky and uncoordinated as the neonate works against
gravity in contrast to the buoyancy of the amniotic fluid. Jerky movements must be
differentiated from the tremors of hypoglycemia, hypocalcemia, and neurological
dysfunction.
(C) Growth of the newborn progresses in a cephalocaudal, proximal-distal fashion.
Knowledge regarding infant development may facilitate parental involvement and infant
stimulation.
(D) Asymmetrical movements of the extremities are indicative of neurological dysfunction.
QUESTION 648
A 2-year-old child with a scalp laceration and subdural hematoma of the temporal area as a
result of falling out of bed should be prevented from:
A. Crying
B. Falling asleep
C. Rolling from his back to his tummy
D. Sucking his thumb
Answer: A
Explanation:
(A) A child with a subdural hematoma has increased ICP. Crying may significantly increase
this pressure.
(B) Adequate sleep is essential, but it is important that the child can be aroused from sleep
after head injury.
(C) This child is free to roll from his back to his abdomen.
(D) Thumb-sucking serves to reduce anxiety and should not be prevented at this time.
QUESTION 649
The client will be more comfortable and the results more accurate when the nurse prepares
the client for Leopold’s maneuvers by having her:
A. Empty her bladder
B. Lie on her left side
C. Place her arms over her head
D. Force fluids 1 hour prior to procedure

Answer: A
Explanation:
(A) A full bladder would cause discomfort and possible urinary incontinence during the exam.
(B) The left side- lying position would not accommodate the exam. The head of the exam
table or bed can be slightly elevated to prevent supine hypotension.
(C) Arms extended over the head would cause the abdomen to be tighter and less easily
palpable.
(D) Forcing fluids would encourage a full bladder, which is not desired for the exam.
QUESTION 650
In counseling a client, the nurse emphasizes the danger signals during pregnancy. On the next
visit, the client identifies which of the following as a danger signal that should be reported
immediately?
A. Backache
B. Leaking of clear yellow fluid from breasts
C. Constipation with hemorrhoids
D. Visual changes
Answer: D
Explanation:
(A) Backache is a common complaint during pregnancy. Proper body mechanics, pelvic rock,
back rubs, and other comfort measures should relieve the discomfort. In the presence of
uterine contractions, the backache would radiate to the lower abdomen.
(B) Colostrum is normal and can be present anytime in the second half of pregnancy.
(C) Constipation and hemorrhoids are common and do need attention, but they do not
constitute a dangerous situation.
(D) Visual changes are possibly related to PIH. The client should be assessed immediately to
rule out or prevent worsening of PIH.
QUESTION 651
A female client with major depression stated that “life is hopeless and not worth living”. The
nurse should place highest priority on which of the following questions?
A. “How has your appetite been recently?”
B. “Have you thought about hurting yourself?”
C. “How is your relationship with your husband?”

D. “How has your depression affected your daily living activities?”
Answer: B
Explanation:
(A) Although eating habits are important to assess, they are less important than suicidal
intent.
(B) Maintenance of the client’s life is the priority; assessment of suicidal intent is imperative.
(C) Relationships and support systems are an important part of assessment, but they are less
important than suicidal intent.
(D) Daily living activities will give additional information about the level of depression, and
are less significant than suicidal intent, although this information may give additional
information about the actual plan for a suicidal attempt.
QUESTION 652
A 35-year-old primigravida comes to the clinic for her first prenatal visit. The midwife, on
examining the client, suspects that she is approximately 11 weeks pregnant. The pregnancy is
positively confirmed by finding:
A. Chadwick’s sign
B. FHR by ultrasound
C. Enlargement of the uterus
D. Breast tenderness and enlargement
Answer: B
Explanation:
(A) Chadwick’s sign is a presumptive sign of pregnancy. The coloration may not subside
from past pregnancy or could be caused by other situations that create Vaso congestion.
(B) FHR (movement) observed on ultrasound is a positive diagnosis of pregnancy.
(C) Enlargement of the uterus may be due to fibroids or infection. It is considered a probable
sign.
(D) Breast tenderness and enlargement is a presumptive sign because it may be due to other
conditions, such as premenstrual changes.
QUESTION 653
A family is experiencing changes in their lifestyle in many ways. The invalid grandmother
has moved in with them. The couple have a 2-year-old son by their marriage, and the wife has
two children by her previous marriage. The older children are in high school. In applying

systems theory to this family, it is important for the nurse to remember which of the following
principles?
A. The parts of a system are only minimally related.
B. Dysfunction in one part affects every other part.
C. A family system has no boundaries.
D. Healthy families are enmeshed.
Answer: B
Explanation:
(A) The parts of a system are interrelated.
(B) Any change in any part of the system affects all other parts.
(C) A family system, like any other system, has boundaries.
(D) Healthy families are neither enmeshed nor disengaged.
QUESTION 654
In healthcare settings, nurses must be familiar with primary, secondary, and tertiary levels of
care. As a nurse in the community, which of the following interventions might be a primary
prevention strategy?
A. Crisis intervention with an intoxicated teenager whose mother just committed suicide
B. Referring a client who has been on a detoxification unit to a rehabilitation center
C. Teaching fifth-grade children the harmful effects of substance abuse
D. Counseling a client with post-traumatic stress disorder
Answer: C
Explanation:
(A) The teenager is already coping ineffectively and requires early detection and treatment,
which is secondary prevention.
(B) The client must be sent to a rehabilitation unit, which requires tertiary prevention.
(C) Reducing the incidence of disease through education supports primary prevention.
(D) A client with identified symptoms of post-traumatic stress disorder requires intervention
by treatment.
QUESTION 655
A client’s prenatal screening indicated that she has no immunity to rubella. She is now 10
weeks pregnant. The best time to immunize her is:
A. In the immediate postpartum period

B. After the first trimester
C. At 28 week’s gestation
D. Within 72 hours postpartum
Answer: A
Explanation:
(A) The rubella vaccine is made with attenuated virus and is given in the immediate
postpartal period to prevent infection during pregnancy and subsequent adverse fetal and
neonatal sequelae. Mothers are advised to prevent pregnancy for 3 months following
immunization.
(B) Rubella infection during the second trimester may result in permanent hearing loss for the
fetus.
(C) RhoGam is the drug generally administered at 28 week’s gestation to Rh-negative
women. It is contraindicated to administer rubella vaccine during pregnancy.
(D) RhoGam is the drug administered within 72 hours postpartum to Rh-negative women to
prevent the development of antibodies to fetal cells.
QUESTION 656
When assessing the client 6 hours postpartum, the fundus is found to be U +3, displaced to
the right of midline, and slightly boggy. The nurse would first:
A. Increase the IV oxytocin drip rate
B. Give methergine IM
C. Assess for a full bladder
D. Grasp the uterus and massage vigorously
Answer: C
Explanation:
(A) Oxytocin may not be necessary if the bladder is emptied and if the uterus remains firm,
midline, and at about U11 after massage.
(B) The same rationale as for answer “A applies.”
(C) A full bladder is the most common cause of uterine atony. If the bladder is full, it should
be emptied and the uterus reassessed before further intervention.
(D) If the bladder is full, the uterus will not stay contracted or return to a normal position.
Overly vigorous massage also encourages uterine atony.

QUESTION 657
The nurse in the mental health center is instructing a depressed client about the dietary
restrictions necessary in taking her medication, which is a monoamine oxidase (MAO)
inhibitor. Which of the following is she restricting from the client’s diet?
A. Cream cheese
B. Fresh fruits
C. Aged cheese
D. Yeast bread
Answer: C
Explanation:
(A) Cream cheese does not contain tyramine, which might cause a hypertensive crisis.
(B) Fresh fruits do not contain tyramine, which might cause a hypertensive crisis.
(C) Aged or matured cheese combined with a monoamine oxidase predisposes the client to a
hypertensive crisis.
(D) Bread products raised with yeast do not contain tyramine.
QUESTION 658
Two weeks after a client’s admission for depression, the physician orders a consult for
electroconvulsive therapy (ECT). Which of the following conditions, if present, would be a
contraindication for ECT?
A. Brain tumor or other space-occupying lesion
B. History of mitral valve prolapse
C. Surgically repaired herniated lumbar disk
D. History of frequent urinary tract infections
Answer: A
Explanation:
(A) A contraindication for ECT is a space-occupying lesion such as a brain tumor. During
ECT, intracranial pressure increases. Therefore, ECT would not be prescribed for a client
whose intracranial pressure is already elevated.
(B) Any cardiac dysrhythmias or complications that arise during ECT are usually attributed to
the IV anesthetics used, not to preexisting cardiac structural conditions.
(C) Musculoskeletal injuries during ECT are extremely rare because of the IV use of centrally
acting muscle relaxers.

(D) A history of any kind of infection would not contraindicate the use of ECT. In fact,
concurrent treatment of infections with ECT is not uncommon.
QUESTION 659
A 20-year-old female client delivers a stillborn infant. Following the delivery, an appropriate
response by the labor nurse to the question, “Why did this happen to my baby?” is:
A. “It’s God’s will. It was probably for the best. There was something probably wrong with
your baby.”
B. “You’re young. You can have other children later.”
C. “I know your other children will be a great comfort to you.”
D. “I can see you’re upset. Would you like to see and hold your baby?”
Answer: D
Explanation:
(A) The mother and the father require support; the nurse should not minimize their grief in
this situation.
(B) Attachment to this infant occurs during the pregnancy for both the mother and father.
Another child cannot replace this child.
(C) Attachment to this infant occurs during the pregnancy for both the mother and father.
Siblings will not replace their feelings or minimize their loss of this infant.
(D) Holding and viewing the infant decreases denial and may facilitate the grief process. The
nurse should prepare family members for how the infant appears (“she is bruised”) and
provide support.
QUESTION 660
An 18-year-old girl is admitted to the hospital with a depressed skull fracture as a result of a
car accident. If the nurse were to observe a rising pulse rate and lowering blood pressure, the
nurse would suspect that the client:
A. Has a sudden and severe increase in intracranial pressure
B. Has sustained an internal injury in addition to the head injury
C. Is beginning to experience a dangerously high level of anxiety
D. Is having intracranial bleeding
Answer: B
Explanation:

(A) Widening pulse pressure (high systolic and low diastolic) with compensatory slowing of
pulse rate are late signs of increasing ICP.
(B) Rising pulse rate and lowering blood pressure are indicative of hypovolemia due to
hemorrhage.
(C) High anxiety, in the absence of hemorrhage, would result in a high pulse rate and a high
blood pressure.
(D) Intracranial bleeding results in increased ICP. A change in level of consciousness is an
early sign of increasing ICP, and vital sign changes are late signs of increasing ICP.
QUESTION 661
A client is placed in five-point restraints after exhibiting sudden violence after illegal drug
use, and haloperidol
(Haldol) 5 mg IM is administered. After 1 hour, his behavior is more subdued, but he tells the
nurse, “The devil followed me into this room”, I see him standing in the corner with a big
knife. When you leave the room, he’s going to cut out my heart. The nurse’s best response is:
A. “I know you’re feeling frightened right now, but I want you to know that I don’t see
anyone in the corner.”
B. “You’ll probably see strange things for a while until the PCP wears off.”
C. “Try to sleep. When you wake up, the devil will be gone.”
D. “You’re probably feeling guilty because you used illegal drugs tonight.”
Answer: A
Explanation:
(A) The nurse is the client’s link to reality. This response validates the authenticity of the
client’s experience by casting doubt on his belief and reinforcing reality.
(B) Although this statement may be literally correct, it is nontherapeutic because it lacks
validation.
(C) This response encourages the client to attempt to do something that may be impossible at
this time, offers false reassurance, and reinforces delusional content.
(D) The nurse is making an incorrect assumption about the client’s feelings by offering a
nontherapeutic interpretation of the motivation for the client’s actions.
QUESTION 662
A client has had amniocentesis. One of the tests performed on the amniotic fluid is a
lecithin/sphingomyelin (L/S) ratio. The results show a ratio of 1:1. This is indicative of:

A. Lung immaturity
B. Intrauterine growth retardation (IUGR)
C. Intrauterine infection
D. Neural tube defect
Answer: A
Explanation:
(A) At about 30–32 week’s gestation, the amounts of the surfactants, lecithin, and
sphingomyelin become equal. As the fetal lungs mature, the concentration of lecithin begins
to exceed that of sphingomyelin. At 35 weeks, the L/S ratio is 2:1. Respiratory distress
syndrome is unlikely if birth occurs at this time.
(B) IUGR is associated with compromised uteroplacental perfusion or with viral infections,
chromosomal disorders, congenital malformations, and maternal malnutrition. IUGR is not
specifically assessed by analysis of the L/S ratio.
(C) Analysis of the L/S ratio is not an assessment used to confirm intrauterine infection.
(D) Elevated levels of fetoprotein in maternal serum or in amniotic fluid have been found to
reflect open neural tube defects, such as spina bifida and anencephaly.
QUESTION 663
While the nurse is taking a male client’s blood pressure, he makes flirtatious remarks to her.
The nurse will handle this effectively if she:
A. Politely tells the client, “Keep your hands off”
B. Ignores the remarks and hopes he will not try it again
C. Confronts the remarks but attempts not to reject the client
D. Leaves the room in order to compose herself
Answer: C
Explanation:
(A) This response does not recognize normal feelings of attraction and rejects the client.
(B) By ignoring the situation, the nurse has not set limits to discourage other remarks or
perhaps more sexually aggressive behavior.
(C) By confronting the remarks, she can recognize that his feelings of attraction may be
normal but are not appropriate within the context of their nurse-client relationship.
(D) Leaving the room does not deal with setting limits for future interactions.

QUESTION 664
Which of the following blood values would require further nursing action in a newborn who
is 4 hours old?
A. Hemoglobin 17.2 g/dL
B. Platelets 250,000/mm3
C. Serum glucose 30 mg/dL
D. White blood cells 18,000/mm3
Answer: C
Explanation:
(A) The normal range for hemoglobin in the newborn is 17–19 g/dL; 17.2 g/dL is within
normal limits.
(B) A normal value range for platelets in the newborn is 150,000–400,000 mm3; 250,000/mm3
is within normal range.
(C) A serum glucose of 30 mg/dL in the first 72 hours of life is indicative of hypoglycemia
and warrants further intervention.
(D) On the day of birth, a white blood cell count of 18,000–40,000/mm3 is normal in the
newborn.
QUESTION 665
A violent client remains in restraints for several hours. Which of the following interventions
is most appropriate while he is in restraints?
A. Give fluids if the client requests them.
B. Assess skin integrity and circulation of extremities before applying restraints and as they
are removed.
C. Measure vital signs at least every 4 hours.
D. Release restraints every 2 hours for client to exercise.
Answer: D
Explanation:
(A) Fluids (nourishment) should be offered at regular intervals whether the client requests (or
refuses) them or not.
(B) Skin integrity and circulation of the extremities should be checked regularly while the
client is restrained, not only before restraints are applied and after they are removed.
(C) Vital signs should be checked at least every 2 hours. If the client remains agitated in
restraints, vital signs should be monitored even more closely, perhaps every 1–2 hours.

(D) Restraints should be released every 2 hours for exercise, one extremity at a time, to
maintain muscle tone, skin and joint integrity, and circulation.
QUESTION 666
A husband and wife and their two children, age 9 and age 5, are requesting family therapy.
Which of the following strategies is most therapeutic for the nurse to use during the initial
interaction with a family?
A. Always allow the most vocal person to state the problem first.
B. Encourage the mother to speak for the children.
C. Interpret immediately what seems to be going on within the family.
D. Allow family members to assume the seats as they choose.
Answer: D
Explanation:
(A) One will always hear what the most vocal person has to say. It is best to start with the
quietest family member to encourage that person to express emotions.
(B) All family members are encouraged to speak for themselves.
(C) In the initial family assessment, only data collection occurs; interpretations are made
later.
(D) Allowing family members to choose their own seats will assist the nurse in assessing the
family system and in determining who feels closer to whom.
QUESTION 667
In acute episodes of mania, lithium is effective in 1–2 weeks, but it may take up to 4 weeks,
or even a few months, to treat symptoms fully. Sometimes an antipsychotic agent is
prescribed during the first few days or weeks of an acute episode to manage severe behavioral
excitement and acute psychotic symptoms. In addition to the lithium, which one of the
following medications might the physician prescribe?
A. Diazepam (Valium)
B. Haloperidol (Haldol)
C. Sertraline (Zoloft)
D. Alprazolam (Xanax)
Answer: B
Explanation:

(A) Diazepam is an antianxiety medication and is not designed to reduce psychotic
symptoms.
(B) Haloperidol is an antipsychotic medication and may be used until the lithium takes effect.
(C) Sertraline is an antidepressant and is used primarily to reduce symptoms of depression.
(D) Alprazolam is an antianxiety medication and is not designed to reduce psychotic
symptoms.
QUESTION 668
A client is admitted to the labor room. She is dilated 4 cm. She is placed on electric fetal
monitoring. Which of the following observations necessitates notifying the physician?
A. Contractions every 2 minutes, lasting 100 seconds
B. Fetal heart decelerations during a contraction
C. Beat-to-beat variability between contractions
D. Fetal heart decelerations at the beginning of contractions
Answer: A
Explanation:
(A) These are tetanic in nature and can cause rupture of the uterus.
(B) The FHR decreases during contractions owing to vasoconstriction and should recover
after the contraction.
(C) Beat-to-beat variability is a normal finding and demonstrates fetal well-being.
(D) The FHR may decrease at the beginning of a contraction owing to head compression.
QUESTION 669
On admission to the postpartal unit, the nurse’s assessment identifies the client’s fundus to be
soft, 2 fingerbreadths above the umbilicus, and deviated to the right. This is most likely an
indication of:
A. Normal involution
B. A full bladder
C. An infection pain
D. A hemorrhage
Answer: B
Explanation:
(A) Immediately after expulsion of the placenta, the fundus should be in the midline and
remain firm.

(B) A boggy displaced uterus in the immediate postpartum period is a sign of urinary
distention. Because uterine ligaments are stretched, a full bladder can displace the uterus.
(C) Symptoms of infection may include unusual uterine discomfort, temperature elevation,
and foul-smelling lochia. The stem of this question does not address any of these factors.
(D) While excessive bleeding is associated with a soft, boggy uterus, the stem of this question
includes displacement of the uterus, which is more commonly associated with bladder
distention.
QUESTION 670
A 28-year-old multigravida has class II heart disease. At her prenatal visit at 34 week’s
gestation, all of the following observations are made. Which would require intervention?
A. Weight gain of 2 kg in 4 weeks
B. Blood pressure of 128/78
C. Subjective data: shortness of breath after showering
D. Ankle edema reported present in late afternoon and evenings
Answer: C
Explanation:
(A) This is not an excessive weight gain indicative of fluid retention.
(B) The blood pressure is within normal range.
(C) Showering should not cause shortness of breath. This could be a sign of cardiac
decompensation.
(D) Dependent ankle edema is normal late in the day among pregnant women. Progressive
edema would be a dangerous development.
QUESTION 671
A client suspected of having anorexia nervosa is placed on bed rest with an IV infusion and a
high-carbohydrate liquid diet. Within 72 hours, the results of her lab work show a return to
normal limits. She is transferred to the psychiatric service for further treatment. A behavior
modification plan is initiated. Three days after her transfer, the client tells the nurse, “I
haven’t exercised in 6 days”. I won’t be eating lunch today. This statement by her most likely
reflects:
A. Her lack of internal awareness about the outcome of the behavior
B. Increased knowledge about personal exercise plans
C. A manipulative technique to trick the nurse into allowing her to miss a meal

D. A true desire to stay fit while in the hospital
Answer: A
Explanation:
(A) Indirect self-destructive behavior such as that seen in anorexia nervosa is characterized
by the client’s lack of insight and the awareness that the outcome of the dieting, exercising,
and weight loss will ultimately result in death if uninterrupted.
(B) Although the client is knowledgeable about exercise, knowledge about the balance
between nutrition, exercise, and rest is absent.
(C) The client’s level of denial and lack of awareness disallow this behavior as a manipulative
trick.
(D) The client’s illness-maintaining behaviors are inconsistent with fitness.
QUESTION 672
The doctor has ordered a restricted fluid intake for a 2- year-old child with a head injury.
Normal fluid intake for a child of 2 years is:
A. 900 mL/24 hr
B. 1300 mL/24 hr
C. 1600 mL/24 hr
D. 2000 mL/24 hr
Answer: C
Explanation:
(A, B, D) These values are incorrect. Normal intake for a child of 2 years is about 1600 mL in
24 hours.
(C) This value is correct. Normal intake for a child of 2 years is about 1600 mL in 24 hours.
QUESTION 673
When discussing the relationship between exercise and insulin requirements, a 26-year-old
client with IDDM should be instructed that:
A. When exercise is increased, insulin needs are increased
B. When exercise is increased, insulin needs are decreased
C. When exercise is increased, there is no change in insulin needs
D. When exercise is decreased, insulin needs are decreased
Answer: B
Explanation:

(A) If the client’s insulin is increased when activity level is increased, hypoglycemia may
result.
(B) Exercise decreased the blood sugar by promoting uptake of glucose by the muscles.
Consequently, less insulin is needed to metabolize ingested carbohydrates. Extra food may be
required for extra activity.
(C) This statement directly contradicts the correct answer and is inaccurate.
(D) When exercise is decreased, the client’s insulin dose does not need to be altered unless
the blood sugar becomes unstable.
QUESTION 674
Blood work reveals the following lab values for a client who has been diagnosed with
anorexia nervosa: hemoglobin 9.6 g/dL, hemocrit 27%, potassium 2.7 mEq/L, sodium 126
mEq/L. The greatest danger to her at this time is:
A. Hypoglycemia from low-carbohydrate intake
B. Possible cardiac dysrhythmias secondary to hypokalemia
C. Dehydration from vomiting
D. Anoxia secondary to anemia
Answer: B
Explanation:
(A) There is no lab data to support hypoglycemia.
(B) Hypokalemia, caused by vomiting and decreased dietary intake of potassium, can result
in life- threatening dysrhythmias.
(C) Evidence of dehydration is not life threatening at this time, although fluid volume deficit
does need to be addressed.
(D) The client’s hemoglobin does not reflect a life threatening value sufficient to render the
client anoxic.
QUESTION 675
A 14-year-old boy has a head injury with laceration of his scalp over his ear. The nurse should
call the physician to report:
A. Blood pressure increase from 100/80 to 115/85 after lunch
B. Headache that is unresponsive to acetaminophen (Tylenol)
C. Pulse rate ranges between 68 bpm and 76 bpm
D. Temperature rise to 102°F rectally

Answer: D
Explanation:
(A) This change in blood pressure may not be significant and does not indicate a widening
pulse pressure, a late sign of increased ICP. It is important to continue to monitor for change
in blood pressure.
(B) Acetaminophen may be ineffective in relieving headache after head injury. Stronger
analgesics are contraindicated because they mask neurological signs and may depress the
CNS.
(C) Pulse rates between 68 bpm and 76 bpm are within normal limits for a 14-year-old child.
It is important to monitor for a consistent drop in pulse rate, which is a late sign of increasing
ICP.
(D) An elevated temperature is abnormal and requires further assessment and medical
intervention. The temperature may be unrelated to the head injury, but CNS infection is
serious and difficult to control.
QUESTION 676
A 2-year-old boy is in the hospital outpatient department for observation after falling out of
his crib and hitting his head. The nurse calls the physician to report:
A. Evidence of perineal irritation
B. Pulse fell from 102 to 96
C. Pulse increased from 96 to 102
D. Temperature rose to 102°F rectally
Answer: D
Explanation:
(A) Perineal irritation needs to be addressed, but it is probably not necessary to call the
physician.
(B) This fall in pulse rate remains within normal limits and is probably insignificant. It is
important to monitor for continued change.
(C) This rise in pulse rate is probably not significant, but it is important to monitor for
continued change.
(D) This temperature is above normal limits and needs medical investigation. It may or may
not be related to the head injury.

QUESTION 677
A client has been in labor 10 hours and is becoming very tired. She has dilated to 7 cm and is
at 0 station with the fetus in a right occipitoposterior position. She is complaining of severe
backache with each contraction. One comfort measure the nurse can employ is to:
A. Place her in knee-chest position during the contraction
B. Use effleurage during the contraction
C. Apply strong sacral pressure during the contraction
D. Have her push with each contraction
Answer: C
Explanation:
(A) This measure is inappropriate. The knee-chest position is employed to take pressure off
the cord.
(B) Effleurage is a comfort measure but not the one that will contribute most to the relief of
backache caused by a posterior position.
(C) Sacral pressure will counteract the pressure created by the position of the fetal head.
(D) The client is not completely dilated. Pushing is contraindicated until the second stage of
labor.
QUESTION 678
The nurse is admitting an infant with bacterial meningitis and is prepared to manage the
following possible effects of meningitis:
A. Constipation
B. Hypothermia
C. Seizure
D. Sunken fontanelles
Answer: C
Explanation:
(A) Constipation may occur if the child is dehydrated, but it is not directly associated with
meningitis.
(B) It is more likely the child will have fever.
(C) Seizure is often the initial sign of meningitis in children and could become frequent.
(D) It is more likely the child will have bulging fontanelles.

QUESTION 679
The nurse explains perineal hygiene self-care postpartum to the client. She should be
instructed to:
A. Wear gloves for the procedure
B. Place and adjust the pad from back to front
C. Cleanse and wipe the perineum from front to back
D. Protect the outer surface of the pad from contamination
Answer: C
Explanation:
(A) Perineal hygiene is a clean procedure and does not require the client to wear gloves. A
care provider should wear gloves to adhere to universal precautions.
(B) The pad should be applied from front to back to prevent contamination of the birth canal
or urinary tract from rectal bacteria.
(C) Wiping from front to back and discarding the wipe prevents contamination of the urinary
tract and birth canal from rectal bacteria.
(D) The inner surface of the pad should not be touched to maintain asepsis.
QUESTION 680
On morning rounds, the nurse found a manic-depressive client who is taking lithium in a
confused mental state, vomiting, twitching, and exhibiting a coarse hand tremor. Which one
of the following nursing actions is essential at this time?
A. Administer her next dosage of lithium, and then call the physician.
B. Withhold her lithium, and report her symptoms to the physician.
C. Place her on NPO to decrease the excretion of lithium from her body, and call the
physician.
D. Contact the lab and request a lithium level in 30 minutes, and call the physician.
Answer: B
Explanation:
(A) The client has lithium toxicity, and the nurse must withhold further dosages.
(B) Because of her level of toxicity, further lithium could cause coma and death. The nurse
needs further orders from the physician to stabilize the client’s lithium level.
(C) Ensuring adequate intake of sodium chloride will promote excretion of lithium and will
assist in managing the client’s lithium toxicity.

(D) A lithium blood level must be drawn immediately to determine the seriousness of the
toxicity and to provide the physician with data for medical orders.
QUESTION 681
A registered nurse is trying to determine the appropriate care that she should provide for her
obstetrical clients. Which of the following documents is considered the legal standard of
practice?
A. State nursing practice act
B. AWHONN Standards for the Nursing Care of Women and Newborns
C. American Nurses Association Standards of Maternal- Child Health Nursing
D. International Council of Nurses Code
Answer: A
Explanation:
(A) The state nursing practice act determines the standard of care for the professional nurse.
(B) AWHONN Standards are published as recommendations and guidelines for maternalnewborn nursing.
(C) American Nurses Association Standards are published as recommendations and
guidelines for maternal child health nursing.
(D) The International Council of Nurses Code emphasizes the nurse’s obligations to the client
rather than to the physician. It is published as recommendations and guidelines by the
international organization for professional nursing.
QUESTION 682
A 26-year-old female client presents at 10 week’s gestation. She currently is a G3 1-0-1-1.
Her mother and grandmother have heart disease. Her grandmother also has insulin-dependent
diabetes. The client’s previous delivery was a term female infant weighing 9 lb 13 oz. The
client is 5 ft 6 inches tall and her current weight is 130 lb. Based on her history, she is at risk
for developing diabetes in pregnancy. Which of the following factors places her at risk for
gestational diabetes?
A. Age 25 years
B. Maternal weight
C. Previous birth of an infant weighing 9 lb
D. Family history of heart disease
Answer: C

Explanation:
(A) Maternal age older than 30 years is an identified risk factor for diabetes. Age younger
than 30 years is insignificant for diabetes unless there is a familial history of diabetes.
(B) The client’s weight is appropriate for her height. Obesity or pregnancy weight 20% of the
ideal weight is a contributing factor to the development of gestational diabetes.
(C) The birth of an infant weighing 9 lb (4000 g) is an identified risk factor for gestational
diabetes.
(D) A familial history of heart disease is insignificant in the development of diabetes.
However, a familial history of type II diabetes mellitus is identified as a risk factor in the
development of diabetes during pregnancy.
QUESTION 683
A client’s membranes have just ruptured spontaneously. Which of the following nursing
actions should take priority?
A. Assess quantity of fluid.
B. Assess color and odor of fluid.
C. Document on fetal monitor strip and chart.
D. Assess fetal heart rate (FHR).
Answer: D
Explanation:
(A) Assessing the quantity of amniotic fluid is important as an indication of maternal fetal
well-being, but it does not take priority over assessment of FHR.
(B) Greenish-brown discoloration of amniotic fluid indicates presence of meconium. Foul
odor may indicate presence of infection. Both of these are important assessment data, but they
do not take priority over possible life-threatening compression of the umbilical cord.
(C) Documentation is important, but it does not take priority over the possible life-threatening
compression of the umbilical cord.
(D) If changes in the FHR are noted, the nurse should check for umbilical cord prolapse. This
intervention has priority over the other actions. The danger of a prolapsed cord is increased
once membranes have ruptured, especially if the presenting part of the fetus does not fit
firmly against the cervix.

QUESTION 684
After several days, an IDDM client’s serum glucose stabilizes, and the registered nurse
continues client teaching in preparation for his discharge. The nurse helps him plan an
American Diabetes Association diet and explains how foods can be substituted on the
exchange list. He can substitute 1 oz of poultry for:
A. One frankfurter
B. One ounce of ham
C. Two slices of bacon
D. One-fourth cup dry cottage cheese
Answer: D
Explanation:
(A) A frankfurter is a high-fat meat on the diabetic exchange list.
(B) Ham is a medium-fat meat on the diabetic exchange list, unless it is a center-cut slice.
(C) One strip of bacon equals one fat exchange rather than a meat exchange. Dietary
substitutions should occur within exchange lists and not between exchange lists.
(D) Diabetic meat-exchange lists are categorized into lean meat foods, medium-fat meats, and
high-fat meats. Cottage cheese (dry, 2% butterfat), one-fourth cup, can substitute for one
lean-meat exchange.
QUESTION 685
An IDDM client’s condition stabilizes. He begins to receive a daily injection of NPH insulin
at 6:30 AM. The nurse can most likely expect a hypoglycemic reaction to occur that same day
at:
A. 8:30 AM–10:30 AM
B. 2:30 PM–4:30 PM
C. 7:30 PM–9:30 PM
D. 10:30 PM–11:30 PM
Answer: B
Explanation:
(A) This time describes the time of onset of NPH insulin’s action, rather than its peak effect.
(B) NPH insulin, an intermediate acting insulin, usually begins to lower serum glucose levels
about 2 hours after administration. The action of NPH insulin peaks 8–14 hours after
administration. It has a 20–30 hour duration.
(C) The time stated is not the time of peak action for NPH insulin administered at 6:30 AM.

(D) The time stated is not the time of peak action for NPH insulin administered at 6:30 AM.
QUESTION 686
A husband asks if he can visit with his wife on her ECT treatment days and what to expect
after the initial treatment. The nurse’s best response is:
A. “You’ll have to get permission from the physician to visit. Clients are pretty sick after the
first treatment.”
B. “Visitors are not allowed. We will telephone you to inform you of her progress.”
C. “There’s really no need to stay with her. She’s going to sleep for several hours after the
treatment.”
D. “Yes, you may visit. She may experience temporary drowsiness, confusion, or memory
loss after each treatment.”
Answer: D
Explanation:
(A) It is within the nurse’s realm of practice to grant visiting privileges according to hospital
policy. ECT treatments do not make clients sick.
(B) Visitors are allowed and encouraged, particularly family members.
(C) Clients are usually awake within 1 hour posttreatment.
Drowsiness wanes as the anesthetic wears off.
(D) A family member is encouraged to stay with the client after return to the unit. The nurse
has used an opportunity to do family teaching and allay fears by explaining temporary side
effects of the treatment.
QUESTION 687
A female client admitted to the labor and delivery unit thinks her bag of water “broke”
approximately 2 hours ago. She is having mild contractions 5 minutes apart. The most
immediate nursing intervention would be to:
A. Note the color and amount of fluid on her clothes.
B. Assess the FHR.
C. Notify the physician.
D. Place the nitrazine test paper at the cervical os and note the color change.
Answer: B
Explanation:

(A) Amniotic fluid is generally pale and straw colored. Meconium- stained amniotic fluid
would indicate a previous hypoxic episode. This intervention, though appropriate, is not the
immediate priority.
(B) With rupture of the membranes, the umbilical cord may prolapse if the presenting part
does not fill the pelvis. Assessing FHR ascertains fetal well-being.
(C) More information regarding fetal status and assessing for membrane rupture is needed
prior to contacting the physician.
(D) Nitrazine test paper differentiates amniotic fluid from urine. Amniotic fluid is normally
alkaline in contrast to urine, which is acidic. This intervention, though appropriate, is not the
immediate priority.
QUESTION 688
Medication is administered to a client who has been placed in restraints after a sudden violent
episode, and his EPSs subside. Restraints can be removed when:
A. The physician orders it
B. A therapeutic alliance has been established, and violent behavior subsides
C. The violent behavior subsides, and the client agrees to behave
D. The nurse deems that removal of restraints is necessary
Answer: B
Explanation:
(A) The physicianmay order release of restraints, but prior to that, the client must meet
criteria for release.
(B) While the client is still restrained, but after violent behavior has subsided, a therapeutic
bridge is built. This alliance encourages dialogue between nurse and client, allowing the
client to determine causative factors, feelings prior to loss of control, and adaptive
alternatives to violence.
(C) If the client only “agrees to behave after violent behavior subsides”, he has developed no
insight into cause and effect of violence or his response to stress.
(D) Removal of restraints occurs only when the client meets the criteria for release, not just
because the nurse says it is necessary.

QUESTION 689
The nurse assesses a client’s monitor strip and finds the following: uterine contractions every
3–4 minutes, lasting 60–70 seconds; FHR baseline 134–146 bpm, with accelerations to 158
bpm with fetal movement. Which nursing intervention is appropriate?
A. Notify physician of no reassuring FHR pattern.
B. Turn the client to her left side.
C. Start IV for fetal distress and administer O2 at 6–8 liters by mask.
D. Evaluate to see if the monitor strip is reassuring.
Answer: D
Explanation:
(A) These indices are within normal parameters; therefore, the nurse does not need to contact
the physician.
(B) The purpose of turning a client to her left side is to maximize uteroplacental blood flow.
Based on the above assessment, there is no indication that blood flow is compromised.
(C) These interventions are appropriate nursing interventions for late and prolonged
decelerations. Following these interventions, the nurse should notify the physician. These
indices are within normal parameters; therefore, the nurse does not need to start an IV and
administer O2.
(D) Variations of 20 bpm above or below the baseline FHR is considered normal. Normal
FHRs range from 120–160 bpm. As the fetus moves, the FHR increases, and accelerations
often occur in concert with contractions. During the active phase of labor, the frequency of
uterine contractions is every 2–4 minutes, with an appropriate duration of 60 sec.
QUESTION 690
A 14-year-old boy fell off his bike while “popping a wheelie on the dirt trails”. He has
sustained a head injury with laceration of his scalp over his temporal lobe. If he were to
complain of headache during the first 24 hours of his hospitalization, the nurse would:
A. Ask the physician to order a sedative
B. Have the client describe his headache every 15 minutes
C. Increase his fluid intake to 3000 mL/24 hr
D. Offer diversionary activities
Answer: D
Explanation:

(A) CNS depressants are not given for headache due to head injury because they would mask
changes in neurological status and because they could further depress the CNS.
(B) The client should not be asked to think about his headache every 15 minutes.
(C) Fluid intake should be normal or restricted for a client with a head injury. Normal fluid
intake for a 14 year old is about 2000–2400 mL daily.
(D) Diversion may help the child to focus on a pleasant activity instead of on his headache.
QUESTION 691
A client at 6 months gestation complains of tiredness and dizziness. Her hemoglobin level is
10 g/dL, and her hematocrit value is 32%. Her nutritional intake is assessed as sufficient. The
most likely diagnosis is:
A. Iron-deficiency anemia
B. Physiological anemia
C. Fatigue due to stress
D. No problem indicated
Answer: A
Explanation:
(A) This clinical situation is indicative of iron-deficiency anemia because the client has
inadequate nutritional intake. Her blood volume is increasing faster than her red blood cell
volume. Anemia is present in the second trimester when the hemoglobin level is 10.5 and the
hematocrit value falls below 35%. She needs increased iron supplements with follow-up.
(B) The client’s values are below levels for physiological anemia.
(C) The client is fatigued because of a low hemoglobin level.
(D) Her hemoglobin level is low and will probably decrease even more when the blood
volume peaks at 28 weeks.
QUESTION 692
A baby is circumcised. Immediate postoperative care should include:
A. Applying a loose diaper
B. Keeping the baby NPO for 4 hours to avoid vomiting
C. Changing the dressing frequently using dry, sterile gauze
D. Taking the baby to his mother for cuddling
Answer: D
Explanation:

(A) A pressure diaper should be applied to discourage hemorrhage.
(B) The baby can be fed by his mother soon after the procedure, once it is assessed that he is
not in any distress and is stable.
(C) Dressing changes should not be dry. Dry dressing will stick.
(D) Cuddling after the procedure will hopefully quiet the baby. Feeding is also important if
his feeding was withheld prior to the procedure or it is time for a feeding.
QUESTION 693
A client who has been diagnosed with anorexia nervosa reluctantly agrees to eat all prescribed
meals. The most important intervention in monitoring her dietary compliance would be to:
A. Allow her privacy at mealtimes
B. Praise her for eating everything
C. Observe behavior for 1–2 hours after meals to prevent vomiting
D. Encourage her to eat in moderation, choose foods that she likes, and avoid foods that she
dislikes
Answer: C
Explanation:
(A) Eating alone is not recommended for anorexic clients because they tend to hoard food
instead of eating it.
(B) The client should be praised for whatever she eats, which is usually a small portion or
percentage of what is served. Praise should not be withheld until she eats everything.
(C) The client should be observed eye to- eye for at least 1 hour following meals to prevent
discarding food stashed in her clothing at mealtime or engaging in self-induced vomiting.
(D) If offered these choices, the client would choose low-caloric foods, not a nutritious diet.
QUESTION 694
The nurse has been caring for a 16-year-old female who recently experienced date rape. After
having had crisis intervention and been hospitalized for 2 weeks, the nurse knows that the
client is effectively coping with the rape when she tells the nurse:
A. “I know it was my fault that it happened, because I shouldn’t have been out so late.”
B. “If I had not worn that sexy dress that night, he wouldn’t have raped me.”
C. “I know my date just had so much passion he couldn’t handle me saying no.”
D. “I know now that it was not my fault, but I want to continue counseling after my
discharge.”

Answer: D
Explanation:
(A) This response does not show any insight; the client falsely assumes that she is responsible
for the rape.
(B) The client continues to falsely assume responsibility for the rapist’s behavior.
(C) The client believes falsely that rape is an act of passion, rather than one of violence,
control, and domination.
(D) The client has insight into the rape; she does not believe it was her fault and shows good
judgment in deciding to continue with counseling after discharge.
QUESTION 695
A 24-year-old woman who is gravida 1 reports, “I can’t take iron pills because they make me
sick.” She continues, “My bowels aren’t moving either.” In counseling her based on these
complaints, the nurse’s most appropriate response would be, “It would be beneficial for you
to eat . . .”
A. prunes.
B. green leafy vegetables.
C. red meat.
D. eggs.
Answer: A
Explanation:
(A) Prunes provide fiber to decrease constipation and are an excellent source of dietary iron,
as the prenatal client is not taking her supplemental iron and iron-deficiency anemia is
common during pregnancy.
(B) Green leafy vegetables provide a source of fiber and iron; however, prunes are a better
source of both.
(C) Red meat is a good iron source but will not address the constipation problem.
(D) Eggs are a good iron source but do not address the constipation problem.
QUESTION 696
The physician of an alcoholic client places him on a low-protein, high-carbohydrate diet.
When choosing his menu, the client’s best choice from the items below would be:
A. Liver and onions, macaroni and cheese, tea with sugar
B. Baked chicken, baked potato with bacon bits, milk

C. Waffles with butter and honey, orange juice
D. Cheese omelette with ham and mushrooms, milk
Answer: C
Explanation:
(A, B, D) These foods are high in protein, which needs to be restricted.
(C) Serum ammonia levels can be decreased by restricting dietary protein intake. Waffles,
honey, and orange juice are high in carbohydrate and low or completely lacking in protein.
Butter, a concentrated fat, will provide extra calories.
QUESTION 697
In evaluating the effectiveness of magnesium sulfate (MgSO4), which of the following might
indicate that the client was developing MgSO4 toxicity?
A. A 31 patellar tendon reflex
B. Respirations of 12 breaths/min
C. Urine output of 40 mL/hr
D. A 21 proteinuria value
Answer: B
Explanation:
(A) Diminished (not accentuated) patellar tendon reflex is a sign of developing MgSO4
toxicity. A value of 21 is considered a normal tendon reflex; 3+ is considered brisker than
normal.
(B) MgSO4 is a central nervous system (CNS) depressant. It also relaxes smooth muscle. If
the respiratory rate is 16 bpm magnesium toxicity may be developing.
(C) Urine output of 40mL/hr is enough to allow elimination of toxic levels of magnesium.
Urinary output of 100 mL in a 4-hour period may result in toxic levels of magnesium.
(D) Presence of protein in the urine is a symptom of pregnancy-induced hypertension (PIH), a
clinical syndrome for which magnesium sulfate is frequently used in medical management.
Protein in the urine is not induced by magnesium sulfate intake.
QUESTION 698
Assessment of a client reveals a 30% loss of preillness weight, lanugo, and cessation of
menses for 3 months. Her vital signs are BP 90/50, P 96 bpm, respirations 30, and
temperature 97°F. She admits to the nurse that she has induced vomiting 3 times this

morning, but she had to continue exercising to lose “just 5 more lb”. Her symptoms are
consistent with:
A. Pregnancy
B. Bulimia
C. Gastritis
D. Anorexia nervosa
Answer: D
Explanation:
(A) Presenting behaviors collectively are inconsistent with depression.
(B) A preillness weight loss of 30%, lanugo, and cessation of menses are inconsistent with
bulimia.
(C) Symptoms and vital signs do not indicate the presence of infection.
(D) All symptoms and vital signs are consistent with anorexia nervosa.
QUESTION 699
A 16-year-old female client is admitted to the hospital because she collapsed at home while
exercising with videotaped workout instructions. Her mother reports that she has been
obsessed with losing weight and staying slim since cheerleader try-outs 6 months ago, when
she lost out to two of her best friends. The client is 5’4 and weighs 92 lb, which represents a
weight loss of 28 lb over the last 4 months. The most important initial intervention on
admission is to:
A. Obtain an accurate weight
B. Search the client’s purse for pills
C. Assess vital signs
D. Assign her to a room with someone her own age
Answer: C
Explanation:
(A) On admission, vital signs are the highest priority. Weight is not a vital sign.
(B) Belongings are routinely searched on admission to a psychiatric unit, but this search is not
a high priority.
(C) Vital signs are a high priority when working with self-destructive clients.
(D) Room assignment is of low priority.

QUESTION 700
A new mother experiences strong uterine contractions while breast-feeding her baby. She
excitedly rings for the nurse. When the nurse arrives the mother tells her, “Something is
wrong.” This is like my labor. Which reply by the nurse identifies the physiological response
of the client?
A. “Your breasts are secreting a hormone that enters your bloodstream and causes your
abdominal muscles to contract.”
B. “Prolactin increases the blood supply to your uterus, and you are feeling the effects of this
blood vessel engorgement.”
C. “The same hormone that is released in response to the baby’s sucking, causing milk to
flow, also causes the uterus to contract.”
D. “There is probably a small blood clot or placental fragment in your uterus, and your uterus
is contracting to expel it.”
Answer: C
Explanation:
(A) Mammary growth as well as milk production and maintenance in the breast occur in
response to hormones produced primarily by the hypothalamus and the pituitary gland.
(B) Prolactin stimulates the alveolar cells of the breast to produce milk. It is important in the
initiation of breast-feeding.
(C) Oxytocin, which is released by the posterior pituitary, stimulates the let-down reflex by
contraction of the myoepithelial cells surrounding the alveoli. In addition, it causes
contractions of the uterus and uterine involution.
(D) Afterpains may occur with retained placental fragments. A boggy uterus and continued
bleeding are other symptoms that occur in response to retained placental fragments.
QUESTION 701
A client presented herself to the mental health center, describing the following symptoms: a
weight loss of 20 lb in the past 2 months, difficulty concentrating, repeated absences from
work due to “fatigue”, and not wanting to get dressed in the morning. She leaves her recorded
message on her telephone and has lost interest in answering the phone or doorbell. The
nurse’s assessment of her behavior would most likely be:
A. Deep depression
B. Psychotic depression

C. Severe anxiety
D. Severe depression
Answer: D
Explanation:
(A) A client in deep depression would have been brought to the mental health center and
would not be physically able to seek help for herself.
(B) She is not manifesting psychotic symptoms in her behaviors.
(C) The client’s symptoms are more indicative of depression than anxiety.
(D) Although the client was able to bring herself to the mental health center, the extent of her
weight loss and the interference of symptoms with activities of daily living indicate that she is
severely depressed.
QUESTION 702
A client’s record from the ED indicates that she overdosed on phenelzine sulfate (Nardil), a
monoamine oxidase (MAO) inhibitor. Which diet would be the most appropriate at this time?
A. High carbohydrate, low cholesterol
B. High protein, high carbohydrate
C. 1 g sodium
D. Tyramine-free
Answer: D
Explanation:
(A) There are no data to support the need for increased carbohydrates or decreased
cholesterol in the diet.
(B) There is no data to support the need for increased protein or increased carbohydrates in
the diet.
(C) There is no assessment or laboratory data indicating that sodium should be restricted in
the diet.
(D) Tyramine is an amino acid activated by MAO in the liver and intestinal wall. It is released
as proteins are hydrolyzed through aging, pickling, smoking, or spoilage of foods. When
MAO is inhibited, tyramine levels rise, stimulating the adrenergic system to release large
amounts of norepinephrine, which can produce a hypertensive crisis.

QUESTION 703
Which of the following symptoms might the nurse observe in a client with a lithium blood
level over 2.0?
A. Fine hand tremor, headache, mental dullness
B. Vomiting, impaired consciousness, decreased blood pressure
C. Polyuria, polydipsia, edema
D. Gastric irritation, nausea, diarrhea
Answer: B
Explanation:
(A) These symptoms are acute, common, and usually harmless central nervous system side
effects of lithium.
(B) These symptoms of lithium toxicity are usually dose related.
(C) These symptoms are acute, common, and usually harmless renal side effects of lithium.
(D) These symptoms are acute, common, and usually harmless gastrointestinal side effects of
lithium.
QUESTION 704
A client with IDDM is given IV insulin for a blood glucose level of 520 mg/dL. Lifethreatening complications may occur initially, so the nurse will monitor him closely for
serum:
A. Chloride level of 99 mEq/L
B. Sodium level of 136 mEq/L
C. Potassium level of 3.1 mEq/L
D. Potassium level of 6.3 mEq/L
Answer: D
Explanation:
(A) The chloride level is within acceptable limits.
(B) The sodium level is within acceptable limits.
(C) This value indicates hypokalemia, rather than the hyperkalemia that occurs during
diabetic ketoacidosis.
(D) When diabetic ketoacidosis exists, intracellular dehydration occurs and potassium leaves
the cells and enters the vascular system, thus increasing the serum level beyond an acceptable
range. When insulin and fluids are administered, cell walls are repaired and potassium is
transported back into the cells. Normal serum potassium levels range from 3.5–5.0 mEq/L.

QUESTION 705
The nurse and prenatal client discuss the effects of cigarette smoking on pregnancy. It would
be correct for the nurse to explain that with cigarette smoking there is increased risk that the
baby will have:
A. A low birth weight
B. A birth defect
C. Anemia
D. Nicotine withdrawal
Answer: A
Explanation:
(A) Women who smoke during pregnancy are at increased risk for miscarriage, preterm labor,
and IUGR in the fetus.
(B) Although smoking produces harmful effects on the maternal vascular system and the
developing fetus, it has not been directly linked to fetal anomalies.
(C) Smoking during pregnancy has not been directly linked to anemia in the fetus.
(D) Smoking during pregnancy has not been linked to nicotine withdrawal symptoms in the
newborn.
QUESTION 706
A 26-year-old male client is brought by his wife to the emergency department (ED)
unconscious. Blood is drawn for a stat blood count (CBC), fasting blood sugar level, and
electrolytes. An indwelling urinary catheter is inserted. He has a history of type 1 diabetes
(insulin dependent diabetes mellitus [IDDM]). A diagnosis of ketoacidosis is made. Stat lab
values reveal a blood sugar level of 520 mg/dL. Which of the following should the nurse
expect to administer in the ER?
A. D50W by IV push
B. NPH insulin SC
C. Regular insulin by IV infusion
D. Sweetened grape juice by mouth
Answer: C
Explanation:
(A) This action would further increase the client’s blood sugar.

(B) NPH insulin is an intermediate-acting insulin, with an average of 4–6 hours before onset
of action. The client needs insulin that will act immediately. During a ketoacidotic state, the
client is dehydrated, so any insulin administered SC will be poorly absorbed.
(C) Regular insulin is the fastest acting-insulin; when given IV, it will immediately act to
decrease blood sugar. Regular insulin is given to decrease blood glucose levels by promoting
metabolism of glucose, inhibiting lipolysis and formation of ketone bodies.
(D) This action would further increase the client’s blood sugar.
QUESTION 707
In teaching the client about proper umbilical cord care, the nurse recommends that:
A. Petrolatum be placed around the cord after the sponge bath
B. A belly binder be applied to prevent umbilical hernia
C. The area be cleansed at diaper changes with alcohol and inspected for redness or drainage
D. The cord clamp be left on until the cord stump separates
Answer: C
Explanation:
(A) Petrolatum does not allow the cord to dry and will encourage infection.
(B) Belly binders do not facilitate drying of the cord and will encourage abdominal
relaxation.
(C) Frequent applications of alcohol will facilitate drying and discourage infection.
(D) The cord clamp can be removed in 24 hours. Leaving it on is cumbersome and could pull
on the cord unnecessarily.
QUESTION 708
A 24-hours postpartum client complains of discomfort at the episiotomy site. On assessment,
the nurse notes the episiotomy is without signs of infection. To relieve the discomfort, the
nurse should first:
A. Assist her with a sitz bath
B. Administer the prescribed medication for pain
C. Teach her Kegel exercises
D. Apply an ice pack
Answer: A
Explanation:

(A) Warm, moist heat will promote circulation and provide comfort. A sitz bath should be
tried before medication is given.
(B) Pain medication can be given when other comfort measures such as a sitz bath and topical
applications are ineffective.
(C) Kegel exercises facilitate sitting by decreasing tension on the episiotomy. They will not
be effective for pain control or sustained comfort level.
(D) Ice packs are appropriate to apply in the first 12 hours postdelivery to produce
vasoconstriction and to reduce edema to the area.
QUESTION 709
Due to his prolonged history of alcohol abuse, an alcoholic client will most likely have
deficiencies of which of the following nutrients?
A. Vitamin C and zinc
B. Folic acid and niacin
C. Vitamin A and biotin
D. Thiamine and pyroxidine
Answer: D
Explanation:
(A) Chronic alcoholism can lead to deficiencies of B complex vitamins including thiamine
and pyroxidine.
(B) Chronic alcoholism can lead to deficiencies of vitamins A, D, K, and B complex.
(C) Chronic alcoholism can lead to deficiencies of vitamins A, D, K, and B complex.
(D) Vitamins A, D, K, and B require bile salts to be absorbed from the gastrointestinal tract. A
damaged liver does not form bile salts.
QUESTION 710
A female client has been treated since childhood for mitral valve prolapse. The antibiotic of
choice for her during pregnancy would be:
A. Sulfa
B. Tetracycline
C. Hydralazine
D. Erythromycin
Answer: D
Explanation:

(A) Sulfa is a teratogen and will cause kernicterus.
(B) Tetracycline is a teratogen and will affect tooth development.
(C) Hydralazine is not an antibiotic but a calcium channel blocker.
(D) Erythromycin is safe during pregnancy and can be used when the client is allergic to
penicillin.
QUESTION 711
A client is being admitted to the labor and delivery unit. She has had previous admissions for
“false labor”. Which clinical manifestation would be most indicative of true labor?
A. Increased bloody show
B. Progressive dilatation and effacement of the cervix
C. Uterine contractions
D. Decreased discomfort with ambulation
Answer: B
Explanation:
(A) Bloody show is considered a sign of imminent labor, which usually begins in 24–48
hours. An increase in bloody show is an indication that the cervix is changing.
(B) Contractions of true labor produce progressive cervical effacement and dilatation.
(C) Contractions of false labor may mimic those of true labor. However, the contractions of
false labor do not produce progressive effacement and dilatation of the cervix.
(D) In true labor, the discomfort is not relieved by ambulation; walking may intensify the
discomfort.
QUESTION 712
A 2-year-old boy fell out of bed and has a subdural hematoma. When his mother leaves him
for the first time, you will expect the child to:
A. Be comforted when he is held
B. Cry
C. Not notice that his mother has left
D. Withdraw and become listless
Answer: B
Explanation:
(A) It will be difficult to comfort a 2 year old with a headache without his mother.

(B) This baby probably will cry, which should be prevented because it will increase his
intracranial pressure (ICP). Asking the mother to wait until the baby is asleep may help.
(C) An awake 2 year old will notice when his mother leaves.
(D) An older child may withdraw when feeling afraid, but a 2 year old will probably show
more aggressive behavior.
QUESTION 713
A client undergoes a transurethral resection, prostate (TURP). He returns from surgery with a
three-way continuous Foley irrigation of normal saline in progress. The purpose of this
bladder irrigation is to prevent:
A. Bladder spasms
B. Clot formation
C. Scrotal edema
D. Prostatic infection
Answer: B
Explanation:
(A) The purpose of bladder irrigation is not to prevent bladder spasms, but to drain the
bladder and decrease clot formation and obstruction.
(B) A three-way system of bladder irrigation will cleanse the bladder and prevent formation
of blood clots. A catheter obstructed by clots or other debris will cause prostatic distention
and hemorrhage.
(C) Scrotal edema seldom occurs after TURP. Bladder irrigation will not prevent this
complication.
(D) Prostatic infection seldom occurs after TURP. Bladder irrigation will not prevent this
complication.
QUESTION 714
A 2-year-old child is recovering from surgery. Considering growth and development
according to Erikson, the nurse identifies which of the following play activities as most
appropriate?
A. Assembling a puzzle with large pieces
B. Being taken for a wheelchair ride
C. Listening to a story about the Muppets
D. Watching Sesame Street on television

Answer: A
Explanation:
(A) A 2-year-old child is in the stage of autonomy, according to Erikson. Assembling a puzzle
with large pieces enables her to “do it herself”.
(B) A wheelchair ride would probably be fun, but it is not directed toward helping the child to
achieve autonomy.
(C) Listening to a story may be fun and educational, but it is not directed toward helping the
child to achieve autonomy.
(D) Watching television may be a favorite activity, but it does not foster autonomy.
QUESTION 715
A 30-year-old client in the third trimester of her pregnancy asks the nurse for advice about
upper respiratory discomforts. She complains of nasal stuffiness and epistaxis, most
noticeable on the left side. Which reply by the nurse is correct?
A. “It sounds as though you are coming down with a bad cold. I’ll ask the doctor to prescribe
a decongestant for relief of symptoms.”
B. “A good vaporizer will help; avoid the cool air kind. Also, try saline nose drops, and spend
less time on your left side.”
C. “These discomforts are all a result of increased blood supply; one of the pregnancy
hormones, estrogen, causes them.”
D. “This is most unusual. I’m sure your obstetrician will want you to see an ENT (ear, nose,
throat) specialist.”
Answer: C
Explanation:
(A) Decongestants may exaggerate the nasal stuffiness associated with pregnancy. Judicious
use of decongestants and nasal sprays is advocated during pregnancy.
(B) Cool air vaporizers and saline drops may help to relieve the nasal stuffiness. Positioning
on either lateral side does not decrease nasal stuffiness or prevent epistaxis.
(C) Increased estrogen levels result in nasal mucosa edema with subsequent nasal stuffiness.
Estrogen also promotes vasodilation, which contributes to epistaxis. The nurse may
recommend cool air vaporizers and saline drops to help with the nasal stuffiness.
(D) Increased estrogen levels result in nasal mucosa edema with subsequent nasal stuffiness.
Estrogen also promotes vasodilation discomforts associated with pregnancy.

QUESTION 716
The nurse is caring for a 6-week-old girl with meningitis. To help her develop a sense of trust,
the nurse should:
A. Give her a small soft blanket to hold
B. Give her good perineal care after each diaper change
C. Leave the door open to her room
D. Pick her up when she cries
Answer: D
Explanation:
(A) A soft blanket may be comforting, but it is not directed toward developing a sense of
trust.
(B) Good perineal care is important, but it is not directed toward developing a sense of trust.
(C) An infant with meningitis needs frequent attention, but leaving the door open does not
foster trust.
(D) Consistently picking her up when she cries will help the child feel trust in her caregivers.
QUESTION 717
Early in her ninth month of pregnancy, a client has been diagnosed as having mild
preeclampsia. In counseling her about her diet, the nurse must emphasize the importance of:
A. Decreasing her sodium intake
B. Decreasing her fluids
C. Increasing her carbohydrate intake
D. Eating a moderate to high-protein diet
Answer: D
Explanation:
(A) Women with pregnancy-induced hypertension have a reduced plasma volume secondary
to venous vessel constriction, not hypovolemia; therefore, sodium restriction is not
recommended. It is suggested that these women avoid extremely salty foods.
(B) Drinking six to eight glasses of water per day facilitates optimal fluid volume and renal
perfusion, but it will not decrease the venous vessel constriction of pregnancy-induced
hypertension.
(C) Carbohydrate needs increase during pregnancy, specifically during the second and third
trimesters, but they have not been linked to pregnancy-induced hypertension.

(D) Loss of urinary protein (proteinuria) is associated with increased permeability of the large
protein molecules with pregnancy-induced hypertension. Additional dietary protein also helps
increase the plasma colloidal osmotic pressure. Diets deficient in protein have been linked to
pregnancy-induced hypertension.
QUESTION 718
A client is having an amniocentesis. Prior to the procedure, an ultrasound is performed. In
preparing the client, the nurse explains the reason for a sonogram in this situation to be:
A. Determination of multiple gestations
B. Determination of gross anomalies
C. Determination of placental location
D. Determination of fetal age
Answer: C
Explanation:
(A) Sonography can be used to determine the presence of multiple gestation. In this question,
the sonogram is used as a preparatory step for a specific invasive procedure.
(B) Sonography can be used to determine the presence of gross anomalies. In this question,
the sonogram is used as a preparatory step for a specific invasive procedure.
(C) Prior to amniocentesis, the abdomen is scanned by ultrasound to locate the placenta, thus
reducing the possibility of penetrating it with the spinal needle used to obtain amniotic fluid.
(D) Sonography can be used to determine fetal age. In this question, the sonogram is used as
a preparatory step for a specific invasive procedure.
QUESTION 719
The nurse is caring for a 3-month-old girl with meningitis. She has a positive Kernig’s sign.
The nurse expects her to react to discomfort if she:
A. Dorsiflexes her ankle
B. Flexes her spine
C. Plantiflexes her wrist
D. Turns her head to the side
Answer: B
Explanation:
(A) Discomfort with ankle dorsiflexion is not expected with meningitis.

(B) Spinal flexion, flexing the neck or the hips with legs extended, causes discomfort if the
meninges are irritated.
(C) Discomfort with wrist flexion is not expected with meningitis.
(D) Rotating the cervical spine may cause discomfort with meningitis, but pain with flexion is
more indicative of meningeal irritation.
QUESTION 720
On the third postpartum day, a client complains of extremely tender breasts. On palpation, the
nurse notes a very firm, shiny appearance to the breasts and some milk leakage. She is bottle
feeding. The nurse should initially recommend to her to:
A. Take 2 ibuprofen (Motrin) tablets by mouth now because the baby will be returning for
feeding in 20 minutes
B. Allow the infant to breast-feed at the next feeding time to empty the breasts
C. Apply ice packs to the breasts and wear a supportive, well-fitting bra
D. Take a warm shower and express milk from both breasts until empty
Answer: C
Explanation:
(A) Judicious use of analgesics is appropriate with breast engorgement; however, mechanical
suppression would be the initial recommendation.
(B) Breast-feeding every 112–3 hours will reduce and/or prevent breast engorgement. Breastfeeding will promote milk production, which will compound the distention and stasis of the
venous circulation of engorgement in a bottle-feeding mother.
(C) Ice packs reduce milk flow while the snug, supportive bra provides mechanical
suppression and decreases pulling on Cooper’s ligament. In addition, breast binders or ace
bandages may be used for some women.
(D) Warmth promotes milk production and may stimulate the let-down reflex. These
measures would contribute to the venous congestion of engorgement.
QUESTION 721
A psychiatric nurse is providing an orientation to a new staff nurse. She reminds the nurse
that psychiatrists often use categories of medications and that it is important that she recall
that some categories of medications have synonyms. Another name used to describe minor
tranquilizers is which of the following?
A. Antipsychotic medications

B. Antidepressant medications
C. Antianxiety medications
D. Antimania medication
Answer: C
Explanation:
(A) Antipsychotic medications are also known as major tranquilizers.
(B) Antidepressants fall into different categories, such as the tricyclics or the MAO inhibitors.
(C) Antianxiety medications are also known as minor tranquilizers.
(D) Antimania medications are those such as lithium and lithium carbonate (Lithobid).
QUESTION 722
A client is medically cleared for ECT and is tentatively scheduled for six treatments over a 2week period. Her husband asks, “Isn’t that a lot?” The nurse’s best response is:
A. “Yes, that does seem like a lot.”
B. “You’ll have to talk to the doctor about that. The physician knows what’s best for the
client.”
C. “Six to 10 treatments are common. Are you concerned about permanent effects?”
D. “Don’t worry. Some clients have lots more than that.”
Answer: C
Explanation:
(A) This response indicates that the nurse is unsure of herself and not knowledgeable about
ECT. It also reinforces the husband’s fears.
(B) This response is “passing the buck unnecessarily.” The information needed to
appropriately answer the husband’s question is well within the nurse’s knowledge base.
(C) The most common range for affective disorders is 6–10 treatments. This response
confirms and reinforces the physician’s plan for treatment. It also opens communication with
the husband to identify underlying fears and knowledge deficits.
(D) This response offers false reassurance and dismisses the husband’s underlying concerns
about his wife.
QUESTION 723
A client is pleased about being pregnant, yet states, “It is really not the best time”, but I guess
it will be OK. The nurse’s assessment of this response is:
A. Initial maternal-infant bonding may be poor.

B. Client may have a poor relationship with her husband.
C. This response is normal in the first trimester.
D. This response is abnormal, to be re-evaluated at the next visit.
Answer: C
Explanation:
(A) Ambivalence is normal during the first trimester. Reva Rubin addresses the issue of “not
now in the first trimester.” The statement still leaves room for exploration.
(B) There are no data to support this. This statement by the mother still leaves room for
exploration.
(C) Ambivalenceis normal during the first trimester. Reva Rubin addresses the issue of “not
now.” This fact should be shared with the mother during further exploration of the comment.
(D) It is not abnormal. If it were, another month would also be too long to wait.
QUESTION 724
A client suffering from schizophrenia has been taking chlorpromazine (Thorazine) for 6
months. On one of his follow-up visits to the mental health center, the nurse reports to the
physician that he has developed tardive dyskinesia. Which of the following symptoms might
she have observed in the client to support this conclusion?
A. High fever, tachycardia, stupor, renal failure
B. Lip smacking, chewing, blinking, lateral jaw movements
C. Photosensitivity, orthostatic hypotension, dry mouth
D. Constipation, blurred vision, drowsiness
Answer: B
Explanation:
(A) These symptoms are found in clients with neuroleptic malignant syndrome.
(B) These symptoms are found in clients with tardive dyskinesia.
(C) These are normal side effects found in clients taking antipsychotic medications.
(D) These are also normal side effects found in clients taking antipsychotic medications.
QUESTION 725
A pregnant client complains of varicosities in the third trimester. Which of the following
activities should she be advised to avoid?
A. Sitting with legs crossed at ankles
B. Wearing thromboembolic disease (TED) stockings

C. Wearing support pantyhose
D. Wearing knee-high stockings
Answer: D
Explanation:
(A) Sitting with the legs crossed at the ankles does not interfere with circulation or create
pressure points.
(B) TED stockings will help to reduce the varicosity by supporting the vein. Stockings must
be applied with legs elevated.
(C) Support pantyhose help to reduce the varicosity by supporting the vein. They also provide
support to the uterus and allow for better return circulation. Hose must be applied like TED
stockings.
(D) Knee-high stockings create constriction and pressure points that interfere with circulation
in the lower extremities.
QUESTION 726
The nurse is trying to help a mother understand what is happening with her son who has
recently been diagnosed with paranoid schizophrenia. At present, he is experiencing
hallucinations and delusions of persecution and suffers from poor hygiene. The nurse can best
help her understand her son’s condition by which of the following statements?
A. “Sometimes these symptoms are caused by an overstimulation of a chemical called
dopamine in the brain.”
B. “Has anyone in your family ever had schizophrenia?”
C. “If your son has a twin, he probably will eventually develop schizophrenia, too.”
D. “Some of his symptoms may be a result of his lack of a strong mother-child bonding
relationship.”
Answer: A
Explanation:
(A) The most plausible theory to date is that dopamine causes an overstimulation in the brain,
which results in the psychotic symptoms.
(B) This statement will only create anxiety in the mother, and the genetic theory is only one
of the etiological factors.
(C) This statement will cause the mother much alarm, and nothing was mentioned about any
other child.

(D) The mother child relationship is one of the previous theories examined, but it is not one to
be emphasized, thereby causing a lot of anxiety for the mother.
QUESTION 727
A couple is experiencing difficulties conceiving a baby. The nurse explains basal body
temperature (BBT) by instructing the female client to take her temperature:
A. Orally in the morning and at bedtime
B. Only one time during the day as long as it is always at the same time of day
C. Rectally at bedtime
D. As soon as she awakens, prior to any activity
Answer: D
Explanation:
(A) Monitoring temperature twice a day predicts the biphasic pattern of ovulation.
(B) Prediction of ovulation relies on consistency in taking temperature.
(C) Nightly rectal temperatures are more accurate in predicting ovulation.
(D) Activity changes the accuracy of basal body temperature and ability to detect the
luteinizing hormone surge.
QUESTION 728
A 23-year-old male client is admitted to the chemical dependency unit with a medical
diagnosis of alcoholism. He reports that the last time he drank was 3 days ago, and that now
he is starting to “feel kind of shaky”. Based on the information given above, nursing care
goals for this client will initially focus on:
A. Self-concept problems
B. Interpersonal issues
C. Ineffective coping skills
D. Physiological stabilization
Answer: D
Explanation:
(A) Self-concept and self-esteem problems may emerge during the client’s treatment, but
these are not immediate concerns.
(B) Interpersonal issues may become evident during the course of the client’s treatment, but
these are also not immediate areas of concern.

(C) Improving individual coping skills is generally a primary focus in the treatment and
nursing care of persons with substance abuse problems. However, this is still not the
immediate concern in this client situation.
(D) Correction of fluid and electrolyte status and vitamin deficiencies, as well as prevention
of delirium, is the immediate concern in the care of this client.
QUESTION 729
Often children are monitored with pulse oximeter. The pulse oximeter measures the:
A. O2 content of the blood
B. Oxygen saturation of arterial blood
C. PO2
D. Affinity of hemoglobin for O2
Answer: B
Explanation:
(A) The O2 content of whole blood is determined by the partial pressure of oxygen (PO2) and
the oxygen saturation. The pulse oximeter does not measure the PO2.
(B) The pulse oximeter is a non-invasive method of measuring the arterial oxygen saturation.
(C) The PO2 is the amount of O2 dissolved in plasma, which the pulse oximeter does not
measure.
(D) The affinity of hemoglobin for O2 is the relationship between oxygen saturation and PO2
and is not measured by the pulse oximeter.
QUESTION 730
The nurse writes the following nursing diagnosis for a client in acute renal failure—Impaired
gas exchange related to:
A. Decreased red blood cell production
B. Increased levels of vitamin D
C. Increased red blood cell production
D. Decreased production of renin
Answer: A
Explanation:
(A) Red blood cell production is impaired in renal failure owing to impaired erythropoietin
production. This causes a decrease in the delivery of oxygen to the tissue and impairs gas
exchange.

(B) The conversion of vitamin D to its physiologically active form is impaired in renal
failure.
(C) In renal failure, a decrease in red blood cell production occurs owing to an impaired
production of erythropoietin, leading to impaired gas exchange at the cellular level.
(D) The decreased production of renin in renal failure causes an increased production of
aldosterone causing sodium and water retention.
QUESTION 731
In caring at home for a child who just ingested a caustic alkali, the nurse would immediately
tell the mother to:
A. Give vinegar, lemon juice, or orange juice
B. Phone the doctor
C. Take the child to the emergency room
D. Induce vomiting
Answer: A
Explanation:
(A) The immediate action is to neutralize the action of the chemical before further damage
takes place.
(B) This action should be done after neutralizing the chemical.
(C) This action should be done after neutralizing the chemical.
(D) Never induce vomiting with a strong alkali or acid. Additional damage will be done when
the child vomits the chemical.
QUESTION 732
A client hospitalized with a medical diagnosis of adjustment disorder versus personality
disorder states, “Nobody cares about the clients”. The nurse’s most effective response would
be:
A. “How can you say that I don’t care? We just met.”
B. “What makes you think the nurses don’t care?”
C. “You will feel differently about us in a few days.”
D. “You seem angry. Tell me more about how you feel.”
Answer: D
Explanation:

(A) This statement is a defensive response that places the nurse in a vulnerable
countertransference position, and at the same time, fails to challenge the client’s “splitting
behavior.”
(B) This statement is a defensive response by the nurse. In addition, this type of
nontherapeutic statement requests that the client explain the reasons for her behavior, a
difficult task for an individual with limited insight.
(C) This statement is a nontherapeutic response that both ignores the intensity of the client’s
emotions and the dynamics underlying “splitting behavior.”
(D) By simultaneously acknowledging the client’s emotional intensity and gently challenging
her “splitting behavior”, the nurse addresses the client’s current distortions and prepares for
further interventions with angry or ambivalent feelings.
QUESTION 733
A 6-month-old infant who was diagnosed at 4 weeks of age with a ventricular septal defect,
was admitted today with a diagnosis of failure to thrive. His mother stated that he had not
been eating well for the past month. A cardiac catheterization reveals congestive heart failure.
All of the following nursing diagnoses are appropriate. Which nursing diagnosis should have
priority?
A. Altered nutrition: less than body requirements related to inability to take in adequate
calories
B. Altered growth and development related to decreased intake of food
C. Activity intolerance related to imbalance between oxygen supply and demand
D. Decreased cardiac output related to ineffective pumping action of the heart
Answer: D
Explanation:
(A) Altered nutrition occurs owing to the fatigue from decreased cardiac output associated
with congestive heart failure.
(B) The decreased intake occurs due to fatigue from the altered cardiac output.
(C) Fatigue occurs due to the decreased cardiac output.
(D) The ineffective action of the myocardium leads to inadequate O2 to the tissues, which
produces activity intolerance, altered nutrition, and altered growth and development.

QUESTION 734
One of the most reliable assessment tools for adequacy of fluid resuscitation in burned
children is:
A. Blood pressure
B. Level of consciousness
C. Skin turgor
D. Fluid intake
Answer: B
Explanation:
(A) Blood pressure can remain normotensive in a state of hypovolemia.
(B) Capillary refill, alterations in sensorium, and urine output are the most reliable indicators
for assessing hydration.
(C) Skin turgor is not a reliable indicator for assessing hydration in a burn client.
(D) Fluid intake does not indicate adequacy of fluid resuscitation in a burn client.
QUESTION 735
A 35-year-old client is receiving psychopharmacological treatment of his major depression
with tranylcypromine sulfate (Parnate), a monoamine oxidase (MAO) inhibitor. The nurse
teaches the client that while he is taking this type of antidepressant, he needs to restrict his
dietary intake of:
A. Potassium-rich foods
B. Tryptophan
C. Tyramine
D. Saturated fats
Answer: C
Explanation:
(A) The client may need to avoid some potassium-rich foods (such as bananas, raisins, etc.).
However, this is not because of the potassium content of these foods.
(B) Tryptophan is an essential amino acid that is present in high concentrations in animal and
fish protein.
(C) The client will need to watch his dietary intake of tyramine. Tyramine is a by-product of
the conversion of tyrosine to epinephrine. Tyramine is found in a variety of foods and
beverages, ranging from aged cheese to caffeine drinks. Ingestion of tyramine-rich foods

while taking a MAO inhibitor may lead to an increase in blood pressure and/or a lifethreatening hypertensive crisis.
(D) To maintain a healthy lifestyle, restriction of dietary saturated fats is advisable.
QUESTION 736
A 19-month-old child is admitted to the hospital for surgical repair of patent ductus arteriosus.
The child is being given digoxin. Prior to administering the medication, the nurse should:
A. Not give the digoxin if the pulse is 60
B. Not give the digoxin if the pulse is 100
C. Take the apical pulse for a full minute
D. Monitor for visual disturbances, a side effect of digoxin
Answer: C
Explanation:
(A) Digoxin should not be given to adults with an apical pulse 60 bpm.
(B) Digoxin should be given to children with an apical pulse 100 bpm. With a pulse 100 bpm,
the medication should be withheld and the physician notified.
(C) Prior to digoxin administration in both children and adults, an apical pulse should be
taken for 1 full minute. Aside from the rate per minute, the nurse should note any sudden
increase or decrease in heart rate, irregular rhythm, or regularization of a chronic irregular
heart rhythm.
(D) Early indications of digoxin toxicity, such as visual disturbances, occur rarely as initial
signs in children.
QUESTION 737
Two hours after the second injection of haloperidol, a client complains to the nurse of a stiff
neck and inability to sit still. He is experiencing symptoms consistent with:
A. Parkinsonism and dystonia
B. Dystonia and akathisia
C. Akathisia and parkinsonism
D. Neuroleptic malignant syndrome
Answer: B
Explanation:
(A) Stiff neck is consistent with a dystonic reaction, but the client has no symptoms of
drooling, shuffling gait, or pill-rolling movements characteristic of parkinsonism.

(B) Stiff neck is consistent with a dystonic reaction, and inability to sit still with varying
degrees of psychomotor agitation is characteristic of akathisia.
(C) The client has symptoms of dystonia but not of parkinsonism.
(D) The client has none of the characteristic symptoms of neuroleptic malignant syndrome:
hyperpyrexia, generalized muscle rigidity, mutism, obtundation, agitation, sweating,
increased blood pressure and pulse.
QUESTION 738
Three hours postoperatively, a 27-year-old client complains of right leg pain after knee
reduction. The first action by the nurse will be to:
A. Assess vital signs
B. Elevate the extremity
C. Perform a lower extremity neurovascular check
D. Remind the client that he has a client-controlled analgesic pump, and reinstruct him on its
use
Answer: C
Explanation:
(A) Vital signs may be altered if there is acute pain or complications related to bleeding or
swelling, but they should not be assessed before checking the affected extremity.
(B) The extremity will be elevated if ordered by the doctor.
(C) Assessment of the postoperative area is important to determine if bleeding, swelling, or
decreased circulation is occurring.
(D) Reinforcement of teaching on use of the client-controlled analgesic pump is important,
but not the first action.
QUESTION 739
A 4-year-old boy is brought to the emergency room with bruises on his head, face, arms, and
legs. His mother states that he fell down some steps. The nurse suspects that he may have
been physically abused. In accordance with the law, the nurse must:
A. Tell the physician her concerns
B. Report her suspicions to the authorities
C. Talk to the child’s father
D. Confront the child’s mother
Answer: B

Explanation:
(A) Although the nurse probably would talk to the physician about these concerns, the nurse
is not required by law to do so.
(B) All healthcare workers are required by the Federal Child Abuse Prevention and Treatment
Act of 1974 to report suspected and actual cases of child abuse and/or neglect.
(C) Talking to the child’s father may or may not help the child, and the nurse is not required
by law to do so.
(D) Confrontation may not be indicated; the nurse is not required by law to confront the
child’s mother with these suspicions.
QUESTION 740
Parents of a child with rheumatic fever express concern that she will always be arthritic. The
nurse discusses their concerns and tells them the joint pain usually:
A. Subsides in 3 weeks
B. Is relieved by aspirin
C. Is responsive to ibuprofen (Motrin)
D. Subsides in 3–6 days
Answer: A
Explanation:
(A) Joints usually remain inflamed and tender until the disease runs its course in 3 weeks.
(B) This response does not answer the question of whether she will always be arthritic.
(C) This response does not answer the question asked.
(D) The disease takes 3 weeks to run its course.
QUESTION 741
A child is admitted with severe headache, fever, vomiting, photophobia, drowsiness, and stiff
neck associated with viral meningitis. She will be more comfortable if the nurse:
A. Dims the lights in her room
B. Encourages her to breathe slowly and deeply
C. Offers sips of warm liquids
D. Places a large, soft pillow under her head
Answer: A
Explanation:
(A) The discomfort of photophobia is alleviated by dimming the lights.

(B) Helping the child to breathe slowly and deeply may help to reduce anxiety, but it will not
alleviate other discomforts of viral meningitis.
(C) It is important to maintain fluid balance, but sips of warm liquids do not alleviate the
discomforts of meningitis.
(D) A large, soft pillow under her head causing neck flexion is likely to increase her
discomfort owing to stretching of the meninges.
QUESTION 742
Which stage of labor lasts from delivery of the baby to delivery of the placenta?
A. Second
B. Third
C. Fourth
D. Fifth
Answer: B
Explanation:
(A) This stage is from complete dilatation of the cervix to delivery of the fetus.
(B) This is the correct stage for the definition.
(C) This stage lasts for about 2 hours after the delivery of the placenta.
(D) There is no fifth stage of labor.
QUESTION 743
A 3-year-old female client is brought into the pediatric clinic because she limps. She has not
been to the clinic since she was 9 months old. The nurse practitioner describes the limp as a
“Trendelenburg gait”. This gait is characteristic of:
A. Scoliosis
B. Dislocated hip
C. Fractured femur
D. Fractured pelvis
Answer: B
Explanation:
(A, C, D) A Trendelenburg gait is not characteristic of any of these disorders.
(B) The downward slant of one hip is a positive sign of dislocation in the weight-bearing hip.
If one hip is dislocated, the child walks with a characteristic limp known as the Trendelenburg
gait.

QUESTION 744
A 16-year-old diabetic girl has been selected as a cheerleader at her school. She asks the nurse
whether she should increase her insulin when she is planning to attend cheerleading practice
sessions lasting from 8 to 11 AM. The most appropriate answer would be:
A. “You should ask your doctor about this.”
B. “Yes, increase your insulin by 1 U for each hour of practice because exercise causes the
body to need more insulin.”
C. “No, do not increase your insulin. Exercise will not affect your insulin needs.”
D. “No, do not increase your insulin, but eating a snack prior to practice exercise will make
insulin more effective and move more glucose into the cells.”
Answer: D
Explanation:
(A) A nurse can give this information to a client.
(B) Exercise makes insulin more efficient in moving more glucose into the cells. No more
insulin is needed.
(C) Exercise makes insulin more efficient unless the diabetes is poorly controlled.
(D) Exercise makes insulin more efficient in moving more glucose into the cells.
QUESTION 745
A 14-year-old teenager is hospitalized for anorexia nervosa. She is admitted to the adolescent
mental health unit and placed on a behavior modification program. Nursing interventions for
the teenager will most likely include:
A. Establishing routine tasks and activities around mealtimes
B. Administering medications such as lithium
C. Requiring the client to eat more during meals
D. Checking the client’s room frequently
Answer: A
Explanation:
(A) Providing a more structured, supportive environment addresses safety and comfort needs,
thereby helping the anorexic client develop more internal control.
(B) Medications (commonly antidepressants) are frequently ordered for the anorexic client.
However, lithium (used primarily with bipolar disorder) is not commonly used to treat the
anorexic client.

(C) Requiring and/or demanding that the anorexic client “eat more at mealtimes increases the
client’s feelings of powerlessness.”
(D) Like the previous strategy, checking the client’s room frequently contributes to the
client’s feelings of powerlessness.
QUESTION 746
A 47-year-old male client is admitted for colon surgery. Intravenous antibiotics are begun 2
hours prior to surgery. He has no known infection. The rationale for giving antibiotics prior to
surgery is to:
A. Provide cathartic action within the colon
B. Reduce the risk of wound infection from anaerobic bacteria
C. Relieve the client’s concern regarding possible infection
D. Reduce the risk of intraoperative fever
Answer: B
Explanation:
(A) Cathartic drugs promote evacuation of intestinal contents.
(B) The client undergoing intestinal surgery is at increased risk for infection from large
numbers of anaerobic bacteria that inhabit the intestines. Administering antibiotics
prophylactically can reduce the client’s risk for infection.
(C) Antibiotics are indicated in the treatment of infections and have no effect on emotions.
(D) Antipyretics are useful in the treatment of elevated temperatures. Antibiotics would have
an effect on infection, which causes temperature elevation, but would not directly affect such
an elevation.
QUESTION 747
A 23-year-old female client is brought to the emergency room by her roommate for repeatedly
making superficial cuts on her wrists and experiencing wide mood swings. She is very angry
and hostile. Her medical diagnosis is adjustment disorder versus borderline personality
disorder. The client comments to the nurse,
“Nobody in here seems to really care about the clients.” I thought nurses cared about people!
The client is exhibiting the ego defense mechanism:
A. Reaction formation
B. Rationalization
C. Splitting

D. Sublimation
Answer: C
Explanation:
(A) Reaction formation is the development and demonstration of attitudes and/or behaviors
opposite to what an individual actually feels. The client’s comment does reveal her anger and
hostility.
(B) Rationalization, another ego defense mechanism, is offering a socially acceptable or
seemingly logical explanation to justify one’s feelings, behaviors, or motives. The client’s
comment does not reflect rationalization.
(C) Splitting, the viewing of people or situations as either all good or all bad, is frequently
used by persons experiencing a disruption in self-concept. This ego defense mechanism is
reflective of the individual’s inability to integrate the positive and negative aspects of self.
(D) Sublimation, the channeling of socially unacceptable impulses and behaviors into more
acceptable patterns of behavior, is another ego defense mechanism. The client’s comment
reveals that she is not engaging in sublimation.
QUESTION 748
A 38-year-old female client with a history of chronic schizophrenia, paranoid type, is
currently an outpatient at the local mental health and mental retardation clinic. The client
comes in once a week for medication evaluation and/or refills. She self-administers
haloperidol 5 mg twice a day and benztropine 1 mg once a day. During a recent clinic visit,
she says to the nurse, “I can’t stay still at night”. I toss and turn and can’t fall asleep. The
nurse suspects that she may be experiencing:
A. Akathisia
B. Akinesia
C. Dystonia
D. Opisthotonos
Answer: A
Explanation:
(A) Akathisia, or motor restlessness, is a reversible EPS frequently associated with the
administration of antipsychotic drugs such as haloperidol.
(B) Akinesia, or muscular or motor retardation, is an example of reversible EPS frequently
associated with the administration of major tranquilizers such as haloperidol.

(C) Acute dystonic reactions, bizarre and severe muscle contractions usually of the tongue,
face, neck or extraocular muscles, are examples of EPS.
(D) Opisthotonos, a severe type of whole-body dystonic reaction in which the head and heels
are bent backward while the body is bowed forward, is an example of EPS.
QUESTION 749
Which of the following nursing orders has the highest priority for a child with epiglottitis?
A. Vital signs every shift
B. Tracheostomy set at bedside
C. Intake and output
D. Specific gravity every shift
Answer: B
Explanation:
(A) Because of the possibility of fever or respiratory failure, vital signs should be done more
often than every eight hours.
(B) If the epiglottitis worsens, the edema and laryngospasm may close the airway and an
emergency tracheostomy may be necessary.
(C) Although intake and output are a part of the nursing care of a child with epiglottitis, it is
not as important as the safety measure of keeping the tracheostomy set at the bedside.
(D)
Specific gravity will indicate hydration status, but it is not as important as keeping the
tracheostomy set at the bedside.
QUESTION 750
Four days after admission for cirrhosis of the liver, the nurse observes the following when
assessing a male client: increased irritability, asterixis, and changes in his speech pattern.
Which of the following foods would be appropriate for his bedtime snack?
A. Fresh fruit
B. A milkshake
C. Saltine crackers and peanut butter
D. A ham and cheese sandwich
Answer: A
Explanation:

(A) High levels of ammonia, a by-product of protein metabolism, can precipitate metabolic
encephalopathy. These clients need a diet high in carbohydrates and bulk.
(B) Metabolic encephalopathy of the brain associated with liver failure is precipitated by
elevated ammonia levels. Ammonia is a by-product of protein metabolism.
(C, D) Metabolic encephalopathy in liver failure is precipitated by elevated ammonia levels.
Ammonia is a by- product of protein metabolism.
QUESTION 751
Four days after admission for cirrhosis of the liver, the nurse observes the following when
assessing a male client: increased irritability, asterixis, and changes in his speech pattern.
Which of the following foods would be appropriate for his bedtime snack?
A. Fresh fruit
B. A milkshake
C. Saltine crackers and peanut butter
D. A ham and cheese sandwich
Answer: B
Explanation:
(A) Albumin, a blood volume expander, increases the circulating blood volume by exerting an
osmotic pull on tissue fluids, pulling them into the vascular system. This fluid shift causes an
increase in the heart rate and blood pressure.
(B) Albumin, a blood volume expander, exerts an osmotic pull on fluids in the interstitial
spaces, pulling the fluid back into the circulatory system. This fluid shift causes an increase in
the urinary output.
(C) Adventitious breath sounds and dyspnea can occur due to circulatory overload if the
albumin is infused too rapidly.
(D) Chills, fever, itching, and rashes are signs of a hypersensitivity reaction to albumin.
QUESTION 752
Which of the following physician’s orders would the nurse question on a client with chronic
arterial insufficiency?
A. Neurovascular checks every 2 hours
B. Elevate legs on pillows
C. Arteriogram in the morning
D. No smoking

Answer: B
Explanation:
(A) Neurovascular checks are a routine part of assessment with clients having this diagnosis.
(B) Elevation of the legs is contraindicated because it reduces blood flow to areas already
compromised.
(C) Arteriogram is a routine diagnostic order.
(D) Smoking is highly correlated with this disorder.
QUESTION 753
In planning daily care for a client with multiple sclerosis, the nurse would take into
consideration that multiple sclerosis:
A. Becomes progressively debilitating without remission
B. Has unpredictable remissions and exacerbations
C. Is rapidly fatal
D. Responds quickly to antimicrobial therapy
Answer: B
Explanation:
(A) Multiple sclerosis eventually becomes debilitating, but it is characterized by remission of
symptoms.
(B) Remissions and exacerbations are unpredictable with multiple sclerosis. The client
experiences progressive dysfunction after each exacerbation episode.
(C) Multiple sclerosis is usually slowly progressive.
(D) Multiple sclerosis is an autoimmune disease. Antimicrobial therapy has no effect on its
course.
QUESTION 754
The nurse assesses a postoperative mastectomy client and notes that breath sounds are
diminished in both posterior bases. The nurse’s action should be to:
A. Encourage coughing and deep breathing each hour
B. Obtain arterial blood gases
C. Increase O2 from 2–3 L/min
D. Remove the postoperative dressing to check for bleeding
Answer: A
Explanation:

(A) Decreased or absent breath sounds are frequently indicators of postoperative atelectasis.
(B) Arterial blood gases are not indicated because there is no other information indicating
impending danger.
(C) Increasing O2 rate is not indicated without additional information.
(D) Removing the dressing is not indicated without additional information.
QUESTION 755
A 6-year-old child returned to the surgical floor 20 hours ago after an appendectomy for a
gangrenous appendix. His mother tells the nurse that he is becoming more restless and is
anxious. Assessment findings indicate that the child has atelectasis. Appropriate nursing
actions would include:
A. Allowing the child to remain in the position of comfort, preferably semi-or high-Fowler
position
B. Administering analgesics as ordered
C. Having the child turn, cough, and deep breathe every 1–2 hours
D. Remaining with the child and keeping as calm and quiet as possible
Answer: C
Explanation:
(A) Allowing the client to remain in the position of comfort will not resolve the atelectasis.
This position, if left unchanged, over time may actually increase the atelectasis.
(B) Analgesics will not resolve the atelectasis and may contribute to it if proper nursing
actions are not taken to help resolve the atelectasis.
(C) Having the client turn, cough, and deep breathe every 1–2 hours will aid in resolving the
atelectasis. Surgery clients are at risk for postoperative respiratory complications because
pulmonary function is reduced as a result of anesthesia and surgery.
(D) Remaining with the client and keeping him calm and quiet will not affect the client’s
anxiety, restlessness, or help to resolve the atelectasis. The cause (atelectasis) needs to be
treated, not the symptoms (anxiety and restlessness).
QUESTION 756
Prior to his discharge from the hospital, a cardiac client is started on digoxin (Lanoxin) 25 mg
po qd. The nurse initiates discharge teaching. Which of the following statements by the client
would validate an understanding of his medication?

A. “I would notify my physician immediately if I experience nausea, vomiting, and double
vision.”
B. “I could stop taking this medication when I begin to feel better.”
C. “I should only take the medication if my heart rate is greater than 100 bpm.”
D. “I should always take this medication with an antacid.”
Answer: A
Explanation:
(A) The first signs of digoxin toxicity include abdominal pain, anorexia, nausea, vomiting,
and visual disturbances. The physician should be notified if any of these symptoms are
experienced.
(B) The positive inotropic effects of digoxin increase cardiac output and result in an enhanced
activity tolerance. “Feeling better indicates the drug is working and medication therapy must
be continued.”
(C) Clients should be taught to take their pulse prior to taking the digoxin. If their pulse rate
becomes irregular, slows significantly, or is 100 bpm the physician should be notified.
(D) Antacids decrease the effectiveness of digoxin.
QUESTION 757
Cystic fibrosis is transmitted as an autosomal recessive trait. This means that:
A. Mothers carry the gene and pass it to their sons
B. Fathers carry the gene and pass it to their daughters
C. Both parents must have the disease for a child to have the disease
D. Both parents must be carriers for a child to have the disease
Answer: D
Explanation:
(A) Cystic fibrosis is not an X-linked or sex-linked disease.
(B) The only characteristic on the Y chromosome is the trait for hairy ears.
(C) Both parents do not need to have the disease but must be carriers.
(D) If a trait is recessive, two genes (one from each parent) are necessary to produce an
affected child.
QUESTION 758
On the third postpartum day, the nurse would expect the lochia to be:
A. Rubra

B. Serosa
C. Alba
D. Scant
Answer: A
Explanation:
(A) This discharge occurs from delivery through the 3rd day. There is dark red blood,
placental debris, and clots.
(B) This discharge occurs from days 4–10. The lochia is brownish, serous, and thin.
(C) This discharge occurs from day 10 through the 6thweek. The lochia is yellowish white.
(D) This is not a classification of lochia but relates to the amount of discharge.
QUESTION 759
The mother of a preschooler reports to the nurse that he frequently tells lies. The admission
assessment of the child indicates possible child abuse. The nurse knows that his:
A. Behavior is not normal, and a child psychiatrist should be consulted.
B. Mother is lying to protect herself.
C. Lying is normal behavior for a preschool child who is learning to separate fantasy from
reality.
D. Behavior indicates a developmental delay, because preschoolers should be able to tell right
from wrong.
Answer: C
Explanation:
(A) Because preschoolers often tell “stories as they learn to differentiate fantasy from reality,
the child’s behavior is normal.”
(B) The nurse has no reason to believe the child’s mother is lying, because children of his age
often tell lies.
(C) The child’s lying is actually “storytelling as he learns to separate fantasy from reality, a
normal developmental task for his age group.”
(D) The child’s behavior is consistent with his age and does not indicate a developmental
delay.
QUESTION 760
The initial focus when providing nursing care for a child with rheumatic fever during the
acute phase of the illness should be to:

A. Maintain contact with her parents
B. Provide for physical and psychological rest
C. Provide a nutritious diet
D. Maintain her interest in school
Answer: B
Explanation:
(A) This goal is helpful, but rest is essential during the acute phase.
(B) Rest is essential for healing to occur and for pain to be relieved.
(C) This goal is important, but rest is essential.
(D) This goal should be part of the plan of care, but it is not the priority during the acute
phase.
QUESTION 761
A 30-year-old client has been admitted to the psychiatric service with the diagnosis of
schizophrenia. He tells the nurse that when the woman he had been dating broke up with him,
the CIA had replaced her with an identical twin. The client is experiencing:
A. Grandiose delusions
B. Paranoid delusions
C. Auditory hallucinations
D. Visual hallucinations
Answer: B
Explanation:
(A) There are no indications that the client’s thoughts reflect special powers or talents
characteristic of grandiosity.
(B) The client’s thought content is fixed, false, persecutory, and suspicious in nature, which is
characteristic of paranoid delusions.
(C, D) The client is not demonstrating a sensory experience.
QUESTION 762
The mother of a client is apprehensive about taking home her 2 year old who was diagnosed
with asthma after being admitted to the emergency room with difficulty breathing and
cyanosis. She asks the nurse what symptoms she should look for so that this problem will not
happen again. The nurse instructs her to watch for the following early symptoms:
A. Fever, runny nose, and hyperactivity

B. Changes in breathing pattern, moodiness, fatigue, and edema of eyes
C. Fatigue, dark circles under the eyes, changes in breathing pattern, glassy eyes, and
moodiness
D. Fever, cough, paleness, and wheezing
Answer: C
Explanation:
(A) The child with asthma may not have fever unless there is an underlying infection.
(B) Edema of the eyes will not be present because the child with asthma is more likely to
have dehydration related to excessive water loss during the work of breathing.
(C) All of these symptoms indicate decreased oxygenation and are early symptoms of asthma.
(D) Coughing and wheezing are not early signs of difficulty.
QUESTION 763
During the active phase of rheumatic fever, the nurse teaches parents of a child with acute
rheumatic fever to assist in minimizing joint pain and promoting healing by:
A. Putting all joints through full range-of-motion twice daily
B. Massaging the joints briskly with lotion or liniment after bath
C. Immobilizing the joints in functional position using splints, rolls, and pillows
D. Applying warm water bottle or heating pads over involved joints
Answer: C
Explanation:
(A) Any movement of the joint causes severe pain.
(B) Touching or moving the joint causes severe pain.
(C) Immobilization in a functional position allows the joint to rest and heal.
(D) Pressure from the warm water bottle or pads can cause severe pain or burning of the skin.
QUESTION 764
To prevent transmission of bacterial meningitis, the nurse would instruct an infected baby’s
mother to:
A. Avoid touching the baby while in the room.
B. Stay outside of the baby’s room.
C. Wear a gown and gloves and wash her hands before and after leaving the room.
D. Wear a mask while in the room.
Answer: C

Explanation:
(A) The mother should be allowed and encouraged to touch her baby.
(B) With care, transmission can be prevented. There is no need for the mother to stay outside
the room.
(C) Everyone entering the baby’s room should take appropriate measures to prevent
transmission of pathogens.
(D) Wearing a mask will not protect against transmission of pathogens.
QUESTION 765
A female client is started on warfarin (Coumadin) 5 mg po bid. To adequately evaluate the
effectiveness of the warfarin therapy, the nurse must know that this medication:
A. Dissolves any clots already formed in the arteries
B. Prevents the conversion of prothrombin to thrombin
C. Interferes with the synthesis of vitamin K-dependent clotting factors
D. Stimulates the manufacturing of platelets
Answer: C
Explanation:
(A) Thrombolytic agents (e.g., streptokinase) directly activate plasminogen, dissolving fibrin
deposits, which in turn dissolves clots that have already formed.
(B) Heparin prevents the formation of clots by potentiating the effects of antithrombin III and
the conversion of prothrombin to thrombin.
(C) Warfarin prevents the formation of clots by interfering with the hepatic synthesis of the
vitamin K-dependent clotting factors.
(D) Platelets initiate the coagulation of blood by adhering to each other and the site of injury
to form platelet plugs.
QUESTION 766
A psychiatric client has been stabilized and is to be discharged. The nurse will recognize
client insight and behavioral change by which of the following client statements?
A. “When I get home, I will need to take my medicines and call my therapist if I have any
side effects or begin to hear voices.”
B. “If I have any side effects from my medicines, I will take an extra dose of Cogentin.”
C. “When I get home, I should be able to taper myself off the Haldol because the voices are
gone now.”

D. “As soon as I leave here, I’m throwing away my medicines. I never thought I needed them
anyway.”
Answer: A
Explanation:
(A) The client verbalizes that he is responsible for compliance and keeping the treatment
team member informed of progress. This behavior puts him at the lowest risk for relapse.
(B) Noncompliance is a major cause of relapse. This statement reflects lack of responsibility
for his own health maintenance.
(C) This statement reflects lack of insight into the importance of compliance.
(D) This statement reflects no insight into his illness or his responsibility in health
maintenance.
QUESTION 767
A 44-year-old client had an emergency cholecystectomy 3 days ago for a ruptured
gallbladder. She complains of severe abdominal pain. Assessment reveals abdominal rigidity
and distention, increased temperature, and tachycardia. Diagnostic testing reveals an elevated
WBC count. The nurse suspects that the client has developed:
A. Gastritis
B. Evisceration
C. Peritonitis
D. Pulmonary embolism
Answer: C
Explanation:
(A) Assessment findings for gastritis would reveal anorexia, nausea and vomiting, epigastric
fullness and tenderness, and discomfort.
(B) Evisceration is the extrusion of abdominal viscera as a result of trauma or sutures failing
in a surgical incision.
(C) Peritonitis, inflammation of the peritoneum, can occur when an abdominal organ, such as
the gallbladder, perforates and leaks blood and fluid into the abdominal cavity. This causes
infection and irritation.
(D) Assessment findings of pulmonary embolism would reveal severe substernal chest pain,
tachycardia, tachypnea, shortness of breath, anxiety or panic, and wheezing and coughing
often accompanied by blood-tinged sputum.

QUESTION 768
A client is to be discharged from the hospital and is to continue taking warfarin 2.5 mg po
bid. Which of the following should be included in her discharge teaching concerning the
warfarin therapy?
A. “If you forget to take your morning dose, double the night time dose.”
B. “You should take aspirin instead of acetaminophen (Tylenol) for headaches.”
C. “Carry a medications alert card with you at all times.”
D. “You should use a straight-edge razor when shaving your arms and legs.”
Answer: C
Explanation:
(A) Warfarin must always be taken exactly as directed. Clients should be instructed never to
skip or double up on their dosage.
(B) Aspirin decreases platelet aggregation, which would potentiate the effects of the
coumadin.
(C) Healthcare providers need to be aware of persons on warfarin therapy prior to the
initiation of any diagnostic tests and/or surgery to help prevent bleeding complications.
(D) An electric razor should be used to prevent accidental cutting, which can lead to bleeding.
QUESTION 769
A client tells the nurse that he has been hearing voices that tell him to kill his girlfriend
because she is a spy. He further states that he is having difficulty not obeying the voices
because, if he does not, his house will be burned down. The highest priority nursing diagnosis
for him at this time is:
A. Sensory-perceptual alteration: auditory command hallucinations
B. Alteration in thought processes: paranoid delusions
C. Potential for violence directed at others
D. Impaired verbal communication: loose associations
Answer: C
Explanation:
(A) Although the client is having command hallucinations, this is second in priority to real or
potential violence, which can be a threat to life itself.
(B) Although the client is experiencing delusions, this is also a lower priority than his
potential or actual loss of control.
(C) Whether real or potential, violence directed at self or others is always high priority.

(D) There is no evidence of loosening of associations.
QUESTION 770
The physician orders medication for a client’s unpleasant side effects from the haloperidol.
The most appropriate drug at this time is:
A. Lorazepam
B. Triazolam (Halcion)
C. Benztropine
D. Thiothixene
Answer: C
Explanation:
(A) Lorazepam is a benzodiazepine, or antianxiety agent, that potentiates the effects of
aminobutyric acid in the CNS, which is not the CNS neurotransmitter EPS.
(B) Triazolam is a benzodiazepine sedative-hypnotic whose action is mediated in the limbic,
thalamic, and hypothalamic levels of the CNS by - aminobutyric acid.
(C) Benztropine is an anticholinergic agent, and the drug of choice for blocking CNS synaptic
response, which causes EPS.
(D) Thiothixene is an antipsychotic and neuroleptic drug that blocks dopamine
neurotransmission at the CNS synapses, thereby causing EPS.
QUESTION 771
The nurse is admitting a client with folic acid deficiency anemia. Which of the following
questions is most important for the nurse to ask the client?
A. “Do you take aspirin on a regular basis?”
B. “Do you drink alcohol on a regular basis?”
C. “Do you eat red meat?”
D. “Have your stools been normal?”
Answer: B
Explanation:
(A) Aspirin does not affect folic acid absorption.
(B) Folic acid deficiency is strongly associated with alcohol abuse.
(C) Because folic acid is a coenzyme for single carbon transfer purines, calves liver or other
purines are the meat sources.
(D) Folic acid does not affect stool character.

QUESTION 772
Forty-eight hours after a thyroidectomy, a female client complains of numbness and tingling
of the toes and fingers. The nurse notes upper arm and facial twitching. The nurse needs to:
A. Report the findings to the physician
B. Assist the client to do range of motion exercises
C. Check the client’s potassium level
D. Administer the as-needed dose of phenytoin (Dilantin)
Answer: A
Explanation:
(A) Muscular hyperactivity and paraesthesia’s may indicate hypocalcemic tetany and require
immediate administration of calcium gluconate. Tetany can occur if the parathyroid glands
were erroneously excised during surgery.
(B) Range of motion exercises are not appropriate to presenting symptoms.
(C) These characteristics are not usual signs of potassium imbalance, but of calcium
imbalance.
(D) Phenytoin is indicated for seizure activity mainly of neurological origin.
QUESTION 773
Parents should be taught not to prop the bottle when feeding their infants. In addition to the
risk of choking, it puts the infant at risk for:
A. Otitis media
B. Asthma
C. Conjunctivitis
D. Tonsillitis
Answer: A
Explanation:
(A) Because the eustachian tube is short and straight in the infant, formula that pools in the
back of the throat attacks bacteria which can enter the middle ear and cause an infection.
(B) Asthma is not associated with propping the bottle.
(C) Conjunctivitis is an eye infection and not associated with propping the bottle.
(D) Tonsillitis is usually a result of pharyngitis and not propping the bottle.

QUESTION 774
A pregnant client is having a nonstress test (NST). It is noted that the fetal heart beat rises 20
bpm, lasting 20 seconds, every time the fetus moves. The nurse explains that:
A. The test is inconclusive and should be repeated
B. Further testing is needed
C. The test is normal and the fetus is reacting appropriately
D. The fetus is distressed
Answer: C
Explanation:
(A) The test results were normal, so there would be no need to repeat to determine results.
(B) There are no data to indicate further tests are needed, because the result of the NST was
normal.
(C) An NST is reported as reactive if there are two to three increases in the fetal heart rate of
15 bpm, lasting at least 15 seconds during a 15-minute period.
(D) The NST results were normal, so there was no fetal distress.
QUESTION 775
The nurse is teaching a client how to perform monthly testicular self-examination (TSE) and
states that it is best to perform the procedure right after showering. This statement is made by
the nurse based on the knowledge that:
A. The client is more likely to remember to perform the TSE when in the nude
B. When the scrotum is exposed to cool temperatures, the testicles become large and bulky
C. The scrotum will be softer and more relaxed after a warm shower, making the testicles
easier to palpate
D. The examination will be less painful at this time
Answer: C
Explanation:
(A) Nudity is not a trigger for reminding males to perform TSE.
(B) Testicles become more firm when exposed to cool temperatures, but not large and bulky.
(C) The testicles will be lower and more easily palpated with warmer temperatures. A
protective mechanism of the body to protect sperm production is for the scrotum to pull
closer to the body when exposed to cooler temperatures.
(D) The examination should not be painful.

QUESTION 776
When assessing a client, the nurse notes the typical skin rash seen with systemic lupus
erythematosus. Which of the following descriptions correctly describes this rash?
A. Small round or oval reddish brown macules scattered over the entire body
B. Scattered clusters of macules, papules, and vesicles over the body
C. Bright red appearance of the palmar surface of the hands
D. Reddened butterfly shaped rash over the cheeks and nose
Answer: D
Explanation:
(A) The appearance of small, round or oval reddish brown macules scattered over the entire
body is characteristic of rubeola.
(B) The appearance of scattered clusters of macules, papules, and vesicles throughout the
body is characteristic of chickenpox.
(C) Palmar redness is seen in clients with cirrhosis of the liver.
(D) The characteristic butterfly rash over the cheek and nose and into the scalp is seen with
systemic lupus erythematosus.
QUESTION 777
The nurse is interviewing a client with a diagnosis of possible abdominal aortic aneurysm.
Which of the following statements will be reflected in the client’s chief complaint?
A. “I’ve been having a dull pain at the upper left shoulder.”
B. “My legs have been numb for three months.”
C. “I’ve only been urinating three times a day lately.”
D. “I don’t remember anything in particular, I just haven’t felt well.”
Answer: D
Explanation:
(A, B, C) These complaints are not specific signs and symptoms associated with abdominal
aortic aneurysm. If symptoms are present, the aneurysm is expanding or rupture is imminent.
(D) Many clients may experience no symptoms. The only symptom may be a pulsation noted
in the abdomen in the reclining position.

QUESTION 778
Which of the following nursing care goals has the highest priority for a child with
epiglottitis?
A. Sleep or lie quietly 10 hr/day.
B. Consume foods from all four food groups.
C. Be afebrile throughout her hospital stay.
D. Participate in play activities 4 hr/day.
Answer: A
Explanation:
(A) Of these four goals, maintenance of a calm, quiet atmosphere to reduce anxiety and to
allow for rest is the most important.
(B) Although nutrition is important, the child needs fluids to maintain fluid and electrolyte
balance more than solid foods. In addition, the child may not be able to swallow solid foods
owing to epiglottic swelling.
(C) This goal is unrealistic because fever is a common symptom of the infection associated
with epiglottitis.
(D) If overexerted, the child will need more O2 and energy than available, and these
requirements may exacerbate the condition.
QUESTION 779
The nurse notes multiple bruises on the arms and legs of a newly admitted client with lupus.
The client states, “I get them whenever I bump into anything.” The nurse would expect to
note a decrease in which of the following laboratory tests?
A. Number of platelets
B. WBC count
C. Hemoglobin level
D. Number of lymphocytes
Answer: A
Explanation:
(A) Thrombocytopenia, a decrease in platelets, occurs in lupus and causes a decrease in blood
coagulation and thrombus formation.
(B) Clients with lupus will have a decrease in the WBC count decreasing their resistance to
infection.

(C) Clients with lupus may have a decrease in the hemoglobin level causing anemia.
(D) Leukopenia, a decrease in white blood cells, is seen in lupus and decreases resistance to
infection.
QUESTION 780
When preparing insulin for IV administration, the nurse identifies which kind of insulin to
use?
A. NPH
B. Human or pork
C. Regular
D. Long acting
Answer: C
Explanation:
(A, B, D) Intermediate-acting and long-acting preparations contain materials that increase
length of absorption time from the subcutaneous tissues but cause the preparation to be
cloudy and unsuitable for IV use. Human insulin must be given SC.
(C) Only regular insulin can be given IV.
QUESTION 781
A client is having episodes of hyperventilation related to her surgery that is scheduled
tomorrow. Appropriate nursing actions to help control hyperventilating include:
A. Administering diazepam (Valium) 10–15 mg po q4h and q1h prn for hyperventilating
episode
B. Keeping the temperature in the client’s room at a high level to reduce respiratory
stimulation
C. Having the client hold her breath or breathe into a paper bag when hyperventilation
episodes occur
D. Using distraction to help control the client’s hyperventilation episodes
Answer: C
Explanation:
(A) An adult diazepam dosage for treatment of anxiety is 2–10 mg PO 2–4 times daily. The
order as written would place a client at risk for overdose.
(B) A high room temperature could increase hyperventilating episodes by stimulating the
respiratory system.

(C) Breath holding and breathing into a paper bag may be useful in controlling
hyperventilation. Both measures increase CO2 retention.
(D) Distraction will not prevent or control hyperventilation caused by anxiety or fear.
QUESTION 782
Which of the following lab data is representative of a client with aplastic anemia?
A. Hemoglobin 9.2, hematocrit 27, red blood cells 3.2 million
B. White blood cells 4000, erythrocytes 2.5 million, thrombocytes 100,000
C. White blood cells 3000, hematocrit 27, red blood cells 2.8 million
D. Red blood cells 1 million, white blood cells 1500, thrombocytes 16,000
Answer: D
Explanation:
(A, B, C) Although all of the lab data are abnormal and although these values are decreased in
aplastic anemia, the disorder is defined by severe deficits in red cell, white cell, and platelet
counts.
(D) Aplastic anemia is typically defined in terms of abnormalities of red blood cell count,
usually 1 million, white cell count 2,000, and thrombocytes 20,000.
QUESTION 783
The physician orders haloperidol 5 mg IM stat for a client and tells the nurse that the dose can
be repeated in 1– 2 hours if needed. The most likely rationale for this order is:
A. The client will settle down more quickly if he thinks the staff is medicating him
B. The medication will sedate the client until the physician arrives
C. Haloperidol is a minor tranquilizer and will not over sedate the client
D. Rapid neuroleptization is the most effective approach to care for the violent or potentially
violent client
Answer: D
Explanation:
(A) If the client could think logically, he would not be paranoid. In fact, he is probably
suspicious of the staff, too. Newly admitted clients frequently experience high levels of
anxiety, which can contribute to delusions.
(B) The goal of pharmacological intervention is to calm the client and assist with realitybased thinking, not to sedate him.
(C) Haloperidol is a neuroleptic and antipsychotic drug, not a minor tranquilizer.

(D) Haloperidol is a high-potency neuroleptic and first-line choice for rapid neuroleptization,
with low potential for sedation.
QUESTION 784
During the assessment, the nurse observes a client scratching his skin. He has been admitted
to rule out Laennec’s cirrhosis of the liver. The nurse knows the pruritus is directly related to:
A. A loss of phagocytic activity
B. Faulty processing of bilirubin
C. Enhanced detoxification of drugs
D. The formation of collateral circulation
Answer: B
Explanation:
(A) A loss in the phagocytic activity of the Kupffer cells occurs with cirrhosis of the liver,
which increases the susceptibility to infections.
(B) The faulty processing of bilirubin produces bile salts, which are irritating to the skin.
(C) The detoxification of drugs is impaired with cirrhosis of the liver.
(D) Collateral circulation develops due to portal hypertension. This is manifest through the
development of esophageal varices, hemorrhoids, and caput medusae.
QUESTION 785
Nursing assessment of early evidence of septic shock in children at risk includes:
A. Fever, tachycardia, and tachypnea
B. Respiratory distress, cold skin, and pale extremities
C. Elevated blood pressure, hyperventilation, and thready pulses
D. Normal pulses, hypotension, and oliguria
Answer: A
Explanation:
(A) Fever, tachycardia, and tachypnea are the classic early signs of septic shock in children.
(B) Respiratory distress, cold skin, and pale extremities are later signs of septic shock.
(C) Elevated blood pressure, hyperventilation, and thready pulses are later signs of septic
shock.
(D) Normal pulses, hypotension, and oliguria are not early signs of septic shock.

QUESTION 786
When assessing a female child for Turner’s syndrome, the nurse observes for which of the
following symptoms?
A. Tall stature
B. Amenorrhea
C. Secondary sex characteristics
D. Gynecomastia
Answer: B
Explanation:
(A) This syndrome is caused by absence of one of the X chromosomes. These children are
short in stature.
(B) Amenorrhea is a symptom of Turner’s syndrome, which appears at puberty.
(C) Sexual infantilism is characteristic of this syndrome.
(D) Gynecomastia is a symptom in Klinefelter’s syndrome.
QUESTION 787
A 2-year-old toddler is hospitalized with epiglottitis. In assessing the toddler, the nurse would
expect to find:
A. A productive cough
B. Expiratory stridor
C. Drooling
D. Crackles in the lower lobes
Answer: C
Explanation:
(A) A productive cough is not associated with epiglottitis.
(B) Children with epiglottitis seldom have expiratory stridor. Inspiratory stridor is more
common due to edema of the supraglottic tissues.
(C) Because of difficulty with swallowing, drooling often accompanies epiglottitis.
(D) Crackles are not heard in the lower lobes with epiglottitis because the infection is usually
confined to the supraglottic structures.
QUESTION 788
A client is receiving IV morphine 2 days after colorectal surgery. Which of the following
observations indicate that he may be becoming drug dependent?

A. The client requests pain medicine every 4 hours.
B. He is asleep 30 minutes after receiving the IV morphine.
C. He asks for pain medication although his blood pressure and pulse rate are normal.
D. He is euphoric for about an hour after each injection.
Answer: D
Explanation:
(A) Frequent requests for pain medication do not necessarily indicate drug dependence after
complex surgeries such as colorectal surgery.
(B) Sleeping after receiving IV morphine is not an unexpected effect because the pain is
relieved.
(C) A person may be in pain even with normal vital signs.
(D) A subtle sign of drug dependency is the tendency for the person to appear more euphoric
than relieved of pain.
QUESTION 789
The nurse is teaching a child’s parents how to protect the child from lead poisoning. The
nurse knows that a common source of lead poisoning in children is:
A. Dandelion leaves
B. Pencils
C. Old paint
D. Stuffing from toy animals
Answer: C
Explanation:
(A) Dandelion leaves are not a source of lead.
(B) Pencils are not a source of lead poisoning.
(C) Chewing on objects painted before 1960 is a common source of lead poisoning in
children. Gasoline is another source.
(D) Stuffed animals are not a source of lead.
QUESTION 790
A family by court order undergoes treatment by a family therapist for child abuse. The nurse,
who is the child’s case manager knows that treatment has been effective when:
A. The child is removed from the home and placed in foster care
B. The child’s parents identify the ways in which he is different from the rest of the family

C. The child’s father is arrested for child abuse
D. The child’s parents can identify appropriate behaviors for children in his age group
Answer: D
Explanation:
(A) Removing an abused child from the home and placement in a foster home are not the
desired outcome of treatment.
(B) Children who are perceived as “different from the rest of the family are more likely to be
abused.”
(C) Although legal action may be taken against abusive parents, it is not an indicator of an
effective treatment program.
(D) Identification of age-appropriate behaviors is essential to the role of parents, because
misunderstanding children’s normal developmental needs often contributes to abuse or
neglect.
QUESTION 791
A 4 year old has an imaginary playmate, which concerns the mother. The nurse’s best
response would be:
A. “I understand your concern and will assist you with a referral.”
B. “Try not to worry because you will just upset your child.”
C. “Just ignore the behavior and it should disappear by age 8.”
D. “This is appropriate behavior for a preschooler and should not be a concern.”
Answer: D
Explanation:
(A) This is normal for a preschooler, and a referral is not appropriate.
(B) Telling a parent not to worry is unhelpful. This response does not address the mother’s
concern.
(C) This response is incorrect. The behavior is normal and will usually disappear by the time
the child enters school.
(D) This behavior is normal development for a preschooler.
QUESTION 792
A 9-week-old female infant has a diagnosis of bilateral cleft lip and cleft palate. She has been
admitted to the pediatric unit after surgical repair of the cleft lip. Which of the following
nursing interventions would be appropriate during the first 24 hours?

A. Position on side or abdomen.
B. Maintain elbow restraints in place unless she is being directly supervised.
C. Clean suture line every shift.
D. Offer pacifier when she cries.
Answer: B
Explanation:
(A) Placing the infant on her abdomen may allow for injury to the suture line.
(B) Elbow restraints prevent the infant from touching the suture line and yet leaves hands
free.
(C) The suture line is cleaned as often as every hour to prevent crusting and scarring.
(D) Sucking of a bottle or pacifier places pressure on the suture line and may delay healing
and cause scarring.
QUESTION 793
In discussing the plan of care for a child with chronic nephrosis with the mother, the nurse
identifies that the purpose of weighing the child is to:
A. Measure adequacy of nutritional management
B. Check the accuracy of the fluid intake record
C. Impress the child with the importance of eating well
D. Determine changes in the amount of edema
Answer: D
Explanation:
(A) Weighing a child with nephrosis is to assess for edema, not nutrition.
(B, C) This is not the purpose for weighing the child.
(D) Weight and measurement are the primary ways of evaluating edema and fluid shifts.
QUESTION 794
A client had a cardiac catheterization with angiography and thrombolytic therapy with
streptokinase. The nurse should initiate which of the following interventions immediately
after he returns to his room?
A. Place him on NPO restriction for 4 hours.
B. Monitor the catheterization site every 15 minutes.
C. Place him in a high Fowler position.
D. Ambulate him to the bathroom to void.

Answer: B
Explanation:
(A) A contrast dye, iodine, is used in this procedure. This dye is nephrotoxic. The client must
be encouraged to drink plenty of liquids to assist the kidneys in eliminating the dye.
(B) Streptokinase activates plasminogen, dissolving fibrin deposits. To prevent bleeding,
pressure is applied at the insertion site. The client is assessed for both internal and external
bleeding.
(C) The extremity used for the insertion site must be kept straight and be immobilized
because of the potential for bleeding.
(D) The client is kept on bed rest for 8–12 hours following the procedure because of the
potential for bleeding.
QUESTION 795
A child has a nursing diagnosis of fluid volume excess related to compromised regulatory
mechanisms. Which of the following nursing interventions is the most accurate measure to
include in his care?
A. Weigh the child twice daily on the same scale.
B. Monitor intake and output.
C. Check urine specific gravity of each voiding.
D. Observe for edema.
Answer: A
Explanation:
(A) Although all of these interventions are important aspects of care, weight is the most
sensitive indicator of fluid balance.
(B) Although monitoring intake and output is important, weight is a more accurate indicator
of fluid status.
(C) Urine specific gravity does not necessarily indicate fluid volume excess.
(D) Edema may not be apparent, yet the client may have fluid volume excess.
QUESTION 796
Loss of appetite for a child with leukemia is a major recurrent problem. The plan of care
should be designed to:
A. Reinforce attempts to eat
B. Help the child gain weight

C. Increase his appetite
D. Make mealtimes pleasant
Answer: A
Explanation:
(A) Ignoring refusals to eat and rewarding eating attempts are the most successful means of
increasing intake.
(B) This goal is not specific enough or related to the loss of appetite.
(C) This goal is not possible at this time based on his illness.
(D) This goal is helpful, but alone will not address his loss of appetite.
QUESTION 797
A client develops complications following a hysterectomy. Blood cultures reveal
Pseudomonas aeruginosa. The nurse expects that the physician would order an appropriate
antibiotic to treat P. aeruginosa such as:
A. Cefoperazone (Cefobid)
B. Clindamycin (Cleocin)
C. Dicloxacillin (Dycill)
D. Erythromycin (Erythrocin)
Answer: A
Explanation:
(A) Cefoperazone is indicated in the treatment of infection with Pseudomonas aeruginosa.
(B) Clindamycin is not indicated in the treatment of infection with P. aeruginosa.
(C) Dicloxacillin is not indicated in the treatment of infection with P. aeruginosa.
(D) Erythromycin is not indicated in the treatment of infection with P. aeruginosa.
QUESTION 798
A 45-year-old client diagnosed with major depression is scheduled for electroconvulsive
therapy (ECT) in the morning. Which of the following medications are routinely administered
either before or during ECT?
A. Thioridazine (Mellaril), lithium, and benztropine
B. Atropine, sodium brevitol, and succinylcholine chloride (Anectine)
C. Sodium, potassium, and magnesium
D. Carbamazepine (Tegretol), haloperidol, and trihexyphenidyl (Artane)
Answer: B

Explanation:
(A) Thioridazine (an antipsychotic drug), lithium (an antimanic drug), and benztropine (an
antiparkinsonism agent) are generally administered to treat schizophrenic and bipolar
disorders.
(B) Atropine (a cholinergic blocker), sodium brevitol (a shortacting anesthetic), and
succinylcholine (a neuromuscular blocker) are administered either before or during ECT to
count tract bradycardia and to provide anesthesia and total muscle relaxation.
(C) These are electrolyte substances administered to correct fluid and electrolyte imbalances
in the body.
(D) Carbamazepine (an anticonvulsant), haldoperidol (an antipsychotic), and trihexyphenydyl
(an antiparkinsonism agent) are usually administered in psychiatric settings to control
problems associated with psychotic behavior.
QUESTION 799
A 25-year-old client is admitted for a tonsillectomy. She tells the nurse that she has had
episodes of muscle cramps, weakness, and unexplained temperature elevation. Many years
ago her father died shortly after surgery after developing a high fever. She further tells the
nurse that her surgeon is having her take dantrolene sodium (Dantrium) prophylactically prior
to her tonsillectomy. Dantrolene sodium is ordered preoperatively to reduce the risk or
prevent:
A. Infection postoperatively
B. Malignant hyperthermia
C. Neuroleptic malignant syndrome
D. Fever postoperatively
Answer: B
Explanation:
(A, D) Dantrolene sodium is a peripheral skeletal muscle relaxant and would have no effect
on a postoperative infection.
(B) Dantrolene sodium is indicated prophylactically for clients with malignant hyperthermia
or with a family history of the disorder. The mortality rate for malignant hyperthermia is high.
(C) Neuroleptic malignant syndrome is an exercise-induced muscle pain and spasm and is
unrelated to malignant hyperthermia.

QUESTION 800
A 3-year-old child is in the burn unit following a home accident. The first sign of sepsis in
burned children is:
A. Disorientation
B. Low-grade fever
C. Diarrhea
D. Hypertension
Answer: A
Explanation:
(A) Disorientation is the first sign of sepsis in burn children.
(B) Low-grade fever is not indicative of sepsis.
(C) Diarrhea is not indicative of sepsis.
(D) Hypertension is not indicative of sepsis.
QUESTION 801
The parents of a 9-year-old child with acute lymphocytic leukemia expressed concern about
his alopecia from cranial irradiation. The nurse explains that:
A. Alopecia is an unavoidable side effect.
B. There are several wig makers for children.
C. Most children select a favorite hat to protect their heads.
D. His hair will grow back in a few months.
Answer: D
Explanation:
(A) Alopecia has occurred, and knowing it is a side effect does not address their concern.
(B) Although true, it does not give them hope for the future.
(C) Although true, it does not provide them with information of the temporary nature of the
situation.
(D) Knowing the hair will grow back provides comfort that the alopecia is temporary.
QUESTION 802
The family member of a child scheduled for heart surgery states, “I just don’t understand this
open-heart or closed-heart business.” I’m so confused! Can you help me understand it? The
nurse explains that patent ductus arteriosus repair is:

A. Open-heart surgery. The child will be placed on a heart-lung machine while the surgery is
being performed.
B. Closed-heart surgery. It does not require that the child be placed on the heart-lung machine
while the surgery is being performed.
C. A pediatric version of the coronary artery bypass graft surgery performed on adults. It is an
open-heart surgery.
D. A pediatric version of percutaneous transluminal coronary angioplasty performed on
adults. It is a closed- heart surgery.
Answer: B
Explanation:
(A) Patent ductus arteriosus repair is a closed-heart procedure. The client is not placed on a
heart-lung machine.
(B) Patent ductus arteriosus is a ductus arteriosus that does not close shortly after birth but
remains patent. Repair is a closed-heart procedure involving ligation of the patent ductus
arteriosus.
(C) Coronary artery bypass graft surgery is an open-heart surgical procedure in which
blocked coronary arteries are bypassed using vessel grafts.
(D) Percutaneous transluminal coronary angioplasty is a closed heart procedure that improves
coronary blood flow by increasing the lumen size of narrowed vessels.
QUESTION 803
A child with celiac disease is being discharged from the hospital. The mother demonstrates
knowledge of nutritional needs of her child when she is able to state the foods which are
included in a:
A. Lactose-restricted diet
B. Gluten-restricted diet
C. Phenylalanine-restricted diet
D. Fat-restricted diet
Answer: B
Explanation:
(A) A lactose-restricted diet is prescribed for children with lactose intolerance or diarrhea.
(B) A gluten-restricted diet is the diet for children with celiac disease.
(C) A phenylalanine restricted diet is prescribed for children with phenylketonuria.

(D) A fat-restricted diet is prescribed for children with disorders of the liver, gallbladder, or
pancreas.
QUESTION 804
The pediatric nurse charts that the parents of a 4-yearold child are very anxious. Which
observation would indicate to the nurse unhealthy coping by these parents:
A. Discussing their needs with the nursing staff
B. Discussing their needs with other family members
C. Seeking support from their minister
D. Refusing to participate in the child’s care
Answer: D
Explanation:
(A, B, C) These methods are healthy ways of dealing with anxiety.
(D) Participation minimizes feelings of helplessness and powerlessness. It is important that
parents have accurate information and that they seek support from sources available to them.
QUESTION 805
A client with a head injury asks why he cannot have something for his headache. The nurse’s
response is based on the understanding that analgesics could:
A. Counteract the effects of antibiotics
B. Elevate the blood pressure
C. Mask symptoms of increasing intracranial pressure
D. Stimulate the central nervous system
Answer: C
Explanation:
(A) Analgesic medication does not counteract the effects of antibiotics.
(B) Analgesic medication may lower blood pressure elevated due to anxiety.
(C) Analgesic medication, especially CNS depressants, is not given if there is danger of
increasing ICP, because neurological changes may not be apparent. Also, further depression
of the CNS is contraindicated.
(D) Analgesics do not stimulate the CNS.

QUESTION 806
Which of the following findings would necessitate discontinuing an IV potassium infusion in
an adult with ketoacidosis?
A. Urine output 22 mL/hr for 2 hours
B. Serum potassium level of 3.7
C. Small T wave of ECG
D. Serum glucose level of 180
Answer: A
Explanation:
(A) Adequate renal flow of 30 mL/hr is a necessity with potassium infusions because
potassium is excreted renally.
(B) Because potassium level will decrease during correction of diabetic ketoacidosis,
potassium will be infused even if plasma levels of potassium are normal.
(C) A small T wave is normal and desired on the electrocardiogram. A tall, peaked T-wave
could indicate over infusion of potassium and hyperkalemia.
(D) Glucose levels of 200 are desirable.
QUESTION 807
A male client is considering having laser abdominal surgery and asks the nurse if there is any
advantage in having this type of surgery? The nurse will respond based on the knowledge that
laser surgery:
A. Has a smaller postoperative infection rate than routine surgery
B. Will eliminate the need for preoperative sedation
C. Will result in less operating time
D. Generally eliminates problems with complications
Answer: A
Explanation:
(A) A lower postoperative infection has been documented as a result of laser therapy versus
routine surgery.
(B) Clients will still need preoperative sedation to facilitate anxiety reduction.
(C) Operating time may actually increase in some laser surgeries.
(D) The client must still be observed for postoperative complications.

QUESTION 808
The physician decides to prescribe both a short-acting insulin and an intermediate-acting
insulin for a newly diagnosed 8-year-old diabetic client. An example of a short-acting insulin
is:
A. Novolin Regular
B. Humulin NPH
C. Lente Beef
D. Protamine zinc insulin
Answer: A
Explanation:
(A) Novolin is a short-acting insulin.
(B, C) NPH and Lente are intermediate-acting insulins.
(D) Protamine zinc insulin is a long-acting insulin preparation.
QUESTION 809
A male client is scheduled to have angiography of his left leg. The nurse needs to include
which of the following when preparing the client for this procedure?
A. Validate that he is not allergic to iodine or shellfish.
B. Instruct him to start active range of motion of his left leg immediately following the
procedure.
C. Inform him that he will not be able to eat or drink anything for 4 hours after the procedure.
D. Inform him that vital signs will be taken every hour for 4 hours after the procedure.
Answer: A
Explanation:
(A) Angiography, an invasive radiographic examination, involves the injection of a contrast
solution (iodine) through a catheter that has been inserted into an artery.
(B) The client is kept on complete bed rest for 6–12 hours after the procedure. The extremity
in which the catheter was inserted must be immobilized and kept straight during this time.
(C) The contrast dye, iodine, is nephrotoxic. The client must be instructed to drink a large
quantity of fluids to assist the kidneys in excreting this contrast media.
(D) The major complication of this procedure is hemorrhage. Vital signs are assessed every
15 minutes initially for signs of bleeding.

QUESTION 810
The nurse will be alert to the most potentially life-threatening side effect associated with the
administration of monoamine oxidase (MAO) inhibitor. This is:
A. Oculogyric crisis
B. Hypertensive crisis
C. Orthostatic hypotension
D. Tardive dyskinesia
Answer: B
Explanation:
(A) Oculogyric crisis, involuntary upward deviation and fixation of the eyeballs, is usually
associated with either postencephalitic parkinsonian or drug-induced extrapyramidal
symptoms (EPS).
(B) Hypertensive crisis is a potentially life-threatening side effect. This may occur if the
client ingests foods, beverages, or medications containing tyramine.
(C) Orthostatic hypotension, a drop in blood pressure resulting from a rapid change of body
position, can occur with the administration of antidepressants.
(D) Tardive dyskinesia, characterized by slow, rhythmical, automatic or stereotyped muscular
movements, usually is associated with the administration of certain antipsychotic
medications.
QUESTION 811
A client reports to the nurse that the voices are practically nonstop and that he needs to leave
the hospital immediately to find his girlfriend and kill her. The best verbal response to the
client by the nurse at this time is:
A. “I understand that the voices are real to you, but I want you to know I don’t hear them.
They are a symptom of your illness.”
B. “Just don’t pay attention to the voices. They’ll go away after some medication.”
C. “You can’t leave here. This unit is locked and the doctor has not ordered your discharge.”
D. “We will have to put you in seclusion and restraints for a while. You could hurt someone
with thoughts like that.”
Answer: A
Explanation:
(A) This response validates the client’s experience and presents reality to him.
(B) This nontherapeutic response minimizes and dismisses the client’s verbalized experience.

(C) This response can be interpreted by a paranoid client as a threat, thereby increasing the
client’s potential for violence and loss of control.
(D) This response is also threatening. The client’s behavior does not call for restraints because
he has not lost control or hurt anyone. If seclusion or restraints were indicated, the nurse
should never confront the client alone.
QUESTION 812
Discharge teaching for the client who has a total gastrectomy should include which of the
following?
A. Need for the client to increase fluid intake to 3000 mL/day
B. Follow-up visits every 3 weeks for the first 6 months
C. B12 injections needed for the rest of the client’s life
D. Need to eat three full meals with plenty of fiber per day
Answer: C
Explanation:
(A) There will be no need to increase fluid intake excessively, because dumping syndrome
could present a problem.
(B) Follow-up visits every 3 weeks are not a standard recommendation. Follow-up visits will
be highly individualized.
(C) With removal of the stomach, intrinsic factor will no longer be produced. Intrinsic factor
is necessary for vitamin B12 absorption. Parenteral injections of B12 will be needed on a
monthly basis for the rest of the person’s life.
(D) Smaller, more frequent meals, rather than large, bulky meals, are recommended to
prevent problems with dumping syndrome.
QUESTION 813
The nurse is collecting a nutritional history on a 28- year-old female client with irondeficiency anemia and learns that the client likes to eat white chalk. When implementing a
teaching plan, the nurse should explain that this practice:
A. Will bind calcium and therefore interfere with its metabolism
B. Will cause more premenstrual cramping
C. Interferes with iron absorption because the iron precipitates as an insoluble substance
D. Causes competition at iron-receptor sites between iron and vitamin B1
Answer: C

Explanation:
(A) Eating chalk is not related to calcium and its absorption.
(B) Poor nutritional habits may result in increased discomfort during premenstrual days, but
this is not a primary reason for the client to stop eating chalk. Premenstrual discomfort has
not been mentioned.
(C) Iron is rendered insoluble and is excreted through the gastrointestinal tract.
(D) There is no competition between the two nutrients.
QUESTION 814
The nurse knows that children are more susceptible to respiratory tract infections owing to
physiological differences. These childhood differences, when compared to an adult, include:
A. Fewer alveoli, slower respiratory rate
B. Diaphragmatic breathing, larger volume of air
C. Larger number of alveoli, diaphragmatic breathing
D. Rounded shape of chest, smaller volume of air
Answer: D
Explanation:
(A) Although a child has fewer alveoli than an adult, the child’s respiratory rate is faster.
(B) Although a child may use diaphragmatic breathing, the adult exchanges a larger volume
of air.
(C) The adult has a larger number of alveoli than a child.
(D) The child’s chest is rounded whereas the adult chest is more of an oval shape, and the
child does exchange a smaller volume of air than an adult.
QUESTION 815
A client delivered a stillborn male at term. An appropriate action of the nurse would be to:
A. State, “You have an angel in heaven.”
B. Discourage the parents from seeing the baby.
C. Provide an opportunity for the parents to see and hold the baby for an undetermined
amount of time.
D. Reassure the parents that they can have other children.
Answer: C
Explanation:

(A) This is not a supportive statement. There are also no data to indicate the family’s religious
beliefs.
(B) Seeing their baby assists the parents in the grieving process. This gives them the
opportunity to say “good-bye.”
(C) Parents need time to get to know their baby.
(D) This is not a comforting statement when a baby has died. There are also no guarantees
that the couple will be able to have another child.
QUESTION 816
A client develops an intestinal obstruction postoperatively. A nasogastric tube is attached to
low, intermittent suction with orders to “Irrigate NG tube” with sterile saline q1h and prn. The
rationale for using sterile saline, as opposed to using sterile water to irrigate the NG tube is:
A. Water will deplete electrolytes resulting in metabolic acidosis.
B. Saline will reduce the risk of severe, colicky abdominal pain during NG irrigation.
C. Water is not isotonic and will increase restlessness and insomnia in the immediate
postoperative period.
D. Saline will increase peristalsis in the bowel.
Answer: A
Explanation:
(A) Water is a hypotonic solution and will deplete electrolytes and cause metabolic acidosis
when used for nasogastric irrigation.
(B) Irrigating with saline does not cause abdominal discomfort. Severe, colicky abdominal
pain is a symptom of intestinal obstruction.
(C) Irrigating with water will not cause restlessness or insomnia in the postoperative client.
Restlessness and insomnia can be emotional complications of surgery.
(D) A nasogastric tube placed in the stomach is used to decompress the bowel. Irrigating with
saline ensures a patent, well-functioning tube. Irrigating with saline will not increase
peristalsis.
QUESTION 817
A 29-year-old client is admitted for a hysterectomy. She has repeatedly told the nurses that
she is worried about having this surgery, has not slept well lately, and is afraid that her
husband will not find her desirable after the surgery. Shortly into the preoperative teaching,
she complains of a tightness in her chest, a feeling of suffocation, light-headedness, and

tingling in her hands. Her respirations are rapid and deep. Assessment reveals that the client
is:
A. Having a heart attack
B. Wanting attention from the nurses
C. Suffering from complete upper airway obstruction
D. Hyperventilating
Answer: D
Explanation:
(A) Classic symptoms of a heart attack include heaviness or squeezing pain in the chest, pain
spreading to the jaw, neck, and arm. Nausea and vomiting, sweating, and shortness of breath
may be present. The client does not exhibit these symptoms.
(B) Clients suffering from anxiety or fear prior to surgical procedures may develop
hyperventilation. This client is not seeking attention.
(C) Symptoms of complete airway obstruction include not being able to speak, and no airflow
between the nose and mouth. Breath sounds are absent.
(D) Tightness in the chest; a feeling of suffocation; light-headedness; tingling in the hands;
and rapid, deep respirations are signs and symptoms of hyperventilation. This is almost
always a manifestation of anxiety.
QUESTION 818
On assessment, the nurse learns that a chronic paranoid schizophrenic has been taking “the
blue pill” (haloperidol) in the morning and evening, and “the white pill” (benztropine) right
before bedtime. The nurse might suggest to the client that she try:
A. Doubling the daily dose of benztropine
B. Decreasing the haloperidol dosage for a few days
C. Taking the benztropine in the morning
D. Taking her medication with food or milk
Answer: C
Explanation:
(A) Suggesting that a client increase a medication dosage is an inappropriate (and illegal)
nursing action. This action requires a physician’s order.
(B) To suggest that a client decrease a medication dosage is an inappropriate
(and illegal) nursing action. This action requires a physician’s order.

(C) This response is an appropriate independent nursing action. Because motor restlessness
can also be a side effect of cogentin, the nurse may suggest that the client try taking the drug
early in the day rather than at bedtime.
(D) Certain medications can cause gastric irritation and may be taken with food or milk to
prevent this side effect.
QUESTION 819
The mother of a child taking phenytoin will need to plan appropriate mouth care and gingival
stimulation. When tooth-brushing is contraindicated, the next most effective cleansing and
gingival stimulation technique would be:
A. Using a water pik
B. Rinsing with water
C. Rinsing with hydrogen peroxide
D. Rinsing with baking soda
Answer: A
Explanation:
(A) This technique provides effective rinsing and gingival stimulation.
(B) This technique does not provide gingival stimulation.
(C) This technique provides effective rinsing but not gingival stimulation. Using peroxide is
not pleasant for the child.
(D) This technique provides effective rinsing but not gingival stimulation.
QUESTION 820
A client has been taking lithium 300 mg po bid for the past two weeks. This morning her
lithium level was 1 mEq/L. The nurse should:
A. Notify the physician immediately
B. Hold the morning lithium dose and continue to observe the client
C. Administer the morning lithium dose as scheduled
D. Obtain an order for benztropine (Cogentin)
Answer: C
Explanation:
(A) There is no need to phone the physician because the lithium level is within therapeutic
range and because there are no indications of toxicity present.

(B) There is no reason to withhold the lithium because the blood level is within therapeutic
range. Also, it is necessary to give the medication as scheduled to maintain adequate blood
levels.
(C) The lab results indicate that the client’s lithium level is within therapeutic range (0.2–1.4
mEq/L), so the medication should be given as ordered.
(D) Benztropine is an antiparkinsonism drug frequently given to counteract extrapyramidal
symptoms associated with the administration of antipsychotic drugs (not lithium).
QUESTION 821
A client admitted with a diagnosis of possible myocardial infarction is admitted to the unit
from the emergency room. The nurse’s first action when admitting the client will be to:
A. Obtain vital signs
B. Connect the client to the cardiac monitor
C. Ask the client if he is still having chest pain
D. Complete the history profile
Answer: B
Explanation:
(A) Obtaining vital signs is important after connecting the client to the monitor because vital
signs should be stable before the client is discharged from the emergency room.
(B) All are important, but the first priority is to monitor the client’s rhythm.
(C) If the client is in severe pain, pain medication should be given after connecting him to the
monitor and obtaining vital signs.
(D) Completion of the history profile is the least important of the nursing actions.
QUESTION 822
The nurse is developing a plan of care for a client with an electrolyte imbalance and identifies
a nursing diagnosis of decreased physical mobility. Which alteration is most likely the
etiology?
A. Hypernatremia
B. Hypocalcemia
C. Hypokalemia
D. Hypomagnesemia
Answer: C
Explanation:

(A) A deficit in sodium concentration results in muscular weakness and lethargy.
(B) Muscle fatigue and hypotonia are caused by hypercalcemia.
(C) Muscle weakness and fatigue are classic signs of hypokalemia.
(D) Hypermagnesemia can cause muscle weakness, paralysis, and coma.
QUESTION 823
The nurse discovers that a 78-year-old client who received hydralazine (Apresoline) 20 mg 45
minutes ago has a blood pressure of 70/40 mm Hg. The client has been on this dose of the
medication for 3 years. Which of the following data is most likely significant in relation to the
cause of the low blood pressure?
A. Pedal pulses 11 (weak)
B. Twenty-four-hour intake 1000 mL/day for past 2 days
C. Serum potassium 3.3
D. Pulse rate 150 bpm
Answer: B
Explanation:
(A, D) Decreased pulse volume and increased pulse rate are signs of an acute hypotensive
episode.
(B) Inadequate fluid volume when taking vasodilators can result in a drop in blood pressure
when vasodilation starts to physiologically occur as an action of the drug.
(C) A potassium level of 3.3 would not be associated with a significant drop in blood
pressure.
QUESTION 824
The nurse notes scattered crackles in both lungs and 1+ pitting edema when assessing a
cardiac client. The physician is notified and orders furosemide (Lasix) 80 mg IV push stat.
Which of the following diagnostic studies is monitored to assess for a major complication of
this therapy?
A. Serum electrolytes
B. Arterial blood gases
C. Complete blood count
D. 12-Lead ECG
Answer: A
Explanation:

(A) Furosemide, a potassium-depleting diuretic, inhibits the reabsorption of sodium and
chloride from the loop of Henle and the distal renal tubules. Serum electrolytes are monitored
for hypokalemia.
(B) Severe acid-base imbalances influence the movement of potassium into and out of the
cells, but arterial blood gases to not measure the serum potassium level.
(C) Furosemide is a potassium-depleting diuretic. A complete blood count does not reflect
potassium levels.
(D) Abnormalities in potassium (both hyperkalemia and hypokalemia) are reflected in ECG
changes, but these changes do not occur until the abnormality is severe.
QUESTION 825
A client is admitted to the hospital with a diagnosis of aplastic anemia and placed on
isolation. The nurse notices a family member entering the room without applying the
appropriate apparel. The nurse will approach the family member using the following
information as a basis for discussion:
A. The risks of exposure of the visitor to infectious organisms is great.
B. Hospital regulations mandate that everyone in the facility adhere to appropriate codes.
C. The client is at extreme risk of acquiring infections.
D. Adherence to the guidelines are the latest Centers for Disease Control and Prevention
recommendations on use of protective apparel.
Answer: C
Explanation:
(A) Although clients with a compromised immune system may acquire infections, the
primary emphasis is on protecting the client.
(B, D) Most people are aware of the guidelines once they see posted signs, so quoting
regulations is not likely to result in consistent adherence to regulations.
(C) Clients with aplastic anemia have white cell counts of 2000 or lower, making them more
vulnerable to infections from others.
QUESTION 826
The nurse enters the room of a client on which a “do not resuscitate order” has been written
and discovers that she is not breathing. Once the husband realizes what has occurred he yells,
“please save her!” The nurse’s action would be:
A. Call the physician and inform him that the client has expired.

B. Remind the husband that the physician wrote an order not to resuscitate.
C. Discuss with the husband that these orders are written only on clients who are not likely to
recover with resuscitative efforts.
D. Call a code and proceed with cardiopulmonary resuscitation.
Answer: D
Explanation:
(A, B, C) The last request from the husband overrides the decision not to initiate resuscitation
efforts.
(D) The nurse should begin cardiopulmonary resuscitation unless a living will and durable
power of attorney are in force. In the meantime, the nurse should talk with the husband and
notify the doctor.
QUESTION 827
Goal setting for a client with Meniere’s disease should include which of the following?
A. Frequent ambulation
B. Prevention of a fall injury
C. Consumption of three meals per day
D. Prevention of infection
Answer: B
Explanation:
(A) Although not contraindicated, initially ambulation may be difficult because of vertigo and
is recommended only with assistance.
(B) Vertigo resulting in balance problems is one of the most common manifestations of
Meniere’s disease.
(C) Adequate nutrition is important, but the emphasis in Meniere’s disease is not the number
of meals per day but a decrease in intake of sodium.
(D) Infection is not an anticipated problem.
QUESTION 828
A 40-year-old client has been admitted to the hospital with severe substernal chest pain
radiating down his left arm. The nurse caring for the client establishes the following priority
nursing diagnosis—Alteration in comfort, pain related to:
A. Increased excretion of lactic acid due to myocardial hypoxia
B. Increased blood flow through the coronary arteries

C. Decreased stimulation of the sympathetic nervous system
D. Decreased secretion of catecholamines secondary to anxiety
Answer: A
Explanation:
(A) Anaerobic metabolism results because the decreased blood supply to the myocardium
causes a release of lactic acid. Lactic acid is an irritant to the myocardial neural receptors,
producing chest pain.
(B) Chest pain is caused by a decrease in the O2 supply to the myocardial cells. Treatment
modalities for chest pain are aimed toward increasing the blood flow through coronary
arteries.
(C) Chest pain causes an increase in the stimulation of the sympathetic nervous system. This
stimulation increases the heart rate and blood pressure, causing an increase in myocardial
workload aggravating the chest pain.
(D) Chest pain and anxiety cause increased secretion of catecholamines by stimulating the
sympathetic nervous system. This stimulation increases chest pain by increasing the workload
of the heart.
QUESTION 829
A 20-year-old male client is being treated for protein deficiency. If he likes all of the
following foods, which one would the nurse recommend to increase in the diet?
A. Cantaloupe
B. Rice
C. Chicken
D. Green beans
Answer: C
Explanation:
(A) Cantaloupe is a good source of carbohydrates, vitamin C, and vitamin A.
(B) Rice contains about 4 g of protein per 200 g.
(C) Chicken contains 35 g protein per breast. Chicken is a rich source of vitamin B6
(pyridoxine), which is needed for adequate protein synthesis. As protein intake increases,
vitamin B6 intake must also be increased. Vitamin B6 is a coenzyme in amino acid
metabolism.
(D) Green beans only contain 2 g of protein per cup.

QUESTION 830
A client is started on prednisone 2.5 mg po bid. Which of the following instructions should be
included in her discharge teaching specific to this medication?
A. Increase your oral intake of fluids to at least 4000 mL every day.
B. Avoid contact with people who have contagious illnesses.
C. Brush your teeth at least 4 times a day with a firm toothbrush.
D. Immediately stop taking the prednisone if you feel depressed.
Answer: B
Explanation:
(A) Fluid retention is a side effect of prednisone. The nurse should teach clients to weigh
themselves daily and to observe for signs of edema. If these signs of fluid retention occur,
they should notify the physician.
(B) Prednisone, a glucocorticoid, suppresses the normal immune response making the client
more susceptible to infections.
(C) An increase in bleeding tendencies is a side effect of prednisone therapy. The nurse
should teach clients to use preventive measures (i.e., electric razors and soft toothbrushes).
(D) Depression and personality changes are side effects of prednisone therapy. Prednisone
should never be discontinued abruptly.
QUESTION 831
During discharge planning, parents of a child with rheumatic fever should be able to identify
which of the following as toxic symptoms of sodium salicylate?
A. Tinnitus and nausea
B. Dermatitis and blurred vision
C. Unconsciousness and acetone odor of the breath
D. Chills and an elevation of temperature
Answer: A
Explanation:
(A) These are toxic symptoms of sodium salicylate.
(B, C, D) These are not symptoms associated with sodium salicylate.
QUESTION 832
A 33-year-old client is diagnosed with bipolar disorder, acute phase. This is her first
psychiatric hospitalization, and she is being evaluated for treatment with lithium. Which of

the following diagnostic tests are essential prior to the initiation of lithium therapy with this
client?
A. Hematocrit, hemoglobin, and white blood cell (WBC) count
B. Blood urea nitrogen, electrolytes, and creatinine
C. Glucose, glucose tolerance test, and random blood sugar
D. X-rays, electroencephalogram, and electrocardiogram (ECG)
Answer: B
Explanation:
(A) These are general diagnostic blood studies (usually done on admission), but they are not
reliable indicators of lithium therapy clearance.
(B) These are the primary diagnostic tests to determine kidney functioning. Because lithium
is excreted through the kidneys and because it can be very toxic, adequate renal function must
be ascertained before therapy begins.
(C) These are diagnostic blood tests used to determine the presence of endocrine (not renal)
dysfunction.
(D) These are other types of diagnostic procedures used to determine musculoskeletal, neural,
and cardiac (rather than renal) functioning.
QUESTION 833
A 52-year-old female client is admitted to the hospital in acute renal failure. She has been on
hemodialysis for the past 2 years. Stat arterial blood gases are drawn on the client yielding the
following results: pH 7.30, PCO2 51 mm Hg, HCO3, 18 mEq/L, PaO2, 84 mm Hg. The nurse
would interpret these results as:
A. Compensated metabolic alkalosis
B. Respiratory acidosis
C. Partially compensated metabolic alkalosis
D. Combined respiratory and metabolic acidosis
Answer: D
Explanation:
(A) Compensated metabolic alkalosis would be reflected by the following: pH within normal
limit (7.35–7.45), PCO2 45 mm Hg, HCO3 26 mEq/L.
(B) Respiratory acidosis would be reflected by the following: pH 7.35, PCO2 45 mm Hg,
HCO3 within normal limits (22–26 mEq/L).

(C) Partially compensated metabolic alkalosis would be reflected by the following: pH 7.45,
PCO2 45 mm Hg, HCO3 26 mEq/L.
(D) Combined respiratory and metabolic acidosis would be reflected by the following: pH
7.35, PCO2 45 mm Hg, HCO3 22 mEq/L.
QUESTION 834
The nurse is assessing breath sounds in a bronchovesicular client. She should expect that:
A. Inspiration is longer than expiration
B. Breath sounds are high pitched
C. Breath sounds are slightly muffled
D. Inspiration and expiration are equal
Answer: D
Explanation:
(A) Inspiration is normally longer in vesicular areas.
(B) High-pitched sounds are normal in bronchial.
(C) Muffled sounds are considered abnormal.
(D) Inspiration and expiration are equal normally in this area, and sounds are medium
pitched.
QUESTION 835
Stat serum electrolytes ordered for a client in acute renal failure revealed a serum potassium
level of 6.4. The physician is immediately notified and orders 50 mL of dextrose and 10 U of
regular insulin IV push. The nurse administering these drugs knows the Rationale for this
therapy is to:
A. Remove the potassium from the body by renin exchange
B. Protect the myocardium from the effects of hypokalemia
C. Promote rapid protein catabolism
D. Drive potassium from the serum back into the cells
Answer: D
Explanation:
(A) Sodium polystyrene sulfonate (Kayexalate), a cation exchange resin, exchanges sodium
ions for potassium ions in the large intestine reducing the serum potassium.
(B) Calcium is administered to protect the myocardium from the adverse effects of
hyperkalemia . Serum levels reflect hyperkalemia.

(C) Rapid catabolism releases potassium from the body tissue into the bloodstream. Infection
and hyperthermia increase the process of catabolism.
(D) The administration of dextrose and regular insulin IV forces potassium back into the cells
decreasing the potassium in the serum.
QUESTION 836
A 9-year-old child was in the garage with his father, who was repairing a lawnmower. Some
gasoline ignited and caused an explosion. His father was killed, and the child has splitthickness and full-thickness burns over 40% of his upper body, face, neck, and arms. All of
the following nursing diagnoses are included on his care plan. Which of these nursing
diagnoses should have top priority during the first 24–48 hours postburn?
A. Pain related to tissue damage from burns
B. Potential for infection related to contamination of wounds
C. Fluid volume deficit related to increased capillary permeability
D. Potential for impaired gas exchange related to edema of respiratory tract
Answer: D
Explanation:
(A, B, C) These answers are all correct; however, maintenance of airway is the top priority.
(D) Persons burned about the face and neck during an explosion are also likely to suffer burns
of the respiratory tract, which can lead to edema and respiratory arrest.
QUESTION 837
When planning care for a 9-year-old client, the nurse uses which of the most effective means
of helping siblings cope with their feelings about a brother who is terminally ill?
A. Open discussion and understanding
B. Play-acting out feelings in different roles
C. Storytelling
D. Drawing pictures
Answer: B
Explanation:
(A) When dealing with grief, siblings are usually most comfortable initially with open
discussion.
(B) Assuming different roles allows children to act out their feelings without fear of reprisals
and to gain insight and control.

(C) This method may be helpful, but having the child take an active part through role playing
is more effective.
(D) This technique may be helpful, but being an active participant through role playing is
more effective.
QUESTION 838
The following nursing diagnosis is written for a comatose client with cirrhosis of the liver and
secondary splenomegaly—High risk for injury: Increased susceptibility to bleeding related to:
A. Increased absorption of vitamin K
B. Thrombocytopenia due to hypersplenism
C. Diminished function of the Kupffer cells
D. Increased synthesis of the clotting factors
Answer: B
Explanation:
(A) There is a decreased absorption of vitamin K with cirrhosis of the liver. This decrease
impairs blood coagulation and the formation of prothrombin.
(B) Thrombocytopenia, an increased destruction of platelets, occurs secondary to
hypersplenism.
(C) A diminished function of the Kupffer cells occurs with cirrhosis of the liver, causing the
client to become more susceptible to infections.
(D) A decrease in the synthesis of fibrinogen and clotting factors VII, IX, and X occurs with
cirrhosis of the liver and increases the susceptibility to bleeding.
QUESTION 839
A 27-year-old male client is admitted to the acute care mental health unit for observation. He
has recently lost his job, and his wife told him yesterday that she wants a divorce. The client
is placed on suicide precautions. In assessing suicide potential, the nurse should pay close
attention to the client’s:
A. Level of insight
B. Thought processes
C. Mood and affect
D. Abstracting abilities
Answer: C
Explanation:

(A) Assessing the client’s level of insight is an important part of the mental status exam
(MSE), but it does not reflect suicide potential.
(B) Assessing the client’s thought processes is an important part of the MSE, but it does not
reflect suicide potential.
(C) Assessing the client’s mood and affect is an important part of the MSE, and it can be a
very valuable indicator of suicide potential. Frequently a client who has decided to proceed
with suicide plans will exhibit a suddenly improved mood and affect.
(D) Assessing a client’s abstracting abilities is an important part of the MSE, but it does not
reflect suicide potential.
QUESTION 840
A 35-year-old client has returned to her room following surgery on her right femur. She has
an IV of D5 in one-half normal saline infusing at 125 mL/hr and is receiving morphine sulfate
10–15 mg IM q4h prn for pain. She last voided 51/2 hours ago when she was given her
preoperative medication. In monitoring and promoting return of urinary function after
surgery, the nurse would:
A. Provide food and fluids at the client’s request
B. Maintain IV, increasing the rate hourly until the client voids
C. Report to the surgeon if the client is unable to void within 8 hours of surgery
D. Hold morphine sulfate injections for pain until the client voids, explaining to her that
morphine sulfate can cause urinary retention
Answer: C
Explanation:
(A) Provision of food and fluid promotes bowel elimination. Nutritional needs
postoperatively are determined by the physician, not the client.
(B) Increasing IV fluids postoperatively will not cause a client to void. Any change in rate of
administration of IV fluids should be determined by the physician.
(C) The postoperative client with normal kidney function who cannot void 8 hours after
surgery is retaining urine. The client may need catheterization or medication. The physician
must provide orders for both as necessary.
(D) Although morphine sulfate can cause urinary retention, withholding pain medication will
not ensure that the client will void. The client with uncontrolled pain will probably not be
able to void.

QUESTION 841
An 80-year-old male client with a history of arteriosclerosis is experiencing severe pain in his
left leg that started approximately 20 minutes ago. When performing the admission
assessment, the nurse would expect to observe which of the following:
A. Both lower extremities warm to touch with 2 pedal pulses
B. Both lower extremities cyanotic when placed in a dependent position
C. Decreased or absent pedal pulse in the left leg
D. The left leg warmer to touch than the right leg
Answer: C
Explanation:
(A) This statement describes a normal assessment finding of the lower extremities.
(B) This assessment finding reflects problems caused by venous insufficiency.
(C) Decreased or absent pedal pulses reflect a problem caused by arterial insufficiency.
(D) The leg that is experiencing arterial insufficiency would be cool to touch due to the
decreased circulation.
QUESTION 842
A woman diagnosed with multiple sclerosis is disturbed with diplopia. The nurse will teach
her to:
A. Limit activities which require focusing (close vision)
B. Take more frequent naps
C. Use artificial tears
D. Wear a patch over one eye
Answer: D
Explanation:
(A) Limiting activities requiring close vision will not alleviate the discomfort of double
vision.
(B) Frequent naps may be comforting, but they will not prevent double vision.
(C) Artificial tears are necessary in the absence of a corneal reflex, but they have no effect on
diplopia.
(D) An eye patch over either eye will eliminate the effects of double vision during the time
the eye patch is worn. An eye patch is safe for a person with an intact corneal reflex.

QUESTION 843
A 14-year-old boy has had diabetes for 7 years. He takes 30 U of NPH insulin and 10 U of
regular insulin every morning at 7 AM. He eats breakfast at 7:30 AM and lunch at noon. What
time should he expect the greatest risk for hypoglycemia?
A. 9 AM
B. 1 PM
C. 11 AM
D. 3 PM
Answer: C
Explanation:
(A) This time is incorrect because regular insulin would peak after the teenager has eaten
breakfast.
(B) This time is incorrect because it is after lunch when the NPH peaks.
(C) Regular insulin peaks in 2–3 hours and has a duration of 4–6 hours. NPH insulin’s onset
is 4–6 hours and peaks in 8–16 hours. Blood sugar would peak after meals and be lowest
before meals and during the night.
(D) This time is incorrect because it is before the NPH and after the regular insulin peak
times.
QUESTION 844
A child is to receive atropine 0.15 mg (1/400 g) as part of his preoperative medication. A vial
containing atropine 0.4 mg (1/150 g)/mL is on hand. How much atropine should be given?
A. 0.06 mL
B. 0.38 mL
C. 2.7 mL
D. Information given insufficient to determine the amount of atropine to be administered
Answer: B
Explanation:
(A, C) Information was incorrectly placed in the formula, resulting in an incorrect answer.
(B) The answer is correct. 0.4 mg = 1 mL:0.15 mg 5 = mL 0.4 x = 0.15 x = 0.15/0.4 x = 0.375
or 0.38 mL

(D) Sufficient information is provided to determine the amount of atropine to administer. The
amount of atropine available and the amount of atropine ordered is required to determine the
amount of atropine to be given.
QUESTION 845
Morphine sulfate 4 mg IV push q2h prn for chest pain was ordered for a client in the
emergency room with severe chest pain. The nurse administering the morphine sulfate knows
which of the following therapeutic actions is related to the morphine sulfate?
A. Increased level of consciousness
B. Increased rate and depth of respirations
C. Increased peripheral vasodilation
D. Increased perception of pain
Answer: C
Explanation:
(A) Morphine sulfate, a narcotic analgesic, causes sedation and a decrease in level of
consciousness.
(B) The side effects of morphine sulfate include respiratory depression.
(C) Morphine sulfate causes peripheral vasodilation, which decreases afterload, producing a
decrease in the myocardial workload.
(D) Morphine sulfate alters the perception of pain through an unclear mechanism. This
alteration promotes pain relief.
QUESTION 846
A female baby was born with talipes equinovarus. Her mother has requested that the nurse
assigned to the baby come to her room to discuss the baby’s condition. The nurse knows that
the pediatrician has discussed the baby’s condition with her mother and that an orthopedist
has been consulted but has not yet seen the baby. What should the nurse do first?
A. Call the orthopedist and request that he come to see the baby now.
B. Question the mother and find out what the pediatrician has told her about the baby’s
condition.
C. Tell the mother that this is not a serious condition.
D. Tell the mother that this condition has been successfully treated with exercises, casts,
and/or braces.
Answer: B

Explanation:
(A) The nurse should call the orthopedist after assessing the mother’s knowledge.
(B) The nurse must first assess the knowledge of the parent before attempting any
explanation.
(C) The nurse should assess the mother’s knowledge of the baby’s condition as the first
priority.
(D) This answer is correct, but the priority is B.
QUESTION 847
A client states to his nurse that “I was told by the doctor not to take one of my drugs because
it seems to have caused decreasing blood cells.” Based on this information, which drug might
the nurse expect to be discontinued?
A. Prednisone
B. Timolol maleate (Blocadren)
C. Garamycin (Gentamicin)
D. Phenytoin (Dilantin)
Answer: D
Explanation:
(A) Prednisone is not linked with hematological side effects.
(B) Timolol, a -adrenergic blocker is metabolized by the liver. It has not been linked to blood
dyscrasia.
(C) Gentamicin is ototoxic and nephrotoxic.
(D) Phenytoin usage has been linked to blood dyscrasias such as aplastic anemia. The drug
most commonly linked to aplastic anemia is chloramphenicol (Chlormycetin).
QUESTION 848
The nurse is in the hallway and one of the visitors faints. The nurse should:
A. Sit the victim up and lightly slap his face
B. Elevate the victim’s legs
C. Apply a cool cloth to the victim’s neck and forehead until he recovers
D. Sit the victim up and place the head between the knees
Answer: B
Explanation:
(A) Sitting the client up defeats the goal of re-establishing cerebral blood flow.

(B) Elevating the legs anatomically redirects blood flow to the cerebral.
(C) This strategy is a nice general comfort measure after the victim has regained
consciousness.
(D) This strategy is not as effective a strategy in helping the client to regain consciousness as
elevating the legs.
QUESTION 849
A client with cirrhosis of the liver becomes comatose and is started on neomycin 300 mg q6h
via nasogastric tube. The rationale for this therapy is to:
A. Prevent systemic infection
B. Promote diuresis
C. Decrease ammonia formation
D. Acidify the small bowel
Answer: C
Explanation:
(A) Neomycin is an antibiotic, but this is not the Rationale for administering it to a client in
hepatic coma.
(B) Diuretics and salt-free albumin are used to promote diuresis in clients with cirrhosis of
the liver.
(C) Neomycin destroys the bacteria in the intestines. It is the bacteria in the bowel that break
down protein into ammonia.
(D) Lactulose is administered to create an acid environment in the bowel. Ammonia leaves
the blood and migrates to this acidic environment where it is trapped and excreted.
QUESTION 850
Diabetes mellitus is a disorder that affects 3.1 out of every 1000 children younger than 20
years old. It is characterized by an absence of, or marked decrease in, circulating insulin.
When teaching a newly diagnosed diabetes client, the nurse includes information on the
functions of insulin:
A. Transport of glucose into body cells and storage of glycogen in the liver
B. Glycogenolysis and facilitation of glucose use for energy
C. Glycogenolysis and catabolism
D. Catabolism and hyperglycemia
Answer: A

Explanation:
(A) Lack of insulin causes glycogenolysis, catabolism, and hyperglycemia.
(B) Insulin promotes the conversion of glucose to glycogen for storage and regulates the rate
at which carbohydrates are used by cells for energy.
(C) Insulin is anabolic in nature.
(D) Glucose stimulates protein synthesis within the tissue and inhibits the breakdown of
protein into amino acids.
QUESTION 851
Following a gastric resection, a 70-year-old client is admitted to the postanesthesia care unit.
He was extubated prior to leaving the suite. On arrival at the postanesthesia care unit, the
nurse should:
A. Check airway, feeling for amount of air exchange noting rate, depth, and quality of
respirations
B. Obtain pulse and blood pressure readings noting rate and quality of pulse
C. Reassure the client that his surgery is over and that he is in the recovery room
D. Review physician’s orders, administering medications as ordered
Answer: A
Explanation:
(A) Adequate air exchange and tissue oxygenation depend on competent respiratory function.
Checking the airway is the nurse’s priority action.
(B) Obtaining the vital signs is an important action, but it is secondary to airway
management.
(C) Reorienting a client to time, place, and person after surgery is important, but it is
secondary to airway and vital signs.
(D) Airway management takes precedence over physician’s orders unless they specifically
relate to airway management.
QUESTION 852
A client is scheduled for a magnetic resonance imaging (MRI) to locate a cerebral lesion. It is
important for the nurse to find out if he has a(n):
A. Allergy to seafood
B. History of seizures
C. Movable metal implant

D. Pin or screw in any bone
Answer: C
Explanation:
(A) Iodine is not used as a contrast medium for MRI. It is important to inquire about allergy
to seafood if the client is to have an arteriogram or enhanced computer tomography.
(B) MRI is safe if seizures are under control. It is more important to inquire about movable
metal implants.
(C) Clients with movable metal implants such as shrapnel or aneurysm clips or clients with
permanent pacemakers or implanted pumps can be traumatized during an MRI.
(D) Nonmovable metal prostheses or hardware will not cause trauma during an MRI.
QUESTION 853
A measurable outcome criterion in the nursing care of an adolescent with anorexia nervosa
would be:
A. Accepting her present body image
B. Verbalizing realistic feelings about her body
C. Having an improved perception of her body image
D. Exhibiting increased self-esteem
Answer: B
Explanation:
(A) This outcome criterion is inadequate because the term “accepts is not directly
measurable.”
(B) This outcome criterion is directly measurable because specific goal-related verbalizations
can be heard and verified by the nurse.
(C) “Improved perception of body image is not directly measurable and is therefore open to
many interpretations.”
(D) Although long-term goals for the anorexic client should focus on increased self-esteem,
this outcome criterion (as stated) does not include specific indicators or behaviors for which
to observe.
QUESTION 854
Which of the following should the nurse anticipate receiving as an as-needed order for a
postoperative carotid endarterectomy client?
A. Nifedipine 10 mg SL for B/P 140/90

B. Furosemide 20 mg/PO for decreased urine output
C. Magnesium salicylate to decrease inflammation
D. Nitroglycerin gr 1/150 for chest pain
Answer: A
Explanation:
(A) It is important to maintain a normal to slightly lower pressure to prevent the graft from
blowing and excessive pressure to surgical vascular areas.
(B, C, D) None of these drugs is related to managing the problem at hand. Also, none of the
problems for which these drugs would be indicated is expected with this type of surgery,
except if there is a prior history.
QUESTION 855
An 18-month-old child has been playing in the garage. His mother brings him to a nurse’s
home complaining of his mouth being sore. His lips and mouth are soapy and white, with
small ulcerated areas beginning to form. The child begins to vomit. His pulse is rapid and
weak. The nurse suspects that the child has:
A. Inhaled gasoline fumes
B. Ingested a caustic alkali
C. Eaten construction chalk
D. Lead poisoning
Answer: B
Explanation:
(A, C, D) These agents would not cause ulcerations on mouth and lips.
(B) Strong alkali or acids will cause burns and ulcerations on the mucous membranes.
QUESTION 856
A 35-year-old client is admitted to the hospital for elective tubal ligation. While the nurse is
doing preoperative teaching, the client says, “The anesthesiologist said she was going to give
me balanced anesthesia.” What exactly is that? The best explanation for the nurse to give the
client would be that balanced anesthesia:
A. Is a type of regional anesthesia
B. Uses equal amounts of inhalation agents and liquid agents
C. Does not depress the central nervous system

D. Is a combination of several anesthetic agents or drugs producing a smooth induction and
minimal complications
Answer: D
Explanation:
(A) Regional anesthesia does not produce loss of consciousness and is indicated for excision
of moles, cysts, and endoscopic surgeries.
(B) Varying amounts of anesthetic agents are used when employing balanced anesthesia.
Amounts depend on age, weight, condition of the client, and surgical procedure.
(C) General anesthesia is a drug-induced depression of the central nervous system that
produces loss of consciousness and decreased muscle activity.
(D) Balanced anesthesia is a combination of a number of anesthetic agents that produce a
smooth induction, appropriate depth of anesthesia, and appropriate muscle relaxation with
minimal complications.
QUESTION 857
One afternoon 3 weeks into his alcohol treatment program, a client says to the nurse, “It’s
really not all my fault that I have a drinking problem.” Alcoholism runs in my family. Both
my grandfather and father were heavy drinkers. The nurse’s best response would be:
A. “That might be a problem. Tell me more about them.”
B. “Risk factors can often be controlled by self-responsibility.”
C. “It sounds like you’re intellectualizing your drinking problem.”
D. “Your grandfather and father were both alcoholics?”
Answer: B
Explanation:
(A) Focusing is an effective therapeutic strategy. This response, however, allows the client to
“defocus off” the topic of learning how to accept responsibility for his behavior and future
growth.
(B) The nurse can educate the client about both the “genetic risk for the development of
alcoholism and ways to make long-term healthy lifestyle changes.”
(C) This response is inappropriately confrontational and condescending to the client.
(D) Reflection of content can be an effective verbal therapeutic technique. It is used
inappropriately here.

QUESTION 858
The most important goal in the care plan for a child who was hospitalized with an accidental
overdose would be to:
A. Determine child’s activity pattern
B. Reduce mother’s sense of guilt
C. Instruct parents in use of ipecac
D. Teach parents appropriate safety precautions
Answer: D
Explanation:
(A) This goal is not the most important.
(B) There is always some guilt when an accident occurs; however, the priority is to be sure
future accidents are prevented.
(C) Ipecac is not used for caustic alkali and acid ingestions.
(D) Determining the parent’s knowledge about safety hazards and teaching appropriate
preventive measures are likely to prevent recurrence of accidents.

Document Details

Related Documents

person
Harper Mitchell View profile
Close

Send listing report

highlight_off

You already reported this listing

The report is private and won't be shared with the owner

rotate_right
Close
rotate_right
Close

Send Message

image
Close

My favorites

image
Close

Application Form

image
Notifications visibility rotate_right Clear all Close close
image
image
arrow_left
arrow_right